SlideShare a Scribd company logo
1 of 790
Download to read offline
www.eduwaves360.com | Telegram : @eduwaves360
Unlocked the Medical premiums
Join us on Telegram :
Click here : @eduwaves360
Click here : www.eduwaves360.com
Medical Courses : https://t.me/usmle_study_materials_2
Discussion Group : @usmle_discussion_group
Question # 1
A 53-year-old man comes to the physician because of fatigue, recurrent diarrhea, and an 8-kg (17.6-lb)
weight loss over the past 6 months. He has a 4-month history of recurrent blistering rashes on different
parts of his body that grow and develop into pruritic, crusty lesions before resolving spontaneously.
Physical examination shows scaly lesions in different phases of healing with central, bronze-colored
induration around the mouth, perineum, and lower extremities. Laboratory studies show:
Hemoglobin 10.1 mg/dL
Mean corpuscular volume 85 μm3
Mean corpuscular hemoglobin 30.0 pg/cell
Serum
Glucose 236 mg/dL
Abdominal ultrasonography shows a 3-cm, solid mass located in the upper abdomen. This patient's mass
is most likely derived from which of the following types of cells?
Answer Image
A Gastrointestinal enterochromaffin cells
B Pancreatic α-cells
C Pancreatic β-cells
D Pancreatic δ-cells
www.eduwaves360.com | Telegram : @eduwaves360
Answer Image
E Gastric G-cells
www.eduwaves360.com | Telegram : @eduwaves360
Hint
This patient's rash, known as necrolytic migratory erythema, is strongly associated with his underlying
tumor.
www.eduwaves360.com | Telegram : @eduwaves360
Correct Answer
A - Gastrointestinal enterochromaffin cells
Explanation Why
Carcinoid tumors, which are functional neuroendocrine tumors derived from enterochromaffin cells,
produce serotonin and can manifest with weight loss and recurrent diarrhea. These tumors are
associated with skin lesions caused by niacin deficiency (e.g., casal necklace), but they are not
associated with necrolytic migratory erythema. In addition, this patient lacks other typical
manifestations of a carcinoid tumor, including flushing and wheezing.
B - Pancreatic α-cells
Explanation Why
This patient has a glucagonoma, a functional neuroendocrine tumor derived from pancreatic α-cells.
These tumors secrete glucagon, which increases gluconeogenesis and glycogenolysis and thus results
in hyperglycemia. Other features of glucagonoma include chronic diarrhea, weight loss,
neuropsychiatric symptoms (e.g., depression, dementia, ataxia), deep vein thrombosis, normocytic
anemia (due to anemia of chronic disease and the inhibitory effect of glucagon on erythropoiesis),
and necrolytic migratory erythema. The pathophysiology that underlies the development of
necrolytic migratory erythema is not well understood, but it may be related to direct action of
glucagon on the skin, protein deficiency (because glucagon causes proteolysis), and/or a deficiency
of zinc and fatty acids due to chronic diarrhea.
C - Pancreatic β-cells
Explanation Why
Insulinomas, which are functional neuroendocrine tumors derived from pancreatic β-cells, produce
insulin and manifest with weight gain (not weight loss) and hypoglycemia (not hyperglycemia).
Moreover, they are not associated with necrolytic migratory erythema.
www.eduwaves360.com | Telegram : @eduwaves360
D - Pancreatic δ-cells
Explanation Why
Somatostatinomas, which are functional neuroendocrine tumors derived from pancreatic δ-cells,
secrete somatostatin and can manifest with weight loss and glucose intolerance. However, these
tumors are not associated with necrolytic migratory erythema. In addition, this patient lacks other
typical manifestations of a somatostatinoma, including abdominal pain, steatorrhea, and
cholelithiasis.
E - Gastric G-cells
Explanation Why
Gastrinomas, which are functional neuroendocrine tumors derived from gastric G-cells, secrete
gastrin and can manifest with weight loss and recurrent diarrhea. However, these tumors are not
associated with hyperglycemia or necrolytic migratory erythema. In addition, this patient lacks other
typical manifestations of a gastrinoma, including abdominal pain and dyspepsia.
www.eduwaves360.com | Telegram : @eduwaves360
Question # 2
A previously healthy 41-year-old woman comes to the physician for the evaluation of recurrent episodes
of palpitations and sweating over the past month. Her symptoms typically start after swimming practice
and improve after drinking ice tea and eating some candy. She has also had a 5-kg (11-lb) weight gain
over the past 3 months. She works as a nurse. Physical examination shows no abnormalities. Fasting
serum studies show:
Glucose 38 mg/dL
Insulin 260 μU/mL (N=11–240)
Proinsulin 65 μU/mL (N <20% of total insulin)
C-peptide 5.0 ng/mL (N=0.8–3.1)
Insulin secretagogues absent
Which of the following is the most likely cause of her symptoms?
Answer Image
A Glucagon-producing tumor
B Exogenous administration of insulin
C Binge eating disorder
D Pancreatic β-cell tumor
E Factitious use of sulfonylureas
www.eduwaves360.com | Telegram : @eduwaves360
Answer Image
F Peripheral resistance to insulin
www.eduwaves360.com | Telegram : @eduwaves360
Hint
This patient's condition can be associated with MEN 1 syndrome.
www.eduwaves360.com | Telegram : @eduwaves360
Correct Answer
A - Glucagon-producing tumor
Explanation Why
Glucagon increases blood glucose levels by stimulating gluconeogenesis and glycogenolysis. A
glucagon-producing tumor (glucagonoma) typically manifests with weight loss, necrolytic migratory
erythema, and impaired glucose tolerance or diabetes mellitus (hyperglycemia). However, this
patient presents with weight gain, hypoglycemia, and laboratory evidence of endogenous
hyperinsulinemia (↑ insulin, ↑ proinsulin, ↑ C-peptide).
B - Exogenous administration of insulin
Explanation Why
Inappropriate exogenous administration of insulin (e.g., malingering) can result in weight gain and
symptomatic hypoglycemia that is corrected with the intake of sugary foods, as in this patient. Her
profession as a nurse also gives her easy access to insulin. However, this patient's elevated C-peptide
and proinsulin levels are evidence that the insulin is endogenously produced, as C-peptide is released
during cleavage of insulin from proinsulin.
C - Binge eating disorder
Explanation Why
Binge eating disorder would also present with weight gain and increased endogenous insulin levels
(as well as C-peptide and proinsulin) secondary to increased nutritional intake. However,
hypoglycemia is not a feature of binge eating disorder.
www.eduwaves360.com | Telegram : @eduwaves360
D - Pancreatic β-cell tumor
Explanation Why
Insulin is secreted by pancreatic β-cells and promotes cellular uptake of glucose. Thus, a pancreatic
β-cell tumor (insulinoma) manifests as hypoglycemia, which would explain this patient's post-
exertional autonomic symptoms (sweating and palpitations) that resolve after consuming sugary
foods. These features, together with the documented hypoglycemia are known as Whipple triad, a
characteristic feature of insulinoma. Insulin also stimulates lipogenesis and inhibits lipolysis, which
explains the weight gain in this patient. As C-peptide is released during cleavage of insulin from
proinsulin, the high levels of serum C-peptide and proinsulin on serum analysis confirm that the
hyperinsulinemia is endogenous, making insulinoma the most likely diagnosis.
E - Factitious use of sulfonylureas
Explanation Why
Sulfonylureas are insulin secretagogues. Individuals using these drugs factitiously would present
with weight gain, features of hypoglycemia, and elevated serum insulin, proinsulin, and C-peptide
levels, as seen in this patient. Her profession as a nurse also gives her easy access to sulfonylureas.
However, patients using sulfonylureas would have insulin secretagogues on laboratory analysis,
which is absent in this patient.
F - Peripheral resistance to insulin
Explanation Why
Patients with type 2 diabetes mellitus have peripheral resistance to insulin and may have elevated
insulin, proinsulin, and C-peptide. However, type 2 diabetics will have hyperglycemia – not
hypoglycemia. In addition, untreated type 2 diabetes causes weight loss due to the insulin resistance
since insulin is an anabolic hormone.
www.eduwaves360.com | Telegram : @eduwaves360
Question # 3
A 48-year-old woman comes to the physician for the evaluation of 24-hour blood pressure monitoring
results. Over the last 3 months, she has had intermittent nausea, decreased appetite, and increasing
weakness and fatigue during the day. She has been treated twice for kidney stones within the past year.
Her current medications include lisinopril, amlodipine, and furosemide. She is 178 cm (5 ft 10 in) tall
and weighs 97 kg (214 lb); BMI is 31 kg/m2
. Her blood pressure is 152/98 mm Hg. Physical
examination shows no abnormalities. Serum studies show:
Na+ 141 mEq/L
Cl− 101 mEq/L
K+ 4.5 mEq/L
HCO3
− 24 mEq/L
Calcium 12.9 mg/dL
Creatinine 1.0 mg/dL
Twenty-four-hour blood pressure monitoring indicates elevated nocturnal blood pressure. Further
evaluation is most likely to show which of the following findings?
Answer Image
A
Increased serum aldosterone-to-
renin ratio
B Increased 24-hour urine cortisol
www.eduwaves360.com | Telegram : @eduwaves360
Answer Image
C
Increased serum parathyroid
hormone
D
Decreased serum thyroid-
stimulating hormone
E Decreased renal blood flow
F
Decreased nocturnal oxygen
saturation
G
Increased serum insulin-like growth
factor 1
www.eduwaves360.com | Telegram : @eduwaves360
Hint
This patient has resistant hypertension despite treatment with 3 antihypertensive drugs, which should
raise concern for secondary hypertension. She also has nephrolithiasis (stones), nausea and decreased
appetite (groans), fatigue, and weakness.
www.eduwaves360.com | Telegram : @eduwaves360
Correct Answer
A - Increased serum aldosterone-to-renin ratio
Explanation Why
Elevated serum aldosterone-to-renin ratio indicates primary hyperaldosteronism (Conn syndrome).
Primary hyperaldosteronism can cause secondary hypertension with features of hypokalemia (e.g.,
fatigue and muscle weakness). However, this patient's potassium level is normal. Moreover,
hypercalcemia and symptoms such as recurrent renal stones or gastrointestinal complaints would not
be expected in Conn syndrome.
B - Increased 24-hour urine cortisol
Explanation Why
Elevated 24-hour urine cortisol is diagnostic of Cushing syndrome (hypercortisolism), which can
cause of treatment-resistant secondary hypertension with lethargy and muscle weakness. However, it
typically also presents with central obesity, thin and easily bruisable skin, abdominal striae, and
proximal muscle atrophy, which are not seen in this patient.
www.eduwaves360.com | Telegram : @eduwaves360
C - Increased serum parathyroid hormone
Image
Explanation But
Primary hyperparathyroidism is also a common finding in MEN 1 and MEN 2A. MEN 2A, in
particular, often also involves pheochromocytoma, another possible (concurrent) cause of secondary
hypertension that should be considered in this patient.
Explanation Why
Increased serum parathyroid hormone would be found in primary hyperparathyroidism, a known
cause of secondary hypertension, especially in patients with hypercalcemia, as seen here.
Hypercalcemia, occurring as a result of primary hyperparathyroidism or otherwise, is thought to
cause hypertension by affecting vascular reactivity and interrupting day-night blood pressure
regulation. The most common causes of hypercalcemia are hyperparathyroidism and malignancy.
www.eduwaves360.com | Telegram : @eduwaves360
D - Decreased serum thyroid-stimulating hormone
Image
Explanation Why
Decreased serum thyroid-stimulating hormone (TSH) would indicate hyperthyroidism, which is an
important cause of treatment-resistant secondary hypertension and may present with weakness and
fatigue, as seen here. Hyperthyroidism could also explain this patient's hypercalcemia, as thyroid
hormones have osteoclastic activity. However, decreased appetite is not typical of hyperthyroidism,
and this patient is lacking other symptoms that would be expected in the setting of thyroid hormone
excess (e.g., heat intolerance, weight loss, diarrhea, tachycardia, tremor).
E - Decreased renal blood flow
Explanation Why
Decreased renal blood flow due to renal artery stenosis or fibromuscular dysplasia is a common
www.eduwaves360.com | Telegram : @eduwaves360
cause of treatment-resistant secondary hypertension, which is often accompanied by fatigue and
weakness. Decreased perfusion triggers the renin-angiotensin-aldosterone system and causes
persistent vasoconstriction as well as volume retention. However, renal artery stenosis would not
explain this patient's hypercalcemia or recurrent kidney stones. Furthermore, constantly elevated
aldosterone would lead to hypokalemia, which this patient does not have.
F - Decreased nocturnal oxygen saturation
Explanation Why
Decreased nocturnal oxygen saturation is characteristic of patients with obstructive sleep apnea
(OSA), which can cause secondary hypertension and excessive daytime fatigue. However, OSA
would not explain this patient's hypercalcemia, recurrent kidney stones, nausea, or decreased
appetite.
G - Increased serum insulin-like growth factor 1
Explanation Why
Elevated serum insulin-like growth factor 1 in an adult can indicate acromegaly, which may cause
secondary hypertension. However, acromegaly presents with enlarged hands and feet, deepening of
the voice, coarsened facial features, doughy skin, oligomenorrhea in women, and possible pituitary
mass effect (headache, bilateral hemianopsia).
www.eduwaves360.com | Telegram : @eduwaves360
Question # 4
A 23-year-old woman comes to physician for an annual health maintenance examination. She feels well.
She is 155 cm (5 ft 1 in) tall and weighs 79 kg (174 lb); BMI is 33 kg/m2
. Examination shows a skin
rash over both axillae. A photograph of her left axilla is shown. This patient's skin finding is most likely
associated with which of the following conditions?
Answer Image
A Niacin deficiency
www.eduwaves360.com | Telegram : @eduwaves360
Answer Image
B Gastric malignant lymphoma
C Primary adrenal insufficiency
D Polycystic ovarian syndrome
E Hashimoto thyroiditis
www.eduwaves360.com | Telegram : @eduwaves360
Answer Image
F Pregnancy
www.eduwaves360.com | Telegram : @eduwaves360
Hint
The picture shows acanthosis nigricans, a skin condition that manifests with thickened, velvety,
hyperpigmented plaques and most frequently involves intertriginous sites such as the axillae or the neck.
www.eduwaves360.com | Telegram : @eduwaves360
Correct Answer
A - Niacin deficiency
Image
Explanation But
Pellagra and acanthosis nigricans can both present with a collar around the neck; also referred to as
Casal necklace in the case of pellagra. However, the skin in niacin deficiency looks sun-burnt rather
than velvety as seen here.
Explanation Why
Niacin deficiency causes pellagra, which presents with a symmetrical, thickened, and
hyperpigmented rash. However, the rash typically occurs in areas of the skin exposed to the sunlight
(e.g., face, neck, hands) rather than in the axillae, as seen here. Moreover, further symptoms of
pellagra, such as vomiting, diarrhea, atrophic glossitis, and neurological symptoms are absent in this
patient. Niacin deficiency is not associated with acanthosis nigricans.
www.eduwaves360.com | Telegram : @eduwaves360
B - Gastric malignant lymphoma
Image
Explanation Why
Gastrointestinal malignancies can manifest with acanthosis nigricans. However, if this skin finding
occurs in association with a malignancy, such as paraneoplastic syndrome, it is characterized by
rapid onset, a verrucous or papulous surface, and involvement of atypical sites (e.g., palms, soles, or
mucous membranes), which is not the case in this patient. Moreover, malignant acanthosis nigricans
is associated with gastric adenocarcinoma, not gastric malignant lymphoma. Lastly, gastric cancer is
rare in young adults and this woman has no signs of gastrointestinal malignancy such as weight loss,
anemia, abdominal discomfort, or melena.
www.eduwaves360.com | Telegram : @eduwaves360
C - Primary adrenal insufficiency
Image
Explanation Why
Primary adrenal insufficiency (Addison disease) presents with hyperpigmented skin lesions caused
by the stimulant effects of excess ACTH on the melanocytes which produce melanin (bronze
disease). Unlike the skin finding in this patient, the hyperpigmentation in Addison disease is
typically generalized and often involves the palm creases or oral mucosa. Instead of obesity, patients
with Addison disease present with weight loss and anorexia due to hypocortisolism. Lastly, the
absence of other symptoms such as hypotension and intestinal complaints (nausea, vomiting, and/or
diarrhea), make Addison disease unlikely.
www.eduwaves360.com | Telegram : @eduwaves360
D - Polycystic ovarian syndrome
Image
Explanation But
Similar skin changes can be caused by obesity alone, in which case the disorder is referred to as
pseudoacanthosis nigricans.
Explanation Why
Acanthosis nigricans is commonly seen in disorders characterized by insulin resistance and
hyperinsulinemia (e.g., polycystic ovarian syndrome, obesity, diabetes mellitus, acromegaly).
Elevated levels of insulin stimulate keratinocyte and dermal fibroblast proliferation via interaction
with insulin-like growth factor 1, resulting in epidermal hyperplasia and plaque-like lesions, as seen
in this patient. In addition, insulin resistance in PCOS is responsible for hyperandrogenism (leading
to acne, hirsutism) and anovulation (causing amenorrhea and impaired fertility).
www.eduwaves360.com | Telegram : @eduwaves360
E - Hashimoto thyroiditis
Explanation Why
Hashimoto thyroiditis usually leads to clinical hypothyroidism, which often presents with obesity
and skin abnormalities. However, findings typically include dry, cold, pale, and scaly skin rather than
the skin lesion described in this patient. Furthermore, the absence of additional features of
hypothyroidism such as fatigue, hair loss, constipation, and/or goiter make Hashimoto thyroiditis
unlikely. Hashimoto thyroiditis is not commonly associated with acanthosis nigricans.
F - Pregnancy
Image
Explanation Why
Pregnancy is associated with hyperpigmentation due to increased melanocyte-stimulating hormone
produced by the placenta. Although the axillae may be affected, hyperpigmentation of the linea alba,
www.eduwaves360.com | Telegram : @eduwaves360
the nipples, areola, and the face (i.e., melasma) are far more common. Moreover, the hyperpigmented
macules and patches often seen during pregnancy are not thickened and velvety as seen here.
Pregnancy is not associated with acanthosis nigricans.
www.eduwaves360.com | Telegram : @eduwaves360
Question # 5
A 17-year-old boy with type 1 diabetes mellitus comes to the emergency department because of nausea
and abdominal pain during the past 6 hours. His finger stick blood glucose concentration is 409 mg/dL.
Urinalysis shows the presence of ketone bodies. Which of the following results of arterial blood gas
analysis is most likely before treatment?
pH PaCO2 (mm Hg) HCO3
-
(mEq/L)
A 7.52 52 41
B 7.46 29 20
C 7.28 51 23
D 7.29 30 14
E 7.49 33 24
F 7.34 56 29
Answer Image
A A
B B
C C
www.eduwaves360.com | Telegram : @eduwaves360
Answer Image
D D
E E
F F
www.eduwaves360.com | Telegram : @eduwaves360
Hint
This patient with type 1 diabetes mellitus presenting with abdominal pain, nausea, acute hyperglycemia,
and ketonuria most likely has an increased anion gap.
www.eduwaves360.com | Telegram : @eduwaves360
Correct Answer
A - A
Explanation Why
Increased pH with increased PaCO2 and increased HCO3
-
suggests a metabolic alkalosis, which is
typically caused by severe vomiting, diuretic use, hyperaldosteronism, or Cushing syndrome. This
patient can develop a metabolic alkalosis due to severe vomiting and/or hypovolemia but a different
primary acid-base disorder would be expected.
B - B
Explanation Why
Mildly increased pH with decreased PaCO2 and decreased HCO3
-
suggests a respiratory alkalosis
that has been compensated for by decreased renal reabsorption of bicarbonate (chronic respiratory
alkalosis). Respiratory alkalosis is typically caused by hyperventilation due to anxiety, certain drug
toxicities (e.g., aspirin), or mechanical ventilation. This patient with ketonuria will hyperventilate as
a compensatory response but the primary acid-base disorder is not a respiratory alkalosis.
C - C
Explanation Why
Decreased pH with increased PaCO2 and a normal HCO3
-
suggests a respiratory acidosis that has not
yet been compensated for by the kidneys (acute respiratory acidosis). Respiratory acidosis is
typically caused by airway obstruction (e.g., COPD, bronchial asthma) or hypoventilation due to
respiratory muscle weakness or CNS depression. A different acid-base disorder would be expected in
this patient with ketonuria.
www.eduwaves360.com | Telegram : @eduwaves360
D - D
Image
Explanation Why
This patient with type 1 diabetes mellitus presents with clinical features of diabetic ketoacidosis
(DKA). DKA occurs when insulin deficiency stimulates lipolysis and increases the production of
ketone bodies (e.g., acetoacetic acid) from free fatty acids. The presence of these inorganic acids
lowers pH and increases the anion gap. In order to buffer these acidic ketones, HCO3
-
is consumed.
To compensate for metabolic acidosis, minute ventilation increases and PaCO2 is lowered, resulting
in an arterial blood gas pattern as seen here.
www.eduwaves360.com | Telegram : @eduwaves360
E - E
Explanation Why
Increased pH with decreased PaCO2 and normal HCO3
-
suggests a respiratory alkalosis that has not
yet been compensated for by the kidneys (acute respiratory alkalosis). Respiratory alkalosis is
typically caused by hyperventilation due to anxiety, certain drug toxicities (e.g., aspirin), or
mechanical ventilation. This patient with ketonuria will hyperventilate as a compensatory response
but the primary acid-base disorder is not a respiratory alkalosis.
F - F
Explanation Why
Mildly decreased pH with increased PaCO2 and increased bicarbonate suggests a respiratory acidosis
that has been compensated for by increased renal reabsorption of bicarbonate (chronic respiratory
acidosis). Respiratory acidosis is typically caused by airway obstruction (e.g., COPD, bronchial
asthma) or hypoventilation due to respiratory muscle weakness or CNS depression. A different acid-
base disorder would be expected in this patient with ketonuria.
www.eduwaves360.com | Telegram : @eduwaves360
Question # 6
A 47-year-old woman comes to the physician for a 2-month history of fatigue, intermittent left-sided
flank pain, and diffuse extremity pain. She was treated for a prolactinoma 5 years ago and has had
recurrent antral and duodenal peptic ulcers despite medical therapy. Her only medication is omeprazole.
Physical examination shows a moderately distended abdomen that is diffusely tender to palpation. There
is mild left-sided costovertebral angle tenderness. Serum studies show a calcium concentration of
12 mg/dL, phosphorus concentration of 2 mg/dL, and parathyroid hormone level of 426 pg/mL. An
ultrasound of the abdomen shows multiple kidney stones in the left ureteropelvic junction. This patient
is most likely to have which of the following protein abnormalities?
Answer Image
A Inactivation of RET proto-oncogene
B Altered merlin protein expression
C Altered menin protein
D Mutation of VHL tumor suppressor
E Mutation in C-Kit protein
www.eduwaves360.com | Telegram : @eduwaves360
Hint
This patient has evidence of hyperparathyroidism (hypercalcemia, elevated PTH level) and a history of
recurrent ulcers, which suggests Zollinger-Ellison syndrome. The constellation of parathyroid, pituitary,
and pancreatic disease should raise suspicion for multiple endocrine neoplasia type 1 (MEN1).
www.eduwaves360.com | Telegram : @eduwaves360
Correct Answer
A - Inactivation of RET proto-oncogene
Explanation Why
Pathologic inactivation of the RET gene causes Hirschsprung disease, a congenital abnormality of
the colon that results in bilious emesis, abdominal distention, and failure to pass meconium. It is not
associated with the parathyroid, pituitary, and pancreatic abnormalities seen in this patient.
B - Altered merlin protein expression
Explanation Why
Altered merlin expression is the hallmark of neurofibromatosis type 2, which is characterized by
bilateral acoustic neuromas, juvenile cataracts, and meningiomas. NF2 would not explain the
parathyroid, pancreatic, and pituitary abnormalities seen in this patient.
C - Altered menin protein
Explanation Why
Altered menin protein function resulting from a mutation in the MEN1 gene is seen in multiple
endocrine neoplasia type 1. MEN1 is characterized by the 3 P's: pituitary tumors (prolactinoma),
parathyroid gland tumors (resulting in primary hyperparathyroidism), and pancreatic tumors (such as
gastrinoma or insulinoma). MEN1 is also associated with angiofibromas and meningiomas.
www.eduwaves360.com | Telegram : @eduwaves360
D - Mutation of VHL tumor suppressor
Explanation Why
VHL gene mutations can result in von Hippel-Lindau disease, which is characterized by bilateral
renal cell carcinoma (RCC). RCC may cause secretion of parathyroid hormone-related protein,
which can also lead to symptoms of hyperparathyroidism (as seen in this patient). However, this
patient lacks other distinguishing features of VHL, such as angiomatosis, pheochromocytomas, and
hemangioblastomas.
E - Mutation in C-Kit protein
Explanation Why
Proto-oncogene c-Kit is a cytokine receptor oncogene that can be found in gastrointestinal stromal
tumors (GIST), as well as mastocytomas, seminomas, embryonal carcinomas, and myeloid
leukemias. However, none of these malignancies would explain this patient's constellation of
parathyroid, pituitary, and pancreatic abnormalities.
www.eduwaves360.com | Telegram : @eduwaves360
Question # 7
A 33-year-old woman comes to the physician because of a 4-day history of fever, anterior neck pain, and
throat swelling. She has no history of serious illness. Her temperature is 38.1°C (100.6°F) and pulse is
109/min. Physical examination shows diaphoresis and a fine tremor of the outstretched hands. The
thyroid gland is enlarged, firm, and tender to palpation. Serum thyroid stimulating hormone level is
0.06 μU/mL and erythrocyte sedimentation rate is 65 mm/h. 123
I scan shows an enlarged thyroid gland
with diffusely decreased uptake. Histologic examination of a thyroid biopsy specimen is most likely to
show which of the following findings?
Answer Image
A Follicular epithelial cell hyperplasia
B Undifferentiated giant cells with areas of necrosis and hemorrhage
C Concentric intracellular lamellar calcifications
D Lymphocytic infiltration with germinal follicle formation
E Noncaseating granulomas with multinucleated giant cells
www.eduwaves360.com | Telegram : @eduwaves360
Hint
Signs of hyperthyroidism, a tender and enlarged thyroid gland, and an elevated ESR in a young female
patient are suggestive of De Quervain thyroiditis, a type of subacute thyroiditis.
www.eduwaves360.com | Telegram : @eduwaves360
Correct Answer
A - Follicular epithelial cell hyperplasia
Explanation Why
Diffuse follicular cell hyperplasia can be seen in Graves disease, the most common cause of
hyperthyroidism. Graves disease can also present with symptoms of hyperthyroidism (e.g.,
tachycardia, diaphoresis, resting tremor) and a diffusely enlarged goiter. However, it would not
explain the tenderness of her goiter or her elevated ESR. Furthermore, an iodine uptake scan in
Graves disease would show diffusely increased uptake of radioactive iodine.
B - Undifferentiated giant cells with areas of necrosis and hemorrhage
Explanation Why
Undifferentiated giant cells with areas of necrosis and hemorrhage are characteristic findings in
anaplastic thyroid carcinoma. It is an aggressive malignancy that is most commonly seen in elderly
populations. Anaplastic thyroid carcinoma typically presents with a rapidly enlarging thyroid mass
that may cause dysphagia, and usually lacks the signs or symptoms of hyperthyroidism seen in this
patient.
C - Concentric intracellular lamellar calcifications
Explanation Why
Concentric lamellar calcifications are consistent with psammoma bodies, which can be seen in
papillary thyroid carcinoma. This disease is usually characterized by painless thyroid nodules with
focal areas of decreased uptake on iodine uptake scan. This patient's painful goiter and diffusely
decreased uptake on iodine uptake scan are not consistent with papillary thyroid carcinoma.
www.eduwaves360.com | Telegram : @eduwaves360
D - Lymphocytic infiltration with germinal follicle formation
Explanation Why
Diffuse lymphocytic infiltration with germinal center formation is seen in Hashimoto thyroiditis, the
most common cause of hypothyroidism in the US. Although Hashimoto thyroiditis may initially have
a hyperthyroid phase (e.g., Hashitoxicosis), the disease is typically asymptomatic and characterized
by painless goiter. This patient's tender thyroid exam is not consistent with Hashimoto thyroiditis.
E - Noncaseating granulomas with multinucleated giant cells
Explanation Why
De Quervain thyroiditis is a transient and self-limited disease characterized by patchy granulomatous
inflammation of the thyroid. This disease is often preceded by a viral upper respiratory tract infection
and typically presents initially with hyperthyroidism (as seen in this patient), followed by
hypothyroidism. Classic signs include thyroid tenderness, increased ESR, and jaw pain.
www.eduwaves360.com | Telegram : @eduwaves360
Question # 8
A 27-year-old man comes to the physician because of a 4-month history of unintentional weight gain,
fatigue, and decreased sexual desire. There is no personal or family history of serious illness. His blood
pressure is 149/88 mm Hg. Physical examination shows central obesity and abdominal striae. He has a
prominent soft tissue bulge at the dorsum of his neck. Laboratory studies show a 24-hour urinary free
cortisol of 200 μg (N < 50) and a morning serum ACTH of 1 pg/mL (N = 7–50). Which of the following
tests is most likely to confirm the underlying etiology of this patient's symptoms?
Answer Image
A CRH stimulation test
B ACTH stimulation test
C Low-dose dexamethasone suppression test
D Petrosal vein catheterization
E Chest CT
F Abdominal CT
www.eduwaves360.com | Telegram : @eduwaves360
Answer Image
G Brain MRI
www.eduwaves360.com | Telegram : @eduwaves360
Hint
This patient presents with typical features of Cushing syndrome, including weight gain, truncal obesity,
striae, hypertension, fatigue, and decreased sexual desire. High cortisol levels in conjunction with low
ACTH levels indicate primary hypercortisolism.
www.eduwaves360.com | Telegram : @eduwaves360
Correct Answer
A - CRH stimulation test
Explanation Why
CRH stimulation test can be used in patients with ACTH-dependent hypercortisolism to differentiate
whether the source of excess ACTH production is pituitary (Cushing disease) or ectopic. If serum
ACTH and cortisol levels increase further after CRH stimulation, Cushing disease is confirmed;
minimal or no change in their levels suggests ectopic ACTH production. Since the patient presents
with ACTH-independent hypercortisolism, this test would not be indicated.
B - ACTH stimulation test
Explanation Why
ACTH stimulation test is used to test the response of the adrenal glands to ACTH in patients
suspected of having adrenal insufficiency (e.g., hypocortisolism), such as Addison disease. Since this
patient presents with hypercortisolism, this test would not be indicated.
C - Low-dose dexamethasone suppression test
Explanation But
High-dose dexamethasone suppression test can be used in patients with ACTH-dependent
hypercortisolism to differentiate whether the source of ACTH production is pituitary (Cushing
disease) or ectopic.
Explanation Why
A low-dose dexamethasone suppression test can be used in the initial screening for hypercortisolism.
In normal individuals, dexamethasone suppresses ACTH secretion and thus decreases the level of
cortisol. However, in this patient, the diagnosis of hypercortisolism using a 24–hour urine sample as
www.eduwaves360.com | Telegram : @eduwaves360
well as a decreased ACTH level in serum has already been established, indicating an ACTH-
independent Cushing syndrome. Therefore a low-dose dexamethasone suppression test would not be
helpful in confirming the cause of his symptoms.
D - Petrosal vein catheterization
Explanation Why
Petrosal vein catheterization is performed in patients with suspected ACTH-secreting pituitary
adenoma if brain MRI studies are unable to identify pituitary adenoma but clinical suspicion is still
high. However, ACTH levels in secondary hypercortisolism would either be normal or elevated, as
opposed to this patient's low ACTH levels.
E - Chest CT
Explanation Why
A chest CT would be helpful if small cell lung cancer, which can lead to paraneoplastic, ectopic
ACTH production and subsequent secondary hypercortisolism, were suspected. However, ectopic
ACTH production would present with normal or elevated ACTH levels, as opposed to this patient's
low ACTH. Also, this patient lacks other symptoms of lung cancer, such as cough, hemoptysis,
dyspnea, and chest pain.
www.eduwaves360.com | Telegram : @eduwaves360
F - Abdominal CT
Image
Explanation But
Prolonged glucocorticoid therapy is the most common cause of hypercortisolism.
Explanation Why
This patient presents with symptoms and laboratory findings consistent with ACTH-independent
Cushing syndrome due to autonomous overproduction of cortisol by the adrenal gland. Once the
Cushing's syndrome has been determined to be ACTH-independent, a thin-section CT or MRI of the
adrenal glands should be performed to evaluate for adrenal mass. Common etiologies for excess
adrenal cortisol production include adrenal adenoma, adrenal carcinoma, and adrenal hyperplasia.
www.eduwaves360.com | Telegram : @eduwaves360
G - Brain MRI
Explanation Why
MRI of the brain may be used to visualize pituitary masses in patients with suspected Cushing
disease due to ACTH-secreting pituitary adenoma (e.g., secondary hypercortisolism). However,
ACTH levels in secondary hypercortisolism would either be normal or elevated, as opposed to this
patient's low ACTH levels.
www.eduwaves360.com | Telegram : @eduwaves360
Question # 9
A 45-year-old woman comes to the physician because of fatigue, irregular menses, and recurrent
respiratory infections for the past 6 months. Her blood pressure is 151/82 mm Hg. Physical examination
shows a round face, thinning of the skin, and multiple bruises on her arms. Further studies confirm the
diagnosis of an ACTH-secreting pituitary adenoma. This patient is at greatest risk for which of the
following?
Answer Image
A Weight loss
B Eosinophilia
C Hypoglycemia
D Bitemporal hemianopsia
E Pathologic fracture
F Hyperkalemia
www.eduwaves360.com | Telegram : @eduwaves360
Hint
This patient presents with typical findings of hypercortisolism (e.g., moon facies, bruisable skin,
irregular menstrual cycle, recurrent infections, hypertension) due to an ACTH-secreting pituitary
adenoma (Cushing disease).
www.eduwaves360.com | Telegram : @eduwaves360
Correct Answer
A - Weight loss
Explanation Why
Cushing disease is associated with weight gain rather than weight loss. Patients with Cushing disease
have a characteristic pattern of weight gain and fat deposition in the neck (buffalo hump), trunk, and
abdomen (central obesity), as well as the face (round, moon facies).
B - Eosinophilia
Explanation Why
Eosinophilia is a hematologic finding associated with chronic adrenal insufficiency, not Cushing
syndrome. This patient has a cortisol-secreting pituitary adenoma and clinical features of
hypercortisolism – not hypocortisolism, as would be seen in adrenal insufficiency.
C - Hypoglycemia
Explanation Why
Cushing disease is most commonly associated with hyperglycemia rather than hypoglycemia.
Hypercortisolism raises blood glucose by increasing insulin resistance and hepatic uptake of glucose
to be used in gluconeogenesis.
www.eduwaves360.com | Telegram : @eduwaves360
D - Bitemporal hemianopsia
Explanation Why
Bitemporal hemianopsia is a complication of pituitary macroadenomas (size > 10 mm) such as
prolactinoma, gonadotropinomas, somatotropinomas, and thyrotropinomas. Bitemporal hemianopsia
is caused by compression of the optic nerve at the level of the optic chiasm. While this patient's
hypercortisolism is due to a pituitary adenoma, she is unlikely to develop bitemporal hemianopsia, as
Cushing disease is most commonly caused by microadenomas (size < 10 mm) rather than
macroadenomas.
E - Pathologic fracture
Image
Explanation Why
Cushing disease is associated with pathological fractures due to osteoporosis. Excess cortisol inhibits
www.eduwaves360.com | Telegram : @eduwaves360
calcitriol and osteoblasts maturation while simultaneously increasing production of RANKL, which
stimulates osteoclasts and results in increased levels of bone resorption. Other features of Cushing
disease include cataracts, peptic ulcer disease, dyslipidemia, muscle atrophy/weakness, acne,
hirsutism, and psychiatric conditions such as generalized anxiety disorder, major depressive disorder,
and psychosis.
F - Hyperkalemia
Explanation Why
Cushing disease is associated with hypokalemia rather than hyperkalemia. Hypokalemia is caused by
the mineralocorticoid effect of cortisol on aldosterone receptors, which stimulates sodium and water
reabsorption (causing hypertension), potassium excretion, and metabolic alkalosis. Hyperkalemia is
usually seen in hypocortisolism as part of primary adrenal insufficiency. This patient has
hypercortisolism.
www.eduwaves360.com | Telegram : @eduwaves360
Question # 10
A 38-year-old woman comes to the physician because of a 4-month history of crampy abdominal pain,
recurrent watery diarrhea, and a 2.5-kg (5.5-lb) weight loss. Her husband has noticed that after meals,
her face and neck sometimes become red, and she develops shortness of breath and starts wheezing.
Examination shows a grade 3/6 systolic murmur heard best at the left lower sternal border. The abdomen
is soft, and there is mild tenderness to palpation with no guarding or rebound. Without treatment, this
patient is at greatest risk of developing which of the following conditions?
Answer Image
A Achlorhydria
B Laryngeal edema
C Pigmented dermatitis
D Megaloblastic anemia
E T-cell lymphoma
www.eduwaves360.com | Telegram : @eduwaves360
Hint
Watery diarrhea, crampy abdominal pain, episodic cutaneous flushing triggered by eating or emotional
events, asthma-like attacks, and a murmur indicative of tricuspid regurgitation are highly suggestive of
carcinoid syndrome. The neuroendocrine tumor is highly metabolically active and secretes excess
amounts of serotonin.
www.eduwaves360.com | Telegram : @eduwaves360
Correct Answer
A - Achlorhydria
Explanation Why
Increased gastric pH levels are not associated with carcinoid syndrome but are seen in achlorhydria,
which is a key feature of WDHA syndrome (watery diarrhea, hypokalemia, achlorhydria) seen in
VIPomas. The unregulated production of VIP leads to excessive secretion of water and electrolytes
into the intestinal lumen. Cutaneous flushing is seen in about 20% of patients with VIPoma.
However, abdominal pain is usually mild or absent. Furthermore, this etiology does not explain why
the patient also presents with a tricuspid murmur and asthma, neither of which are typical findings of
VIPomas.
B - Laryngeal edema
Explanation Why
Laryngeal edema is not associated with carcinoid syndrome but commonly occurs in hereditary
angioedema, a bradykinin-mediated reaction that can also cause colicky abdominal pain and
diarrhea. However, episodic cutaneous flushing and a tricuspid regurgitation murmur would not be
expected. Although mast cell-mediated angioedema can cause flushing, it does not present with
abdominal pain or diarrhea.
www.eduwaves360.com | Telegram : @eduwaves360
C - Pigmented dermatitis
Image
Explanation But
In developed countries, pellagra is most commonly seen in patients with alcohol use disorder or as a
complication of malabsorption. It can also occur in Hartnup disease (due to renal and intestinal loss
of tryptophan) or after prolonged use of isoniazid (due to decreased levels of vitamin B6, which is
essential for the production of niacin).
Explanation Why
Dermatitis is a characteristic feature of pellagra, or niacin deficiency; other key findings include
dementia and diarrhea (i.e., the “three Ds”). Dermatitis in patients with pellagra is characterized by a
symmetrical, pigmented, scaly thickening of the skin in sun-exposed areas, such as the limbs and
around the neck (casal necklace). The increased metabolism of serotonin in carcinoid tumors
depletes the amount of tryptophan precursors available for other organic compounds, including
niacin. A lack of niacin leads to decreased production of the coenzymes NAD+ and NADP+, which
are essential in redox reactions throughout cellular metabolism. Diagnosis is confirmed via 24-hour
urine testing for 5-hydroxyindoleacetic acid (5-HIAA), the end product of serotonin metabolism.
www.eduwaves360.com | Telegram : @eduwaves360
D - Megaloblastic anemia
Explanation But
Gastric carcinoid tumors can cause pernicious anemia. However, because these tumors produce
histamine and not serotonin, they lack some of the typical features of metastatic intestinal carcinoids
that are present here (e.g., diarrhea, carcinoid heart disease).
Explanation Why
Megaloblastic anemia is not caused by carcinoid syndrome but is usually due to folate deficiency or
vitamin B12 deficiency and is commonly associated with malnutrition or malabsorption. Specific
causes for folate deficiency include the use of folate antagonists, whereas vitamin B12 deficiency is
seen in pernicious anemia or after gastrectomy. Although malabsorption can occur in carcinoid
syndrome, it is usually not severe enough to cause folate or vitamin B12 deficiency. Another
complication is more likely to affect this patient.
E - T-cell lymphoma
Explanation But
Enteropathy-associated T-cell lymphoma is a complication of ulcerative jejunitis and
adenocarcinoma of the small bowel.
Explanation Why
Enteropathy-associated T-cell lymphoma is not associated with carcinoid syndrome but is a
complication of celiac disease. Although this patient has had weight loss and diarrhea, she lacks
other typical manifestations of celiac disease, such as steatorrhea, malabsorption symptoms (e.g.,
anemia), and dermatitis herpetiformis. Furthermore, celiac disease does not explain this patient's
tricuspid murmur, asthma, and flushing.
www.eduwaves360.com | Telegram : @eduwaves360
Question # 11
A 45-year-old woman comes to the physician because of fatigue, abdominal cramps, watery diarrhea,
and a weight loss of 4 kg (8.8 lb) over the last 4 months. She has recently avoided drinking alcohol and
eating spicy food because it worsens her diarrhea and causes episodes of heart palpitations and
reddening of the face and neck. She takes lisinopril for hypertension. Her temperature is
36.5°C (97.7°F), pulse is 98/min, and blood pressure is 149/90 mm Hg. The abdomen is soft, and there
is mild tenderness to palpation with no guarding or rebound. Laboratory studies show an increased urine
5-hydroxyindoleacetic acid concentration. Further evaluation of this patient will most likely show which
of the following?
Answer Image
A Proximal muscle wasting
B Achlorhydria
C Adrenal medullary mass
D Purple-red colored urine
E Multiple peptic ulcers
F Pulmonic valve stenosis
www.eduwaves360.com | Telegram : @eduwaves360
Hint
This woman has carcinoid syndrome, which manifests with abdominal cramps, diarrhea, and episodic
cutaneous flushing and palpitations triggered by food ingestion. The diagnosis is confirmed by
laboratory testing that shows an increased urine concentration of 5-hydroxyindoleacetic acid, which is a
product of serotonin metabolism.
www.eduwaves360.com | Telegram : @eduwaves360
Correct Answer
A - Proximal muscle wasting
Explanation Why
Proximal muscle wasting is classically seen in patients with Cushing syndrome, which can manifest
with facial plethora and hypertension. However, this condition does not typically cause weight loss
or diarrhea. In addition, this patient does not have the classic Cushingoid appearance (e.g., moon
facies, truncal obesity, and buffalo hump), purplish skin striae, or easy bruising. Moreover,
laboratory findings in Cushing syndrome include increased serum cortisol concentrations, not
increased urine 5-hydroxyindoleacetic acid concentration.
B - Achlorhydria
Explanation Why
Achlorhydria is seen in patients with VIPoma, a condition that manifests with watery diarrhea, tea-
colored stools, and hypokalemia. Although 20% of patients experience flushing symptoms, they
typically do not have abdominal pain. Moreover, VIPoma is associated with increased serum
concentrations of vasoactive intestinal peptide, not increased urine 5-hydroxyindoleacetic acid
concentration.
C - Adrenal medullary mass
Explanation Why
Pheochromocytoma can manifest as an adrenal medullary mass and cause episodic hypertension,
palpitations, tachycardia, and sweating as seen in this patient. However, this malignancy does not
typically cause abdominal cramps or diarrhea. Moreover, laboratory changes associated with
pheochromocytoma include increased urine concentrations of catecholamines and metanephrines, not
5-hydroxyindoleacetic acid.
www.eduwaves360.com | Telegram : @eduwaves360
D - Purple-red colored urine
Explanation Why
Purple-red colored urine is observed in patients with acute intermittent porphyria (AIP), which also
manifests with abdominal cramps, diarrhea, and, during acute attacks, hypertension and tachycardia.
In addition, alcohol can precipitate attacks of AIP. However, affected patients typically experience
neurological symptoms (e.g., sensory and motor neuropathy, neuropsychiatric symptoms), which are
absent in this case. Moreover, laboratory changes associated with AIP include increased serum and
urine concentrations of porphobilinogen, delta-aminolevulinic acid, and porphyrins, not increased
urine 5-hydroxyindoleacetic acid concentration.
E - Multiple peptic ulcers
Explanation Why
Multiple peptic ulcers can result from excessive gastrin secretion due to a gastrinoma, which in
addition manifests with severe abdominal pain, diarrhea, and weight loss as seen in this case.
However, this woman has episodic palpitations and facial flushing triggered by food and alcohol,
which are not features of a gastrinoma. Moreover, gastrinoma is associated with increased serum
concentrations of gastrin and chromogranin A, not increased urine 5-hydroxyindoleacetic acid
concentration.
F - Pulmonic valve stenosis
Explanation Why
Pulmonic valve stenosis is a manifestation of carcinoid heart disease, a condition that eventually
affects > 50% of patients with carcinoid syndrome. Carcinoid heart disease typically develops as a
result of chronic exposure of the heart to serotonin from liver metastases. Serotonin enters the
inferior vena cava and reaches the right side of the heart, where it induces valvular and myocardial
fibrosis. This can lead to pulmonary valve stenosis, tricuspid insufficiency, and right-sided heart
failure.
www.eduwaves360.com | Telegram : @eduwaves360
www.eduwaves360.com | Telegram : @eduwaves360
Question # 12
A previously healthy 27-year-old man comes to the physician because of a 3-week history of anxiety,
diarrhea, and a 4.1-kg (9-lb) weight loss. On questioning, he also reports that he noticed a painless mass
on his left testicle 2 weeks ago. His pulse is 110/min and irregular and blood pressure is 150/70 mm Hg.
Examination shows diaphoresis and a fine tremor of the outstretched fingers. Testicular examination
shows a 3-cm, firm, nontender mass on the left scrotum that does not transilluminate. This patient's
underlying condition is most likely to be associated with which of the following findings?
Answer Image
A Hydrocele on scrotal ultrasound
B Proptosis on exophthalmometry
C Elevated serum AFP
D Elevated serum TSH
E Positive urine metanephrines
F Positive urine pregnancy test
www.eduwaves360.com | Telegram : @eduwaves360
Hint
The incidental finding of a testicular mass that does not transilluminate should raise suspicion for
testicular cancer. Additionally, the patient's symptoms (anxiety, diarrhea, and weight loss) and other
physical examination findings (tachyarrhythmia, hypertension with widened pulse pressure, diaphoresis,
fine tremor) are classic features of hyperthyroidism. Paraneoplastic hyperthyroidism is a possible
complication of testicular choriocarcinomas.
www.eduwaves360.com | Telegram : @eduwaves360
Correct Answer
A - Hydrocele on scrotal ultrasound
Explanation Why
Hydroceles are fluctuant swellings of the scrotum that are rarely firm or well-circumscribed and do
transilluminate. This patient presents with a firm, non-tender, testicular mass that does not
transilluminate, which is not consistent with a hydrocele. Furthermore, hydroceles are not associated
with hyperthyroidism.
B - Proptosis on exophthalmometry
Explanation Why
Proptosis is associated with hyperthyroidism due to Graves disease. While this patient's symptoms
are suggestive of hyperthyroidism, he has no evidence of Graves disease (e.g., diffuse goiter or
pretibial myxedema). Furthermore, Graves disease would not account for his testicular mass.
C - Elevated serum AFP
Explanation Why
Elevated serum AFP levels are seen in men with certain nonseminomatous germ cell tumors, mainly
yolk sac tumors and, less commonly, embryonal carcinoma. While this patient's testicular mass is
highly suggestive of a testicular tumor, AFP-producing testicular tumors would not account for his
symptoms of hyperthyroidism.
www.eduwaves360.com | Telegram : @eduwaves360
D - Elevated serum TSH
Explanation But
Central hyperthyroidism would cause elevated levels of serum TSH, but it would not account for this
patient's testicular mass.
Explanation Why
Elevated serum TSH is seen in primary hypothyroidism. This patient has symptoms of
hyperthyroidism, which would be associated with low serum TSH. High circulating levels of thyroid
hormones in patients with hyperthyroidism suppress secretion of TSH through feedback inhibition.
E - Positive urine metanephrines
Explanation Why
Positive urine metanephrines are seen in pheochromocytoma, which is associated with symptoms of
adrenergic overactivity. The classic symptom triad of pheochromocytoma (episodic headache,
sweating, and tachycardia) overlaps with the symptoms of hyperthyroidism present in this patient.
However, this patient's testicular mass should raise suspicion for testicular cancer, which would not
cause urine metanephrines to be elevated.
F - Positive urine pregnancy test
Explanation But
The alpha subunit of HCG is also identical to the alpha subunits of luteinizing hormone (LH) and
follicle-stimulating hormone (FSH).
Explanation Why
A positive urine pregnancy test indicates the presence of elevated human chorionic gonadotropin
www.eduwaves360.com | Telegram : @eduwaves360
(hCG) in the patient's urine. Testicular tumors can produce elevated circulating levels of hCG.
Because the alpha subunits of hCG and thyroid-stimulating hormone (TSH) are identical and the beta
subunits of these hormones are structurally very similar, hCG is able to weakly stimulate TSH
receptors, resulting in the release of thyroid hormones. In rare cases, conditions with extremely high
levels of hCG (e.g., choriocarcinoma, as is the likely diagnosis in this man, or gestational
trophoblastic disease in women) may be associated with paraneoplastic hyperthyroidism.
www.eduwaves360.com | Telegram : @eduwaves360
Question # 13
A healthy 31-year-old woman comes to the physician because she is trying to conceive. She is currently
timing the frequency of intercourse with at-home ovulation test kits. An increase in the levels of which
of the following is the best indicator that ovulation has occurred?
Answer Image
A Estrogen
B Gonadotropin-releasing hormone
C Follicle stimulating hormone
D Progesterone
E Luteinizing hormone
www.eduwaves360.com | Telegram : @eduwaves360
Hint
The level of this hormone increases right after ovulation. If fertilization and implantation do not occur,
the same hormone decreases in concentration, causing the endometrium to be shed.
www.eduwaves360.com | Telegram : @eduwaves360
Correct Answer
A - Estrogen
Explanation Why
Ovulation is preceded by an increase in estrogen. After ovulation occurs (luteal phase), estrogen
levels decline.
B - Gonadotropin-releasing hormone
Explanation Why
Gonadotropin-releasing hormone (GnRH) is essential for the development and maturation of ovarian
follicles and stimulation of ovulation. GnRH is secreted in a pulsatile manner and is regulated by
estrogen and androgens, which influence the amount and frequency of GnRH pulses. Its plasma
peaks do not coincide with ovulation, which means it is not an appropriate indicator of when
ovulation has occurred.
C - Follicle stimulating hormone
Explanation Why
During the follicular phase of the menstrual cycle, follicle stimulating hormone (FSH) stimulates the
development of several ovarian follicles in the ovaries. FSH levels increase during days 1–7 of the
menstrual cycle, approximately 1 week prior to ovulation. The hormone is not directly involved in
ovulation and therefore is not an appropriate indicator of when ovulation has occurred.
www.eduwaves360.com | Telegram : @eduwaves360
D - Progesterone
Image
Explanation But
Progesterone maintains the endometrium and inhibits the secretion of gonadotropins (FSH and LH)
throughout the luteal phase of the menstrual cycle. If fertilization and implantation occur, the
production of β-hCG maintains the corpus luteum and thus progesterone for the first weeks of
pregnancy.
Explanation Why
Towards the end of the follicular phase of the menstrual cycle, a peak in estrogen levels triggers an
LH surge, which induces ovulation. The corpus luteum then begins to produce progesterone. An
increase in progesterone levels is therefore an indicator that ovulation has occurred.
www.eduwaves360.com | Telegram : @eduwaves360
E - Luteinizing hormone
Explanation Why
Approximately 24–36 hours prior to ovulation, a peak in luteinizing hormone (LH) occurs, which
induces the rupture of the Graafian follicle. Following ovulation, the LH level rapidly decreases.
Since the LH peak is very short and occurs prior to ovulation, it is not a reliable indicator that
ovulation has occurred.
www.eduwaves360.com | Telegram : @eduwaves360
Question # 14
A 34-year-old woman comes to the physician because of a 3-month history of fatigue and a
4.5-kg (10-lb) weight loss despite eating more than usual. Her pulse is 115/min and blood pressure is
140/60 mm Hg. Physical examination shows warm, moist skin, and a diffuse, non-tender swelling over
the anterior neck. Ophthalmologic examination shows swelling of the eyelids and proptosis bilaterally.
Which of the following is the most likely cause of this patient's symptoms?
Answer Image
A
Nongranulomatous
thyroid
inflammation
B
Thyrotropin receptor
autoantibodies
C
Granulomatous
thyroid
inflammation
www.eduwaves360.com | Telegram : @eduwaves360
Answer Image
D
Parafollicular cell
hyperplasia
E
Thyroid peroxidase
autoantibodies
F
Constitutively active
TSH receptor
www.eduwaves360.com | Telegram : @eduwaves360
Hint
This patient presents with clinical findings of hyperthyroidism due to Graves disease, including
unintentional weight loss, warm, moist skin, tachycardia, hypertension, painless goiter, and
ophthalmopathy.
www.eduwaves360.com | Telegram : @eduwaves360
Correct Answer
A - Nongranulomatous thyroid inflammation
Explanation Why
Nongranulomatous inflammation of the thyroid occurs in Hashimoto thyroiditis and subacute
lymphocytic thyroiditis. Patients with either condition may initially present with an enlarged,
painless thyroid gland and signs of hyperthyroidism secondary to transient thyroxine release from
destroyed thyrocytes. However, these conditions are not associated with eyelid swelling or
exophthalmos.
B - Thyrotropin receptor autoantibodies
Image
www.eduwaves360.com | Telegram : @eduwaves360
Explanation But
Graves disease is associated with HLA-DR3 and HLA-B8 immunophenotypes. Histological
examination of the goiter in Graves disease would reveal tall, follicular epithelial cells with scalloped
colloid spaces.
Explanation Why
Graves disease is caused by thyrotropin-receptor autoantibodies (TRAbs) that activate the TSH
receptor. TRAbs induce hyperthyroidism by stimulating follicular epithelial cells to produce
abnormally high amounts of triiodothyronine (T3) and thyroxine (T4). As seen in this patient, the
metabolically active free T3 increases the basal metabolic rate and lipolysis, which manifests as
weight loss, high blood pressure, and increased body temperature. TSH receptor antibodies also bind
to orbital TSH receptors and stimulate adipocyte proliferation and orbital fibroblast secretion of
glycosaminoglycans. Increased adipocyte count, osmotic muscle swelling, and muscle inflammation
cause expansion of retro-orbital tissue, which manifests as exophthalmos and decreased ocular
motility.
C - Granulomatous thyroid inflammation
Image
www.eduwaves360.com | Telegram : @eduwaves360
Explanation Why
De Quervain thyroiditis is associated with granuloma formation and multinucleated giant cells. This
condition may initially manifest with symptoms of hyperthyroidism caused by transient thyroxine
release from destroyed thyrocytes. However, de Quervain thyroiditis is not associated with eyelid
swelling or exophthalmos. Moreover, patients with this condition classically present with tender
goiter and jaw pain.
D - Parafollicular cell hyperplasia
Image
Explanation Why
Medullary thyroid cancer is caused by hyperplasia of parafollicular cells. Patients with this condition
may present with painless swelling of the thyroid. However, medullary thyroid cancer does not
manifest with symptoms of hyperthyroidism, and it is not associated with eye changes.
www.eduwaves360.com | Telegram : @eduwaves360
E - Thyroid peroxidase autoantibodies
Image
Explanation Why
Thyroid peroxidase antibodies prevent the production of thyroid hormone in Hashimoto thyroiditis.
Patients with this condition may initially present with an enlarged, painless thyroid gland and
hyperthyroidism secondary to transient thyroxine release from destroyed thyrocytes (hashitoxicosis).
However, Hashimoto's thyroiditis is not associated with eyelid swelling or exophthalmos.
F - Constitutively active TSH receptor
Explanation Why
Constitutively active thyroid stimulating hormone (TSH) receptors can lead to the development of a
toxic multinodular goiter. Although patients with the condition present with classic symptoms of
hyperthyroidism, they do not have eyelid swelling or exophthalmos. Moreover, the condition
www.eduwaves360.com | Telegram : @eduwaves360
manifests with nodular neck swelling, as opposed to the diffuse neck swelling in this patient.
www.eduwaves360.com | Telegram : @eduwaves360
Question # 15
A 25-year-old woman comes to the physician because of a 4-month history of anxiety and weight loss.
She also reports an inability to tolerate heat and intermittent heart racing for 2 months. She appears
anxious. Her pulse is 108/min and blood pressure is 145/87 mm Hg. Examination shows a fine tremor of
her outstretched hands. After confirmation of the diagnosis, the patient is scheduled for radioactive
iodine ablation. At a follow-up visit 2 months after the procedure, she reports improved symptoms but
new-onset double vision. Examination shows conjunctival injections, proptosis, and a lid lag. Slit-lamp
examination shows mild corneal ulcerations. The patient is given an additional medication that improves
her diplopia and proptosis. Which of the following mechanisms is most likely responsible for the
improvement in this patient's ocular symptoms?
Answer Image
A Inhibition of iodide oxidation
B Elimination of excess fluid
C Decreased uptake of iodine
D Replacement of thyroid hormones
E
Decreased production of
proinflammatory cytokines
www.eduwaves360.com | Telegram : @eduwaves360
Answer Image
F
Inhibition of peripheral T4 to T3
conversion
www.eduwaves360.com | Telegram : @eduwaves360
Hint
Symptoms of thyrotoxicosis (e.g., anxiety, weight loss, heat intolerance, palpitations, tachycardia,
tremor) together with ophthalmopathy (e.g., conjunctival injections, proptosis, lid lag, diplopia) are
highly suggestive of Graves disease. Though several drugs are effective in treating symptoms of
hyperthyroidism, only one addresses the underlying cause of Graves ophthalmopathy.
www.eduwaves360.com | Telegram : @eduwaves360
Correct Answer
A - Inhibition of iodide oxidation
Explanation Why
Thioamides (e.g., methimazole and propylthiouracil) inhibit iodine oxidation by blocking thyroid
peroxidase. These drugs are used to achieve a euthyroid state in patients with mild hyperthyroidism,
pregnant patients (PTU in the first trimester and methimazole in the second and third trimesters), in
elderly patients with limited life expectancy, and to rapidly lower circulating thyroid hormone levels
prior to radioactive iodine ablation or thyroidectomy. Although thioamides help to control this
patient's hyperthyroidism-related symptoms, they are not used in the management of ocular
symptoms in Graves ophthalmopathy.
B - Elimination of excess fluid
Explanation Why
Diuretics are used to eliminate excess fluid in patients who are hypervolemic. Although one of the
main causes of Graves ophthalmopathy is the accumulation of osmolar glycosaminoglycan and
subsequent fluid retention in the orbital tissues, diuretics only have a small effect on orbital edema
and thus should only be used as an adjunct to first-line therapy.
C - Decreased uptake of iodine
Explanation Why
Perchlorate competitively inhibits iodoine uptake in the thyroid gland and was previously used to
treat hyperthyroidism. However, it is no longer recommended due to the high risk of developing
aplastic anemia and nephrotic syndrome. Furthermore, perchlorate is not used in the management of
Graves ophthalmopathy.
www.eduwaves360.com | Telegram : @eduwaves360
D - Replacement of thyroid hormones
Explanation Why
Levothyroxine is a synthetic thyroid hormone used to treat hypothyroidism. Depending on the dose
of radioactive iodine administered and the corresponding extent of thyroid tissue destruction, patients
who were previously hyperthyroid might develop hypothyroidism and require levothyroxine
following treatment with radioactive iodine. However, levothyroxine is not used in the management
of Graves ophthalmopathy.
E - Decreased production of proinflammatory cytokines
Image
Explanation But
Radioactive iodine ablation is associated with the development or exacerbation of Graves
ophthalmopathy due to the increased levels of anti-TSH receptor antibodies following treatment.
www.eduwaves360.com | Telegram : @eduwaves360
Explanation Why
Glucocorticoids such as prednisolone are the preferred therapy for moderate to severe Graves
ophthalmopathy, which results from inflammation and soft tissue enlargement in the orbit. In Graves
disease, TSH receptor antibodies stimulate orbital fibroblasts to secrete glycosaminoglycan (GAG)
and the adipocytes to proliferate. GAGs are hyperosmolar, drawing fluid out into the interstitial
space, which causes edema of the orbital tissues. GAG accumulation, edema, and increased
adipocytes increase the orbital pressure and push the globe forwards, causing exophthalmos,
compression of the extraocular muscles, ophthalmoplegia (inability to converge eyes, diplopia,
difficulty reading), and ocular pain. Glucocorticoids improve these symptoms via their anti-
inflammatory and immunosuppressive effects.
F - Inhibition of peripheral T4 to T3 conversion
Explanation Why
Propylthiouracil, glucocorticoids, and certain beta-blockers (e.g., propranolol) inhibit peripheral 5'-
deiodinase, which catalyzes the peripheral conversion of prohormone T4 to active T3 (which is 3–5
times more potent than T4). Although these medications help to control this patient's
hyperthyroidism-related symptoms, inhibition of the peripheral conversion of T4 to T3 plays no role
in the treatment of ocular symptoms in Graves ophthalmopathy.
www.eduwaves360.com | Telegram : @eduwaves360
Question # 16
A 75-year-old man with a seizure disorder is brought to the emergency department by a friend because
of progressive confusion over the past two weeks. He is unable to provide any history. His vital signs are
within normal limits. He appears lethargic and is only oriented to person. Oral mucosa is moist. There is
no jugular venous distention. A basic metabolic panel shows a serum sodium concentration of
115 mEq/L but is otherwise normal. Serum osmolality is low and antidiuretic hormone level is elevated.
X-ray of the chest shows no abnormalities. Which of the following is the most likely cause of this
patient’s hyponatremia?
Answer Image
A Low cardiac output
B Insulin deficiency
C Aldosterone deficiency
D Medication effect
E Excess cortisol
www.eduwaves360.com | Telegram : @eduwaves360
Hint
This patient is euvolemic and has a low serum sodium level with high ADH levels, suggesting a
diagnosis of SIADH.
www.eduwaves360.com | Telegram : @eduwaves360
Correct Answer
A - Low cardiac output
Explanation Why
Low cardiac output (e.g., due to congestive heart failure) results in hypervolemic hyponatremia due
to low renal perfusion and subsequent activation of RAAS and ADH secretion, which in turn
stimulates water (and sodium) reabsorption. This patient lacks other signs of low cardiac output,
including pitting edema, pulmonary edema, and/or elevated JVP, making this etiology unlikely.
B - Insulin deficiency
Explanation Why
Hyperglycemia (e.g., from type 1 diabetes mellitus) can cause both hypovolemic and/or dilutional
hyponatremia. Hypovolemic hyponatremia is a characteristic feature of diabetic ketoacidosis and
occurs due to the osmotic diuresis from hyperglycemia and glucosuria. In dilutional hyponatremia,
increased serum osmolality from hyperglycemia causes water to shift from the intracellular to the
extracellular space, resulting in a decreased serum sodium concentration. This patient's normal
metabolic panel and euvolemia on exam make hyponatremia due to hyperglycemia very unlikely.
C - Aldosterone deficiency
Explanation Why
Aldosterone deficiency (e.g., due to adrenal insufficiency, type 4 RTA, or congenital
hypoaldosteronism) can also manifest with hyponatremia and elevated ADH due to decreased
sodium resorption in the collecting ducts. This patient does not have characteristic signs or
symptoms of aldosterone deficiency, such as hyperkalemia, metabolic acidosis, or hypotension.
www.eduwaves360.com | Telegram : @eduwaves360
D - Medication effect
Explanation Why
SIADH is an adverse effect of several medications, including carbamazepine, SSRIs, and
cyclophosphamide and is characterized by an inappropriately elevated serum ADH despite low
plasma osmolality (e.g., the patient is not hypovolemic). Risk factors for developing medication-
induced SIADH include older age (> 65 years old), concomitant use of other antiseizure drugs and
diuretics, and a history of hyponatremia. Other causes of SIADH include CNS disorders, pulmonary
disease, and ectopic production of ADH (e.g., small cell lung cancer).
E - Excess cortisol
Explanation Why
Increased levels of cortisol (e.g., Cushing syndrome) classically manifest with hypernatremia, not
hyponatremia. Other characteristic laboratory findings include hypocalcemia, hyperglycemia, and
hypokalemia.
www.eduwaves360.com | Telegram : @eduwaves360
Question # 17
A 78-year-old woman is brought to the physician by her son because of progressive memory loss for the
past year. She feels tired and can no longer concentrate on her morning crossword puzzles. She has
gained 11.3 kg (25 lb) in the last year. Her father died from complications of Alzheimer disease. She has
a history of drinking alcohol excessively but has not consumed alcohol for the past 10 years. Vital signs
are within normal limits. She is oriented but has short-term memory deficits. Examination shows a
normal gait and delayed relaxation of the achilles reflex bilaterally. Her skin is dry, and she has brittle
nails. Which of the following is the most likely underlying etiology of this woman’s memory loss?
Answer Image
A Thiamine deficiency
B Autoimmune thyroid disease
C Normal pressure hydrocephalus
D Vitamin B12 deficiency
E Alzheimer disease
www.eduwaves360.com | Telegram : @eduwaves360
Hint
In the early stages of this condition, patients may present with weight loss, heat intolerance, and fine
tremor.
www.eduwaves360.com | Telegram : @eduwaves360
Correct Answer
A - Thiamine deficiency
Explanation Why
Thiamine deficiency is the cause of Wernicke-Korsakoff syndrome, which can also result in short-
term memory impairment. However, this patient lacks other typical findings, including ataxia and
nystagmus. Although thiamine deficiency usually occurs in the setting of alcohol use disorder, this
woman's history of alcohol use is remote and an unlikely cause of her current symptoms.
B - Autoimmune thyroid disease
Image
Explanation Why
Hashimoto thyroiditis is the most common cause of hypothyroidism in the United States and is
www.eduwaves360.com | Telegram : @eduwaves360
characterized by progressive destruction of thyroid tissue by antithyroid peroxidase antibodies and
antithyroglobulin antibodies. Initially, it may manifest with transient hyperthyroidism, before
progressing to hypothyroidism. Hypothyroidism results in a generalized decrease of the basal
metabolic rate, which can present with weight gain, fatigue, dry skin and brittle nails, along with
concentration and memory impairment.
C - Normal pressure hydrocephalus
Explanation Why
Normal pressure hydrocephalus classically presents with the triad of urinary incontinence, memory
changes, and a wide-based, slow gait. Although normal pressure hydrocephalus can cause
progressive memory loss, it is an unlikely diagnosis in a patient without urinary incontinence and
gait disturbances. Furthermore, this condition would not explain her other findings (weight gain, dry
skin, brittle nails).
D - Vitamin B12 deficiency
Explanation Why
Vitamin B12 deficiency can cause impaired memory and should be considered in patients with a
history of alcohol use disorder. However, this patient has not consumed alcohol in the last 10 years.
Moreover, the absence of other classic findings, such as anemia, peripheral neuropathy, and gait
abnormalities, make this an unlikely cause in this patient.
E - Alzheimer disease
Explanation Why
Alzheimer disease is characterized by amyloid protein accumulation and also affects short-term
memory and cognitive function. Although this patient's positive family history increases her risk, this
condition does not account for her other findings, such as weight gain, skin and nail changes, and
delayed relaxation phase of deep tendon reflexes.
www.eduwaves360.com | Telegram : @eduwaves360
Question # 18
A 42-year-old man comes to the physician for a follow-up examination. His blood pressure was
146/91 mm Hg at his appointment 1 month ago; subsequent home blood pressure measurements have
ranged from 135/83 mm Hg to 156/96 mm Hg. His blood pressure today is 141/85 mm Hg. Physical
examination shows no abnormalities. Pharmacotherapy with lisinopril is initiated. Administration of this
drug is most likely to result in decreased activity of which of the following sections of a normal adrenal
gland?
Answer Image
A A
www.eduwaves360.com | Telegram : @eduwaves360
Answer Image
B B
C C
D D
E E
www.eduwaves360.com | Telegram : @eduwaves360
Hint
Lisinopril is an angiotensin-converting enzyme (ACE) inhibitor that blocks the conversion of
angiotensin I to angiotensin II. Angiotensin II is the primary regulator of aldosterone production in the
adrenal cortex.
www.eduwaves360.com | Telegram : @eduwaves360
Correct Answer
A - A
Explanation Why
This is the fibrous adrenal capsule, which keeps the organ mechanically stable. Angiotensin II does
not regulate the activity of the capsule.
B - B
Image
Explanation Why
The zona glomerulosa of the adrenal cortex is the outermost layer of adrenal parenchyma and lies
directly beneath the adrenal capsule (A). The cells of the zona glomerulosa are stimulated by
angiotensin II to produce mineralocorticoids, most importantly aldosterone. Inhibition of ACE by
www.eduwaves360.com | Telegram : @eduwaves360
lisinopril leads to decreased availability of angiotensin II, in turn resulting in decreased activity of
the zona glomerulosa. The response of the zona glomerulosa to angiotensin II is attenuated by low
serum potassium or high serum sodium levels.
C - C
Explanation Why
The zona fasciculata is the middle portion of adrenal cortical parenchyma. The cells of the zona
fasciculata are stimulated by adrenocorticotropic hormone (ACTH) to produce glucocorticoids,
including cortisol. Inhibition of ACE and subsequent changes in angiotensin II levels are not
involved in ACTH secretion. The activity of the zona fasciculata is therefore not affected by
lisinopril therapy.
D - D
Explanation Why
The zona reticularis is the innermost portion of the adrenal cortical parenchyma. The cells of the
zona reticularis are stimulated by adrenocorticotropic hormone (ACTH) to produce androgens,
including androstenedione and dehydroepiandrosterone. Inhibition of ACE and subsequent changes
in angiotensin II levels are not involved in ACTH secretion. The activity of the zona reticularis is
therefore not affected by lisinopril therapy.
www.eduwaves360.com | Telegram : @eduwaves360
E - E
Image
Explanation Why
The adrenal medulla is the innermost part of the adrenal gland and contains chromaffin cells that
secrete norepinephrine and epinephrine when triggered by the cholinergic sympathetic preganglionic
nerve fibers lying in close proximity. The inhibition of ACE and the subsequent changes in
angiotensin II levels do not directly affect acetylcholine stimulation of the adrenal medulla.
www.eduwaves360.com | Telegram : @eduwaves360
Question # 19
A 51-year-old woman comes to the physician because of a 3-month history of fatigue, increased urinary
frequency, and low back pain. She reports frequent passing of hard stools, despite using stool softeners.
During this time, she has not been as involved with her weekly book club. Her family is concerned that
she is depressed. She has no history of serious illness. She has smoked 1 pack of cigarettes daily for the
past 20 years. Her pulse is 71/min and blood pressure is 150/90 mm Hg. Abdominal examination shows
right costovertebral angle tenderness. The patient's symptoms are most likely caused by hyperplasia of
which of the following?
Answer Image
A
Chief cells in the parathyroid
gland
B
Chromaffin cells in the adrenal
gland
C
Chromophobic cells in the
pituitary gland
D
Parafollicular cells in the thyroid
gland
E Spindle cells in the kidney
www.eduwaves360.com | Telegram : @eduwaves360
Answer Image
F Kulchitsky cells in the lung
www.eduwaves360.com | Telegram : @eduwaves360
Hint
This patient presents with classic features of hypercalcemia such as fatigue, hypertension, polyuria
(“thrones”), abdominal and lower back pain (“groans, bones”), costovertebral angle tenderness (renal
“stones”), constipation, and depression (“psychiatric overtones”).
www.eduwaves360.com | Telegram : @eduwaves360
Correct Answer
A - Chief cells in the parathyroid gland
Image
Explanation Why
Parathyroid chief cells are the site of parathyroid hormone (PTH) production and secretion. Under
physiological conditions, parathyroid chief cells maintain calcium homeostasis through
PTH-mediated increases in bone resorption, renal calcium absorption, and phosphate excretion. This
patient's hypercalcemia is most likely caused by excessive production and secretion of parathyroid
hormone (primary hyperparathyroidism). The most common causes of primary hyperparathyroidism
include parathyroid adenoma (approx. 80% of cases) and hyperplasia (approx. 15% of cases).
www.eduwaves360.com | Telegram : @eduwaves360
B - Chromaffin cells in the adrenal gland
Explanation Why
Chromaffin cells in the adrenal gland are the site of catecholamine release (e.g., epinephrine and
norepinephrine). Hyperplasia of chromaffin cells is associated with pheochromocytoma, which
commonly manifests as hypertension, which is seen here. This patient, however, lacks other classic
features of pheochromocytoma, such as paroxysmal headaches, diaphoresis, and palpitations.
C - Chromophobic cells in the pituitary gland
Explanation Why
Chromophobic cell hyperplasia in the pituitary gland is typically seen with non-secretory pituitary
adenomas. Destruction of normal pituitary tissue by a large pituitary adenoma can lead to reduced
production of FSH/LH, GH, ACTH, TSH, and ADH (panhypopituitarism). Hypothyroidism caused
by TSH deficiency could result in fatigue, constipation, depression, and musculoskeletal pain.
Diabetes inspidius caused by ADH deficiency would result in increased frequency of micturition due
to polyuria. However, this patient's hypertension and abdominal pain are not explained by
hypopituitarism, and clinical features of a large pituitary tumor (e.g., bitemporal hemianopsia,
headache) are not present.
D - Parafollicular cells in the thyroid gland
Explanation But
Although calcitonin reduces osteoclastic activity and prevents hypercalcemia, an excess of calcitonin
is not known to cause hypocalcemia because of the compensatory increase in PTH secretion, which
has a greater effect on calcium homeostasis. Rarely, excess calcitonin can result in cutaneous
flushing and diarrhea.
www.eduwaves360.com | Telegram : @eduwaves360
Explanation Why
Parafollicular cell hyperplasia is associated with medullary thyroid carcinoma, an aggressive thyroid
malignancy. Parafollicular cells (C cells) of the thyroid gland are the site of calcitonin production
and secretion. Calcitonin excess, however, is typically asymptomatic. Also, this patient does not have
symptoms suggestive of a thyroid malignancy (e.g., thyroid swelling, cervical lymphadenopathy,
vocal cord palsy).
E - Spindle cells in the kidney
Explanation Why
Sheets of spindle cells in the kidney can be seen in patients with angiomyolipoma. However,
angiomyolipoma is not associated with paraneoplastic hypercalcemia. Renal cell carcinoma, which
can cause paraneoplastic hypercalcemia, is typically characterized by polygonal clear cells on
biopsy.
F - Kulchitsky cells in the lung
Explanation Why
Kulchitsky cells (K cells) are neuroendocrine bronchial cells that can undergo malignant
transformation to become small cell lung cancer (SCLC). SCLC is associated with paraneoplastic
syndromes, which could explain some of the features seen in this patient (e.g., fatigue due to
paraneoplastic SIADH-induced hyponatremia). However, unlike squamous cell lung carcinoma,
SCLC is not associated with paraneoplastic hypercalcemia.
www.eduwaves360.com | Telegram : @eduwaves360
Question # 20
A 36-year-old woman, gravida 2, para 1, at 30 weeks' gestation comes to the physician for evaluation of
increased urinary frequency. She has no history of major medical illness. Physical examination shows no
abnormalities. Laboratory studies show an increased serum C-peptide concentration. Ultrasonography
shows polyhydramnios and a large for gestational age fetus. Which of the following hormones is
predominantly responsible for the observed laboratory changes in this patient?
Answer Image
A Estrogen
B Human placental lactogen
C Thyroxine
D Human chorionic gonadotropin
E Adrenocorticotropic hormone
F Progesterone
www.eduwaves360.com | Telegram : @eduwaves360
Hint
An increased serum C-peptide concentration and ultrasound findings of polyhydramnios and
macrosomia suggest a diagnosis of gestational diabetes. Hyperglycemia causes osmotic diuresis, which
leads to increased urinary frequency.
www.eduwaves360.com | Telegram : @eduwaves360
Correct Answer
A - Estrogen
Explanation Why
Estrogen may play a role in the development of gestational diabetes in pregnancy via increased
insulin resistance, but it is not predominantly responsible for the increased serum C-peptide
concentration in this patient. The main functions of estrogen during pregnancy are to promote uterine
growth and proliferation and differentiation of mammary glands.
B - Human placental lactogen
Explanation But
Screening for gestational diabetes is recommended for all pregnant women because this condition
increases the risk of maternal and fetal complications such as preeclampsia and birth trauma. Most
patients are usually asymptomatic. Diagnosis and appropriate therapy of gestational diabetes mellitus
can decrease fetal and maternal morbidity.
Explanation Why
Human placental lactogen causes pancreatic beta-cell hyperplasia and leads to an increase in insulin
(and C-peptide) secretion as well as maternal insulin resistance. This ensures adequate glucose
availability for the fetus. If maternal pancreatic function does not overcome insulin resistance,
patients can develop gestational diabetes. In response to increased serum glucose concentrations,
fetal production of insulin increases, which leads to increased fetal growth (macrosomia) as seen in
this case.
www.eduwaves360.com | Telegram : @eduwaves360
C - Thyroxine
Explanation Why
Thyroxine is not responsible for the increased serum C-peptide concentration in this patient and does
not play a role in the development of gestational diabetes mellitus. Thyroxine, produced by the
maternal thyroid gland and the fetal thyroid gland after 12 weeks of gestation, is vital for fetal
neurologic development and CNS myelination.
D - Human chorionic gonadotropin
Explanation Why
Human chorionic gonadotropin is not responsible for the increased serum C-peptide concentration in
this patient and does not play a role in the development of gestational diabetes mellitus. The main
function of hCG in early pregnancy is to maintain the corpus luteum, which secretes progesterone.
Around 7 to 9 weeks of gestation, the placenta becomes the primary producer of progesterone.
E - Adrenocorticotropic hormone
Explanation Why
Adrenocorticotropic hormone is not responsible for the increased serum C-peptide concentration in
this patient and does not play a role in the development of gestational diabetes mellitus. During
pregnancy, ACTH stimulates the secretion of glucocorticoids in the maternal adrenal cortex, which
promotes surfactant production and the development of striae gravidarum.
www.eduwaves360.com | Telegram : @eduwaves360
F - Progesterone
Explanation Why
Progesterone may play a role in the development of gestational diabetes in pregnancy via increased
insulin resistance, but it is not predominantly responsible for the increased serum C-peptide
concentration in this patient. The main functions of progesterone during gestation are to maintain
pregnancy, prevent menstruation, induces closure of the cervix, inhibit uterine contractions, and
stimulate the expression of uterine oxytocin receptors prior to delivery.
www.eduwaves360.com | Telegram : @eduwaves360
Question # 21
A 26-year-old primigravid woman at 25 weeks' gestation comes to the physician for a prenatal visit. She
has no history of serious illness and her only medication is a daily prenatal vitamin. A 1-hour 50-g
glucose challenge shows a glucose concentration of 167 mg/dL (N < 135). A 100-g oral glucose
tolerance test shows glucose concentrations of 213 mg/dL (N < 180) and 165 mg/dL (N < 140) at
1 and 3 hours, respectively. If she does not receive adequate treatment for her condition, which of the
following complications is her infant at greatest risk of developing?
Answer Image
A Restricted growth
B Elevated calcium levels
C Islet cell hyperplasia
D Decreased amniotic fluid production
E Omphalocele
F Decreased hematocrit
www.eduwaves360.com | Telegram : @eduwaves360
Hint
Glucose readily crosses the placenta, meaning that elevated maternal glucose levels lead to elevated fetal
blood glucose and, accordingly, cause physiologic changes that can lead to complications in the newborn
infant.
www.eduwaves360.com | Telegram : @eduwaves360
Correct Answer
A - Restricted growth
Explanation Why
Infants born to mothers with pregestational diabetes, not gestational diabetes, are at risk for
intrauterine growth restriction. Infants born to mothers with gestational diabetes are at increased risk
for macrosomia due to the anabolic effects of high insulin levels.
B - Elevated calcium levels
Explanation Why
Infants of diabetic mothers are at risk for hypocalcemia, not hypercalcemia. Although the exact
mechanism is not fully understood, this type of hypocalcemia may be due to abnormal maternal
calcium-phosphorus metabolism, and/or decreased maternal vitamin D concentrations. Infants of
diabetic mothers are also at risk for hypomagnesemia due to maternal renal loss of magnesium from
glycosuria. These low levels of magnesium can impair parathyroid hormone release, resulting in
hypoparathyroidism that may also contribute to hypocalcemia.
C - Islet cell hyperplasia
Explanation Why
Hyperplasia of islet cells, the pancreatic cells responsible for producing insulin, can occur in utero in
response to elevated maternal blood glucose levels, e.g., due to in gestational diabetes. This can
cause infantile hyperinsulinemia and put the infant at risk for hypoglycemia once the maternal
placental transfer of glucose ceases after birth.
www.eduwaves360.com | Telegram : @eduwaves360
D - Decreased amniotic fluid production
Explanation Why
Infants born to diabetic mothers have an increased risk for polyhydramnios, or increased amniotic
fluid, not oligohydramnios. Fetal hyperglycemia results in fetal polyuria and, thus, increased
amniotic fluid production.
E - Omphalocele
Explanation Why
Untreated diabetes during pregnancy can have teratogenic effects, putting infants at increased risk for
congenital malformations including hypertrophic cardiomyopathy and small left colon syndrome.
They are not, however, at increased risk for omphalocele. Pregestational diabetes (but not gestational
diabetes) puts infants at increased risk of caudal regression syndrome.
F - Decreased hematocrit
Explanation Why
Infants born to diabetic mothers are at increased risk for polycythemia (Hct > 65%), not anemia.
Fetal hyperinsulinemia results in an increased metabolic demand that can lead to fetal hypoxia and
increased RBC production to compensate. Infants with polycythemia are at increased risk for RBC
sludging, ischemia, and infarction.
www.eduwaves360.com | Telegram : @eduwaves360
Question # 22
A 15-year-old girl comes to the physician because of a 2-month history of progressive fatigue and
weakness. She also reports recurrent headaches for 2 years, which have increased in severity and
frequency. Her blood pressure is 185/95 mm Hg. Serum studies show a morning renin activity of
130 ng/mL per hour (N=1–4), a morning aldosterone concentration of 60 ng/dL (N=5-30), and a
potassium concentration of 2.9 mEq/L. Further evaluation is most likely to show which of the
following?
Answer Image
A ACTH-producing growth in the pituitary gland
B Increased urinary excretion of metanephrines
C Increased 17-hydroxyprogesterone levels
D Pleomorphic modified smooth muscle cells in the renal cortex
E Involution of zona glomerulosa of the adrenal gland
www.eduwaves360.com | Telegram : @eduwaves360
Hint
The findings of hypertension and elevated morning renin and aldosterone concentrations, in a patient
presenting with fatigue, weakness, and headache, is diagnostic of secondary hyperaldosteronism.
www.eduwaves360.com | Telegram : @eduwaves360
Correct Answer
A - ACTH-producing growth in the pituitary gland
Explanation Why
Cushing disease owing to an ACTH-producing pituitary adenoma can present with hypertension and
signs of hypokalemia, as seen in this patient, as cortisol also has mineralocorticoid properties.
However, not only would affected individuals likely exhibit additional features of Cushing disease
(e.g., central and neck obesity, easy bruising, striae), but they would also have low aldosterone and
renin levels in contrast to this patient.
B - Increased urinary excretion of metanephrines
Explanation Why
Elevated urinary metanephrine levels are suggestive of pheochromocytoma, which would also cause
hypertension and headache. However, patients would rather exhibit an episodic pattern with
asymptomatic episodes in between. Moreover, aldosterone and renin are not affected by
pheochromocytoma, as metanephrines are responsible for the classic symptoms.
C - Increased 17-hydroxyprogesterone levels
Explanation Why
Increased 17-hydroxyprogesterone levels can be found in patients with congenital adrenal
hyperplasia due to 21-β-hydroxylase deficiency, which leads to insufficient aldosterone synthesis
with ensuing salt wasting and hyperkalemia. This patient does not exhibit symptoms of 21-β-
hydroxylase deficiency.
www.eduwaves360.com | Telegram : @eduwaves360
D - Pleomorphic modified smooth muscle cells in the renal cortex
Explanation Why
The presence of pleomorphic smooth muscle cells in the renal cortex in combination with elevated
renin and aldosterone concentrations is highly indicative of a juxtaglomerular tumor. Excessive renin
production by these tumor cells stimulates aldosterone release. Consequently, patients exhibit
symptoms of secondary hyperaldosteronism.
E - Involution of zona glomerulosa of the adrenal gland
Explanation Why
In secondary hyperaldosteronism, elevated renin levels ultimately induce excessive aldosterone
production through the RAAS. Therefore, the zona glomerulosa would rather be hypertrophied, not
involuted, in this patient.
www.eduwaves360.com | Telegram : @eduwaves360
Question # 23
A 55-year-old woman with type 2 diabetes mellitus comes to the physician for evaluation of worsening
tingling of her feet at night for the last 6 months. Two years ago, she underwent retinal laser
photocoagulation in both eyes. She admits to not adhering to her insulin regimen. Her blood pressure is
130/85 mm Hg while sitting and 118/70 mm Hg while standing. Examination shows decreased sense of
vibration and proprioception in her toes and ankles bilaterally. Her serum hemoglobin A1C is 11%.
Urine dipstick shows 2+ protein. Which of the following additional findings is most likely in this
patient?
Answer Image
A Dilated pupils
B Incomplete bladder emptying
C Increased lower esophageal sphincter pressure
D Resting bradycardia
E Loss of satiety
F Hyperreflexia
www.eduwaves360.com | Telegram : @eduwaves360
Hint
This patient has clinical and laboratory evidence of poorly-controlled diabetes mellitus, including
elevated HbA1C, peripheral neuropathy, nephropathy, and retinopathy. The presence of postural
hypotension in this patient suggests diabetic autonomic neuropathy, which is a subtype of diabetic
neuropathy that results from poorly controlled diabetes and is characterized by damage to small fiber
autonomic nerves.
www.eduwaves360.com | Telegram : @eduwaves360
Correct Answer
A - Dilated pupils
Explanation Why
Diabetic autonomic neuropathy may lead to impaired dark adaptation and miosis from sympathetic
denervation. Mydriasis would not be expected in this patient.
B - Incomplete bladder emptying
Explanation Why
Genitourinary manifestations of diabetic autonomic neuropathy include incomplete bladder
emptying, urinary retention, erectile dysfunction, retrograde ejaculation, and dyspareunia. The loss of
afferent and efferent autonomic innervation of the bladder results in the inability to sense a full
bladder and incomplete emptying, predisposing patients to overflow incontinence and recurrent UTI.
C - Increased lower esophageal sphincter pressure
Explanation Why
Esophageal dysmotility, one of the gastrointestinal complications of diabetic autonomic neuropathy,
typically presents with decreased lower esophageal sphincter (LES) pressure and subsequent GERD.
Hence, increased LES pressures would not be expected in this patient.
www.eduwaves360.com | Telegram : @eduwaves360
D - Resting bradycardia
Explanation Why
Cardiovascular autonomic neuropathy, which often affects patients with long-standing diabetes, can
present with resting tachycardia (due to unopposed sympathetic nerve activity), orthostatic
hypotension, syncope, arrhythmias, ischemia, or sudden death. Resting bradycardia is not a common
presentation of diabetic autonomic neuropathy.
E - Loss of satiety
Explanation Why
Gastrointestinal manifestations of diabetic autonomic neuropathy include esophageal dysmotility,
gastroparesis, chronic diarrhea, and constipation. Patients with gastroparesis present with nausea,
vomiting, bloating, early satiety, abdominal pain, and weight loss. Hence, loss of satiety would not be
expected with this patient.
F - Hyperreflexia
Explanation Why
Diabetic polyneuropathy affects large myelinated and small myelinated and unmyelinated fibers,
leading to impaired proprioception, vibration, pain, touch, and temperature sensations. Patients
present with early symmetric sensory loss and distal hyporeflexia, followed by intrinsic muscle
atrophy and weakness. Hence, hyperreflexia would not be expected in this patient.
www.eduwaves360.com | Telegram : @eduwaves360
Question # 24
A 3200-g (7.1-lb) female newborn is delivered at 38 weeks' gestation to a 24-year-old woman. The
mother had regular prenatal visits throughout the pregnancy. The newborn's blood pressure is
53/35 mm Hg. Examination in the delivery room shows clitoromegaly and posterior labial fusion. One
day later, serum studies show:
Na+ 131 mEq/L
K+ 5.4 mEq/L
Cl− 102 mEq/L
Urea nitrogen 15 mg/dL
Creatinine 0.8 mg/dL
Ultrasound of the abdomen and pelvis shows a normal uterus and ovaries. Further evaluation of the
newborn is most likely to show which of the following findings?
Answer Image
A
Decreased
dehydroepiandrosterone
B
Increased
17-hydroxyprogesterone
www.eduwaves360.com | Telegram : @eduwaves360
Answer Image
C
Increased
11-deoxycorticosterone
D Increased bicarbonate
E Increased corticosterone
F Decreased renin activity
www.eduwaves360.com | Telegram : @eduwaves360
Hint
This female newborn presents with hypotension, ambiguous external genitalia (clitoromegaly with labial
fusion), hyponatremia, and hyperkalemia, which is characteristic of the salt-losing form of
21β-hydroxylase deficiency.
www.eduwaves360.com | Telegram : @eduwaves360
Correct Answer
A - Decreased dehydroepiandrosterone
Explanation Why
Dehydroepiandrosterone (DHEA) is a direct precursor for estrogen and testosterone biosynthesis,
and therefore, will be elevated (not decreased) in patients with hyperandrogenism due to 21β-
hydroxylase deficiency.
B - Increased 17-hydroxyprogesterone
Image
Explanation But
Patients with 21β-hydroxylase enzyme deficiency are treated with lifelong glucocorticoid and
fludrocortisone replacement therapy. Glucocorticoids correct hypocortisolism and improve
www.eduwaves360.com | Telegram : @eduwaves360
virilization by reducing ACTH secretion and adrenal hyperandrogenism via negative feedback.
Fludrocortisone acts as an aldosterone substitute.
Explanation Why
Increased levels of 17-hydroxyprogesterone, the physiological substrate for 21β-hydroxylase,
confirm the diagnosis of 21β-hydroxylase enzyme deficiency. This enzyme deficiency is the most
common cause of congenital adrenal hyperplasia and results in decreased synthesis of
mineralocorticoids (e.g., aldosterone) and glucocorticoids (e.g., cortisone). Hypoaldosteronism
causes hypotension and hyperkalemia and hypocortisolism leads to an increase in the secretion of
ACTH from the pituitary gland. Stimulation of the adrenal cortex by ACTH results in adrenal
hyperplasia and an increase in the production of adrenal androgens from steroid precursors such as
17-hydroxyprogesterone, thereby causing virilization in females.
C - Increased 11-deoxycorticosterone
Explanation Why
Increased levels of 11-deoxycorticosterone are seen in 11β-hydroxylase deficiency, a different cause
of congenital adrenal hyperplasia, which also presents with female virilization. However, congenital
adrenal hyperplasia typically presents with hypertension and hypokalemia rather than hypotension
and hyperkalemia. 11-deoxycorticosterone is synthesized by 21β-hydroxylase from progesterone in
the aldosterone synthesis pathway and would thus be decreased, not increased, in this patient.
D - Increased bicarbonate
Explanation Why
The lack of aldosterone in patients with 21β-hydroxylase deficiency leads to decreased
Na+
-resorption (hyponatremia), decreased K+
-secretion (hyperkalemia), and decreased H+
-secretion
(metabolic acidosis). Therefore, serum bicarbonate would be decreased, not increased in this patient,
in an effort to buffer the lower pH.
www.eduwaves360.com | Telegram : @eduwaves360
05.Endocrine System 4 Blocks @eduwaves360.pdf
05.Endocrine System 4 Blocks @eduwaves360.pdf
05.Endocrine System 4 Blocks @eduwaves360.pdf
05.Endocrine System 4 Blocks @eduwaves360.pdf
05.Endocrine System 4 Blocks @eduwaves360.pdf
05.Endocrine System 4 Blocks @eduwaves360.pdf
05.Endocrine System 4 Blocks @eduwaves360.pdf
05.Endocrine System 4 Blocks @eduwaves360.pdf
05.Endocrine System 4 Blocks @eduwaves360.pdf
05.Endocrine System 4 Blocks @eduwaves360.pdf
05.Endocrine System 4 Blocks @eduwaves360.pdf
05.Endocrine System 4 Blocks @eduwaves360.pdf
05.Endocrine System 4 Blocks @eduwaves360.pdf
05.Endocrine System 4 Blocks @eduwaves360.pdf
05.Endocrine System 4 Blocks @eduwaves360.pdf
05.Endocrine System 4 Blocks @eduwaves360.pdf
05.Endocrine System 4 Blocks @eduwaves360.pdf
05.Endocrine System 4 Blocks @eduwaves360.pdf
05.Endocrine System 4 Blocks @eduwaves360.pdf
05.Endocrine System 4 Blocks @eduwaves360.pdf
05.Endocrine System 4 Blocks @eduwaves360.pdf
05.Endocrine System 4 Blocks @eduwaves360.pdf
05.Endocrine System 4 Blocks @eduwaves360.pdf
05.Endocrine System 4 Blocks @eduwaves360.pdf
05.Endocrine System 4 Blocks @eduwaves360.pdf
05.Endocrine System 4 Blocks @eduwaves360.pdf
05.Endocrine System 4 Blocks @eduwaves360.pdf
05.Endocrine System 4 Blocks @eduwaves360.pdf
05.Endocrine System 4 Blocks @eduwaves360.pdf
05.Endocrine System 4 Blocks @eduwaves360.pdf
05.Endocrine System 4 Blocks @eduwaves360.pdf
05.Endocrine System 4 Blocks @eduwaves360.pdf
05.Endocrine System 4 Blocks @eduwaves360.pdf
05.Endocrine System 4 Blocks @eduwaves360.pdf
05.Endocrine System 4 Blocks @eduwaves360.pdf
05.Endocrine System 4 Blocks @eduwaves360.pdf
05.Endocrine System 4 Blocks @eduwaves360.pdf
05.Endocrine System 4 Blocks @eduwaves360.pdf
05.Endocrine System 4 Blocks @eduwaves360.pdf
05.Endocrine System 4 Blocks @eduwaves360.pdf
05.Endocrine System 4 Blocks @eduwaves360.pdf
05.Endocrine System 4 Blocks @eduwaves360.pdf
05.Endocrine System 4 Blocks @eduwaves360.pdf
05.Endocrine System 4 Blocks @eduwaves360.pdf
05.Endocrine System 4 Blocks @eduwaves360.pdf
05.Endocrine System 4 Blocks @eduwaves360.pdf
05.Endocrine System 4 Blocks @eduwaves360.pdf
05.Endocrine System 4 Blocks @eduwaves360.pdf
05.Endocrine System 4 Blocks @eduwaves360.pdf
05.Endocrine System 4 Blocks @eduwaves360.pdf
05.Endocrine System 4 Blocks @eduwaves360.pdf
05.Endocrine System 4 Blocks @eduwaves360.pdf
05.Endocrine System 4 Blocks @eduwaves360.pdf
05.Endocrine System 4 Blocks @eduwaves360.pdf
05.Endocrine System 4 Blocks @eduwaves360.pdf
05.Endocrine System 4 Blocks @eduwaves360.pdf
05.Endocrine System 4 Blocks @eduwaves360.pdf
05.Endocrine System 4 Blocks @eduwaves360.pdf
05.Endocrine System 4 Blocks @eduwaves360.pdf
05.Endocrine System 4 Blocks @eduwaves360.pdf
05.Endocrine System 4 Blocks @eduwaves360.pdf
05.Endocrine System 4 Blocks @eduwaves360.pdf
05.Endocrine System 4 Blocks @eduwaves360.pdf
05.Endocrine System 4 Blocks @eduwaves360.pdf
05.Endocrine System 4 Blocks @eduwaves360.pdf
05.Endocrine System 4 Blocks @eduwaves360.pdf
05.Endocrine System 4 Blocks @eduwaves360.pdf
05.Endocrine System 4 Blocks @eduwaves360.pdf
05.Endocrine System 4 Blocks @eduwaves360.pdf
05.Endocrine System 4 Blocks @eduwaves360.pdf
05.Endocrine System 4 Blocks @eduwaves360.pdf
05.Endocrine System 4 Blocks @eduwaves360.pdf
05.Endocrine System 4 Blocks @eduwaves360.pdf
05.Endocrine System 4 Blocks @eduwaves360.pdf
05.Endocrine System 4 Blocks @eduwaves360.pdf
05.Endocrine System 4 Blocks @eduwaves360.pdf
05.Endocrine System 4 Blocks @eduwaves360.pdf
05.Endocrine System 4 Blocks @eduwaves360.pdf
05.Endocrine System 4 Blocks @eduwaves360.pdf
05.Endocrine System 4 Blocks @eduwaves360.pdf
05.Endocrine System 4 Blocks @eduwaves360.pdf
05.Endocrine System 4 Blocks @eduwaves360.pdf
05.Endocrine System 4 Blocks @eduwaves360.pdf
05.Endocrine System 4 Blocks @eduwaves360.pdf
05.Endocrine System 4 Blocks @eduwaves360.pdf
05.Endocrine System 4 Blocks @eduwaves360.pdf
05.Endocrine System 4 Blocks @eduwaves360.pdf
05.Endocrine System 4 Blocks @eduwaves360.pdf
05.Endocrine System 4 Blocks @eduwaves360.pdf
05.Endocrine System 4 Blocks @eduwaves360.pdf
05.Endocrine System 4 Blocks @eduwaves360.pdf
05.Endocrine System 4 Blocks @eduwaves360.pdf
05.Endocrine System 4 Blocks @eduwaves360.pdf
05.Endocrine System 4 Blocks @eduwaves360.pdf
05.Endocrine System 4 Blocks @eduwaves360.pdf
05.Endocrine System 4 Blocks @eduwaves360.pdf
05.Endocrine System 4 Blocks @eduwaves360.pdf
05.Endocrine System 4 Blocks @eduwaves360.pdf
05.Endocrine System 4 Blocks @eduwaves360.pdf
05.Endocrine System 4 Blocks @eduwaves360.pdf
05.Endocrine System 4 Blocks @eduwaves360.pdf
05.Endocrine System 4 Blocks @eduwaves360.pdf
05.Endocrine System 4 Blocks @eduwaves360.pdf
05.Endocrine System 4 Blocks @eduwaves360.pdf
05.Endocrine System 4 Blocks @eduwaves360.pdf
05.Endocrine System 4 Blocks @eduwaves360.pdf
05.Endocrine System 4 Blocks @eduwaves360.pdf
05.Endocrine System 4 Blocks @eduwaves360.pdf
05.Endocrine System 4 Blocks @eduwaves360.pdf
05.Endocrine System 4 Blocks @eduwaves360.pdf
05.Endocrine System 4 Blocks @eduwaves360.pdf
05.Endocrine System 4 Blocks @eduwaves360.pdf
05.Endocrine System 4 Blocks @eduwaves360.pdf
05.Endocrine System 4 Blocks @eduwaves360.pdf
05.Endocrine System 4 Blocks @eduwaves360.pdf
05.Endocrine System 4 Blocks @eduwaves360.pdf
05.Endocrine System 4 Blocks @eduwaves360.pdf
05.Endocrine System 4 Blocks @eduwaves360.pdf
05.Endocrine System 4 Blocks @eduwaves360.pdf
05.Endocrine System 4 Blocks @eduwaves360.pdf
05.Endocrine System 4 Blocks @eduwaves360.pdf
05.Endocrine System 4 Blocks @eduwaves360.pdf
05.Endocrine System 4 Blocks @eduwaves360.pdf
05.Endocrine System 4 Blocks @eduwaves360.pdf
05.Endocrine System 4 Blocks @eduwaves360.pdf
05.Endocrine System 4 Blocks @eduwaves360.pdf
05.Endocrine System 4 Blocks @eduwaves360.pdf
05.Endocrine System 4 Blocks @eduwaves360.pdf
05.Endocrine System 4 Blocks @eduwaves360.pdf
05.Endocrine System 4 Blocks @eduwaves360.pdf
05.Endocrine System 4 Blocks @eduwaves360.pdf
05.Endocrine System 4 Blocks @eduwaves360.pdf
05.Endocrine System 4 Blocks @eduwaves360.pdf
05.Endocrine System 4 Blocks @eduwaves360.pdf
05.Endocrine System 4 Blocks @eduwaves360.pdf
05.Endocrine System 4 Blocks @eduwaves360.pdf
05.Endocrine System 4 Blocks @eduwaves360.pdf
05.Endocrine System 4 Blocks @eduwaves360.pdf
05.Endocrine System 4 Blocks @eduwaves360.pdf
05.Endocrine System 4 Blocks @eduwaves360.pdf
05.Endocrine System 4 Blocks @eduwaves360.pdf
05.Endocrine System 4 Blocks @eduwaves360.pdf
05.Endocrine System 4 Blocks @eduwaves360.pdf
05.Endocrine System 4 Blocks @eduwaves360.pdf
05.Endocrine System 4 Blocks @eduwaves360.pdf
05.Endocrine System 4 Blocks @eduwaves360.pdf
05.Endocrine System 4 Blocks @eduwaves360.pdf
05.Endocrine System 4 Blocks @eduwaves360.pdf
05.Endocrine System 4 Blocks @eduwaves360.pdf
05.Endocrine System 4 Blocks @eduwaves360.pdf
05.Endocrine System 4 Blocks @eduwaves360.pdf
05.Endocrine System 4 Blocks @eduwaves360.pdf
05.Endocrine System 4 Blocks @eduwaves360.pdf
05.Endocrine System 4 Blocks @eduwaves360.pdf
05.Endocrine System 4 Blocks @eduwaves360.pdf
05.Endocrine System 4 Blocks @eduwaves360.pdf
05.Endocrine System 4 Blocks @eduwaves360.pdf
05.Endocrine System 4 Blocks @eduwaves360.pdf
05.Endocrine System 4 Blocks @eduwaves360.pdf
05.Endocrine System 4 Blocks @eduwaves360.pdf
05.Endocrine System 4 Blocks @eduwaves360.pdf
05.Endocrine System 4 Blocks @eduwaves360.pdf
05.Endocrine System 4 Blocks @eduwaves360.pdf
05.Endocrine System 4 Blocks @eduwaves360.pdf
05.Endocrine System 4 Blocks @eduwaves360.pdf
05.Endocrine System 4 Blocks @eduwaves360.pdf
05.Endocrine System 4 Blocks @eduwaves360.pdf
05.Endocrine System 4 Blocks @eduwaves360.pdf
05.Endocrine System 4 Blocks @eduwaves360.pdf
05.Endocrine System 4 Blocks @eduwaves360.pdf
05.Endocrine System 4 Blocks @eduwaves360.pdf
05.Endocrine System 4 Blocks @eduwaves360.pdf
05.Endocrine System 4 Blocks @eduwaves360.pdf
05.Endocrine System 4 Blocks @eduwaves360.pdf
05.Endocrine System 4 Blocks @eduwaves360.pdf
05.Endocrine System 4 Blocks @eduwaves360.pdf
05.Endocrine System 4 Blocks @eduwaves360.pdf
05.Endocrine System 4 Blocks @eduwaves360.pdf
05.Endocrine System 4 Blocks @eduwaves360.pdf
05.Endocrine System 4 Blocks @eduwaves360.pdf
05.Endocrine System 4 Blocks @eduwaves360.pdf
05.Endocrine System 4 Blocks @eduwaves360.pdf
05.Endocrine System 4 Blocks @eduwaves360.pdf
05.Endocrine System 4 Blocks @eduwaves360.pdf
05.Endocrine System 4 Blocks @eduwaves360.pdf
05.Endocrine System 4 Blocks @eduwaves360.pdf
05.Endocrine System 4 Blocks @eduwaves360.pdf
05.Endocrine System 4 Blocks @eduwaves360.pdf
05.Endocrine System 4 Blocks @eduwaves360.pdf
05.Endocrine System 4 Blocks @eduwaves360.pdf
05.Endocrine System 4 Blocks @eduwaves360.pdf
05.Endocrine System 4 Blocks @eduwaves360.pdf
05.Endocrine System 4 Blocks @eduwaves360.pdf
05.Endocrine System 4 Blocks @eduwaves360.pdf
05.Endocrine System 4 Blocks @eduwaves360.pdf
05.Endocrine System 4 Blocks @eduwaves360.pdf
05.Endocrine System 4 Blocks @eduwaves360.pdf
05.Endocrine System 4 Blocks @eduwaves360.pdf
05.Endocrine System 4 Blocks @eduwaves360.pdf
05.Endocrine System 4 Blocks @eduwaves360.pdf
05.Endocrine System 4 Blocks @eduwaves360.pdf
05.Endocrine System 4 Blocks @eduwaves360.pdf
05.Endocrine System 4 Blocks @eduwaves360.pdf
05.Endocrine System 4 Blocks @eduwaves360.pdf
05.Endocrine System 4 Blocks @eduwaves360.pdf
05.Endocrine System 4 Blocks @eduwaves360.pdf
05.Endocrine System 4 Blocks @eduwaves360.pdf
05.Endocrine System 4 Blocks @eduwaves360.pdf
05.Endocrine System 4 Blocks @eduwaves360.pdf
05.Endocrine System 4 Blocks @eduwaves360.pdf
05.Endocrine System 4 Blocks @eduwaves360.pdf
05.Endocrine System 4 Blocks @eduwaves360.pdf
05.Endocrine System 4 Blocks @eduwaves360.pdf
05.Endocrine System 4 Blocks @eduwaves360.pdf
05.Endocrine System 4 Blocks @eduwaves360.pdf
05.Endocrine System 4 Blocks @eduwaves360.pdf
05.Endocrine System 4 Blocks @eduwaves360.pdf
05.Endocrine System 4 Blocks @eduwaves360.pdf
05.Endocrine System 4 Blocks @eduwaves360.pdf
05.Endocrine System 4 Blocks @eduwaves360.pdf
05.Endocrine System 4 Blocks @eduwaves360.pdf
05.Endocrine System 4 Blocks @eduwaves360.pdf
05.Endocrine System 4 Blocks @eduwaves360.pdf
05.Endocrine System 4 Blocks @eduwaves360.pdf
05.Endocrine System 4 Blocks @eduwaves360.pdf
05.Endocrine System 4 Blocks @eduwaves360.pdf
05.Endocrine System 4 Blocks @eduwaves360.pdf
05.Endocrine System 4 Blocks @eduwaves360.pdf
05.Endocrine System 4 Blocks @eduwaves360.pdf
05.Endocrine System 4 Blocks @eduwaves360.pdf
05.Endocrine System 4 Blocks @eduwaves360.pdf
05.Endocrine System 4 Blocks @eduwaves360.pdf
05.Endocrine System 4 Blocks @eduwaves360.pdf
05.Endocrine System 4 Blocks @eduwaves360.pdf
05.Endocrine System 4 Blocks @eduwaves360.pdf
05.Endocrine System 4 Blocks @eduwaves360.pdf
05.Endocrine System 4 Blocks @eduwaves360.pdf
05.Endocrine System 4 Blocks @eduwaves360.pdf
05.Endocrine System 4 Blocks @eduwaves360.pdf
05.Endocrine System 4 Blocks @eduwaves360.pdf
05.Endocrine System 4 Blocks @eduwaves360.pdf
05.Endocrine System 4 Blocks @eduwaves360.pdf
05.Endocrine System 4 Blocks @eduwaves360.pdf
05.Endocrine System 4 Blocks @eduwaves360.pdf
05.Endocrine System 4 Blocks @eduwaves360.pdf
05.Endocrine System 4 Blocks @eduwaves360.pdf
05.Endocrine System 4 Blocks @eduwaves360.pdf
05.Endocrine System 4 Blocks @eduwaves360.pdf
05.Endocrine System 4 Blocks @eduwaves360.pdf
05.Endocrine System 4 Blocks @eduwaves360.pdf
05.Endocrine System 4 Blocks @eduwaves360.pdf
05.Endocrine System 4 Blocks @eduwaves360.pdf
05.Endocrine System 4 Blocks @eduwaves360.pdf
05.Endocrine System 4 Blocks @eduwaves360.pdf
05.Endocrine System 4 Blocks @eduwaves360.pdf
05.Endocrine System 4 Blocks @eduwaves360.pdf
05.Endocrine System 4 Blocks @eduwaves360.pdf
05.Endocrine System 4 Blocks @eduwaves360.pdf
05.Endocrine System 4 Blocks @eduwaves360.pdf
05.Endocrine System 4 Blocks @eduwaves360.pdf
05.Endocrine System 4 Blocks @eduwaves360.pdf
05.Endocrine System 4 Blocks @eduwaves360.pdf
05.Endocrine System 4 Blocks @eduwaves360.pdf
05.Endocrine System 4 Blocks @eduwaves360.pdf
05.Endocrine System 4 Blocks @eduwaves360.pdf
05.Endocrine System 4 Blocks @eduwaves360.pdf
05.Endocrine System 4 Blocks @eduwaves360.pdf
05.Endocrine System 4 Blocks @eduwaves360.pdf
05.Endocrine System 4 Blocks @eduwaves360.pdf
05.Endocrine System 4 Blocks @eduwaves360.pdf
05.Endocrine System 4 Blocks @eduwaves360.pdf
05.Endocrine System 4 Blocks @eduwaves360.pdf
05.Endocrine System 4 Blocks @eduwaves360.pdf
05.Endocrine System 4 Blocks @eduwaves360.pdf
05.Endocrine System 4 Blocks @eduwaves360.pdf
05.Endocrine System 4 Blocks @eduwaves360.pdf
05.Endocrine System 4 Blocks @eduwaves360.pdf
05.Endocrine System 4 Blocks @eduwaves360.pdf
05.Endocrine System 4 Blocks @eduwaves360.pdf
05.Endocrine System 4 Blocks @eduwaves360.pdf
05.Endocrine System 4 Blocks @eduwaves360.pdf
05.Endocrine System 4 Blocks @eduwaves360.pdf
05.Endocrine System 4 Blocks @eduwaves360.pdf
05.Endocrine System 4 Blocks @eduwaves360.pdf
05.Endocrine System 4 Blocks @eduwaves360.pdf
05.Endocrine System 4 Blocks @eduwaves360.pdf
05.Endocrine System 4 Blocks @eduwaves360.pdf
05.Endocrine System 4 Blocks @eduwaves360.pdf
05.Endocrine System 4 Blocks @eduwaves360.pdf
05.Endocrine System 4 Blocks @eduwaves360.pdf
05.Endocrine System 4 Blocks @eduwaves360.pdf
05.Endocrine System 4 Blocks @eduwaves360.pdf
05.Endocrine System 4 Blocks @eduwaves360.pdf
05.Endocrine System 4 Blocks @eduwaves360.pdf
05.Endocrine System 4 Blocks @eduwaves360.pdf
05.Endocrine System 4 Blocks @eduwaves360.pdf
05.Endocrine System 4 Blocks @eduwaves360.pdf
05.Endocrine System 4 Blocks @eduwaves360.pdf
05.Endocrine System 4 Blocks @eduwaves360.pdf
05.Endocrine System 4 Blocks @eduwaves360.pdf
05.Endocrine System 4 Blocks @eduwaves360.pdf
05.Endocrine System 4 Blocks @eduwaves360.pdf
05.Endocrine System 4 Blocks @eduwaves360.pdf
05.Endocrine System 4 Blocks @eduwaves360.pdf
05.Endocrine System 4 Blocks @eduwaves360.pdf
05.Endocrine System 4 Blocks @eduwaves360.pdf
05.Endocrine System 4 Blocks @eduwaves360.pdf
05.Endocrine System 4 Blocks @eduwaves360.pdf
05.Endocrine System 4 Blocks @eduwaves360.pdf
05.Endocrine System 4 Blocks @eduwaves360.pdf
05.Endocrine System 4 Blocks @eduwaves360.pdf
05.Endocrine System 4 Blocks @eduwaves360.pdf
05.Endocrine System 4 Blocks @eduwaves360.pdf
05.Endocrine System 4 Blocks @eduwaves360.pdf
05.Endocrine System 4 Blocks @eduwaves360.pdf
05.Endocrine System 4 Blocks @eduwaves360.pdf
05.Endocrine System 4 Blocks @eduwaves360.pdf
05.Endocrine System 4 Blocks @eduwaves360.pdf
05.Endocrine System 4 Blocks @eduwaves360.pdf
05.Endocrine System 4 Blocks @eduwaves360.pdf
05.Endocrine System 4 Blocks @eduwaves360.pdf
05.Endocrine System 4 Blocks @eduwaves360.pdf
05.Endocrine System 4 Blocks @eduwaves360.pdf
05.Endocrine System 4 Blocks @eduwaves360.pdf
05.Endocrine System 4 Blocks @eduwaves360.pdf
05.Endocrine System 4 Blocks @eduwaves360.pdf
05.Endocrine System 4 Blocks @eduwaves360.pdf
05.Endocrine System 4 Blocks @eduwaves360.pdf
05.Endocrine System 4 Blocks @eduwaves360.pdf
05.Endocrine System 4 Blocks @eduwaves360.pdf
05.Endocrine System 4 Blocks @eduwaves360.pdf
05.Endocrine System 4 Blocks @eduwaves360.pdf
05.Endocrine System 4 Blocks @eduwaves360.pdf
05.Endocrine System 4 Blocks @eduwaves360.pdf
05.Endocrine System 4 Blocks @eduwaves360.pdf
05.Endocrine System 4 Blocks @eduwaves360.pdf
05.Endocrine System 4 Blocks @eduwaves360.pdf
05.Endocrine System 4 Blocks @eduwaves360.pdf
05.Endocrine System 4 Blocks @eduwaves360.pdf
05.Endocrine System 4 Blocks @eduwaves360.pdf
05.Endocrine System 4 Blocks @eduwaves360.pdf
05.Endocrine System 4 Blocks @eduwaves360.pdf
05.Endocrine System 4 Blocks @eduwaves360.pdf
05.Endocrine System 4 Blocks @eduwaves360.pdf
05.Endocrine System 4 Blocks @eduwaves360.pdf
05.Endocrine System 4 Blocks @eduwaves360.pdf
05.Endocrine System 4 Blocks @eduwaves360.pdf
05.Endocrine System 4 Blocks @eduwaves360.pdf
05.Endocrine System 4 Blocks @eduwaves360.pdf
05.Endocrine System 4 Blocks @eduwaves360.pdf
05.Endocrine System 4 Blocks @eduwaves360.pdf
05.Endocrine System 4 Blocks @eduwaves360.pdf
05.Endocrine System 4 Blocks @eduwaves360.pdf
05.Endocrine System 4 Blocks @eduwaves360.pdf
05.Endocrine System 4 Blocks @eduwaves360.pdf
05.Endocrine System 4 Blocks @eduwaves360.pdf
05.Endocrine System 4 Blocks @eduwaves360.pdf
05.Endocrine System 4 Blocks @eduwaves360.pdf
05.Endocrine System 4 Blocks @eduwaves360.pdf
05.Endocrine System 4 Blocks @eduwaves360.pdf
05.Endocrine System 4 Blocks @eduwaves360.pdf
05.Endocrine System 4 Blocks @eduwaves360.pdf
05.Endocrine System 4 Blocks @eduwaves360.pdf
05.Endocrine System 4 Blocks @eduwaves360.pdf
05.Endocrine System 4 Blocks @eduwaves360.pdf
05.Endocrine System 4 Blocks @eduwaves360.pdf
05.Endocrine System 4 Blocks @eduwaves360.pdf
05.Endocrine System 4 Blocks @eduwaves360.pdf
05.Endocrine System 4 Blocks @eduwaves360.pdf
05.Endocrine System 4 Blocks @eduwaves360.pdf
05.Endocrine System 4 Blocks @eduwaves360.pdf
05.Endocrine System 4 Blocks @eduwaves360.pdf
05.Endocrine System 4 Blocks @eduwaves360.pdf
05.Endocrine System 4 Blocks @eduwaves360.pdf
05.Endocrine System 4 Blocks @eduwaves360.pdf
05.Endocrine System 4 Blocks @eduwaves360.pdf
05.Endocrine System 4 Blocks @eduwaves360.pdf
05.Endocrine System 4 Blocks @eduwaves360.pdf
05.Endocrine System 4 Blocks @eduwaves360.pdf
05.Endocrine System 4 Blocks @eduwaves360.pdf
05.Endocrine System 4 Blocks @eduwaves360.pdf
05.Endocrine System 4 Blocks @eduwaves360.pdf
05.Endocrine System 4 Blocks @eduwaves360.pdf
05.Endocrine System 4 Blocks @eduwaves360.pdf
05.Endocrine System 4 Blocks @eduwaves360.pdf
05.Endocrine System 4 Blocks @eduwaves360.pdf
05.Endocrine System 4 Blocks @eduwaves360.pdf
05.Endocrine System 4 Blocks @eduwaves360.pdf
05.Endocrine System 4 Blocks @eduwaves360.pdf
05.Endocrine System 4 Blocks @eduwaves360.pdf
05.Endocrine System 4 Blocks @eduwaves360.pdf
05.Endocrine System 4 Blocks @eduwaves360.pdf
05.Endocrine System 4 Blocks @eduwaves360.pdf
05.Endocrine System 4 Blocks @eduwaves360.pdf
05.Endocrine System 4 Blocks @eduwaves360.pdf
05.Endocrine System 4 Blocks @eduwaves360.pdf
05.Endocrine System 4 Blocks @eduwaves360.pdf
05.Endocrine System 4 Blocks @eduwaves360.pdf
05.Endocrine System 4 Blocks @eduwaves360.pdf
05.Endocrine System 4 Blocks @eduwaves360.pdf
05.Endocrine System 4 Blocks @eduwaves360.pdf
05.Endocrine System 4 Blocks @eduwaves360.pdf
05.Endocrine System 4 Blocks @eduwaves360.pdf
05.Endocrine System 4 Blocks @eduwaves360.pdf
05.Endocrine System 4 Blocks @eduwaves360.pdf
05.Endocrine System 4 Blocks @eduwaves360.pdf
05.Endocrine System 4 Blocks @eduwaves360.pdf
05.Endocrine System 4 Blocks @eduwaves360.pdf
05.Endocrine System 4 Blocks @eduwaves360.pdf
05.Endocrine System 4 Blocks @eduwaves360.pdf
05.Endocrine System 4 Blocks @eduwaves360.pdf
05.Endocrine System 4 Blocks @eduwaves360.pdf
05.Endocrine System 4 Blocks @eduwaves360.pdf
05.Endocrine System 4 Blocks @eduwaves360.pdf
05.Endocrine System 4 Blocks @eduwaves360.pdf
05.Endocrine System 4 Blocks @eduwaves360.pdf
05.Endocrine System 4 Blocks @eduwaves360.pdf
05.Endocrine System 4 Blocks @eduwaves360.pdf
05.Endocrine System 4 Blocks @eduwaves360.pdf
05.Endocrine System 4 Blocks @eduwaves360.pdf
05.Endocrine System 4 Blocks @eduwaves360.pdf
05.Endocrine System 4 Blocks @eduwaves360.pdf
05.Endocrine System 4 Blocks @eduwaves360.pdf
05.Endocrine System 4 Blocks @eduwaves360.pdf
05.Endocrine System 4 Blocks @eduwaves360.pdf
05.Endocrine System 4 Blocks @eduwaves360.pdf
05.Endocrine System 4 Blocks @eduwaves360.pdf
05.Endocrine System 4 Blocks @eduwaves360.pdf
05.Endocrine System 4 Blocks @eduwaves360.pdf
05.Endocrine System 4 Blocks @eduwaves360.pdf
05.Endocrine System 4 Blocks @eduwaves360.pdf
05.Endocrine System 4 Blocks @eduwaves360.pdf
05.Endocrine System 4 Blocks @eduwaves360.pdf
05.Endocrine System 4 Blocks @eduwaves360.pdf
05.Endocrine System 4 Blocks @eduwaves360.pdf
05.Endocrine System 4 Blocks @eduwaves360.pdf
05.Endocrine System 4 Blocks @eduwaves360.pdf
05.Endocrine System 4 Blocks @eduwaves360.pdf
05.Endocrine System 4 Blocks @eduwaves360.pdf
05.Endocrine System 4 Blocks @eduwaves360.pdf
05.Endocrine System 4 Blocks @eduwaves360.pdf
05.Endocrine System 4 Blocks @eduwaves360.pdf
05.Endocrine System 4 Blocks @eduwaves360.pdf
05.Endocrine System 4 Blocks @eduwaves360.pdf
05.Endocrine System 4 Blocks @eduwaves360.pdf
05.Endocrine System 4 Blocks @eduwaves360.pdf
05.Endocrine System 4 Blocks @eduwaves360.pdf
05.Endocrine System 4 Blocks @eduwaves360.pdf
05.Endocrine System 4 Blocks @eduwaves360.pdf
05.Endocrine System 4 Blocks @eduwaves360.pdf
05.Endocrine System 4 Blocks @eduwaves360.pdf
05.Endocrine System 4 Blocks @eduwaves360.pdf
05.Endocrine System 4 Blocks @eduwaves360.pdf
05.Endocrine System 4 Blocks @eduwaves360.pdf
05.Endocrine System 4 Blocks @eduwaves360.pdf
05.Endocrine System 4 Blocks @eduwaves360.pdf
05.Endocrine System 4 Blocks @eduwaves360.pdf
05.Endocrine System 4 Blocks @eduwaves360.pdf
05.Endocrine System 4 Blocks @eduwaves360.pdf
05.Endocrine System 4 Blocks @eduwaves360.pdf
05.Endocrine System 4 Blocks @eduwaves360.pdf
05.Endocrine System 4 Blocks @eduwaves360.pdf
05.Endocrine System 4 Blocks @eduwaves360.pdf
05.Endocrine System 4 Blocks @eduwaves360.pdf
05.Endocrine System 4 Blocks @eduwaves360.pdf
05.Endocrine System 4 Blocks @eduwaves360.pdf
05.Endocrine System 4 Blocks @eduwaves360.pdf
05.Endocrine System 4 Blocks @eduwaves360.pdf
05.Endocrine System 4 Blocks @eduwaves360.pdf
05.Endocrine System 4 Blocks @eduwaves360.pdf
05.Endocrine System 4 Blocks @eduwaves360.pdf
05.Endocrine System 4 Blocks @eduwaves360.pdf
05.Endocrine System 4 Blocks @eduwaves360.pdf
05.Endocrine System 4 Blocks @eduwaves360.pdf
05.Endocrine System 4 Blocks @eduwaves360.pdf
05.Endocrine System 4 Blocks @eduwaves360.pdf
05.Endocrine System 4 Blocks @eduwaves360.pdf
05.Endocrine System 4 Blocks @eduwaves360.pdf
05.Endocrine System 4 Blocks @eduwaves360.pdf
05.Endocrine System 4 Blocks @eduwaves360.pdf
05.Endocrine System 4 Blocks @eduwaves360.pdf
05.Endocrine System 4 Blocks @eduwaves360.pdf
05.Endocrine System 4 Blocks @eduwaves360.pdf
05.Endocrine System 4 Blocks @eduwaves360.pdf
05.Endocrine System 4 Blocks @eduwaves360.pdf
05.Endocrine System 4 Blocks @eduwaves360.pdf
05.Endocrine System 4 Blocks @eduwaves360.pdf
05.Endocrine System 4 Blocks @eduwaves360.pdf
05.Endocrine System 4 Blocks @eduwaves360.pdf
05.Endocrine System 4 Blocks @eduwaves360.pdf
05.Endocrine System 4 Blocks @eduwaves360.pdf
05.Endocrine System 4 Blocks @eduwaves360.pdf
05.Endocrine System 4 Blocks @eduwaves360.pdf
05.Endocrine System 4 Blocks @eduwaves360.pdf
05.Endocrine System 4 Blocks @eduwaves360.pdf
05.Endocrine System 4 Blocks @eduwaves360.pdf
05.Endocrine System 4 Blocks @eduwaves360.pdf
05.Endocrine System 4 Blocks @eduwaves360.pdf
05.Endocrine System 4 Blocks @eduwaves360.pdf
05.Endocrine System 4 Blocks @eduwaves360.pdf
05.Endocrine System 4 Blocks @eduwaves360.pdf
05.Endocrine System 4 Blocks @eduwaves360.pdf
05.Endocrine System 4 Blocks @eduwaves360.pdf
05.Endocrine System 4 Blocks @eduwaves360.pdf
05.Endocrine System 4 Blocks @eduwaves360.pdf
05.Endocrine System 4 Blocks @eduwaves360.pdf
05.Endocrine System 4 Blocks @eduwaves360.pdf
05.Endocrine System 4 Blocks @eduwaves360.pdf
05.Endocrine System 4 Blocks @eduwaves360.pdf
05.Endocrine System 4 Blocks @eduwaves360.pdf
05.Endocrine System 4 Blocks @eduwaves360.pdf
05.Endocrine System 4 Blocks @eduwaves360.pdf
05.Endocrine System 4 Blocks @eduwaves360.pdf
05.Endocrine System 4 Blocks @eduwaves360.pdf
05.Endocrine System 4 Blocks @eduwaves360.pdf
05.Endocrine System 4 Blocks @eduwaves360.pdf
05.Endocrine System 4 Blocks @eduwaves360.pdf
05.Endocrine System 4 Blocks @eduwaves360.pdf
05.Endocrine System 4 Blocks @eduwaves360.pdf
05.Endocrine System 4 Blocks @eduwaves360.pdf
05.Endocrine System 4 Blocks @eduwaves360.pdf
05.Endocrine System 4 Blocks @eduwaves360.pdf
05.Endocrine System 4 Blocks @eduwaves360.pdf
05.Endocrine System 4 Blocks @eduwaves360.pdf
05.Endocrine System 4 Blocks @eduwaves360.pdf
05.Endocrine System 4 Blocks @eduwaves360.pdf
05.Endocrine System 4 Blocks @eduwaves360.pdf
05.Endocrine System 4 Blocks @eduwaves360.pdf
05.Endocrine System 4 Blocks @eduwaves360.pdf
05.Endocrine System 4 Blocks @eduwaves360.pdf
05.Endocrine System 4 Blocks @eduwaves360.pdf
05.Endocrine System 4 Blocks @eduwaves360.pdf
05.Endocrine System 4 Blocks @eduwaves360.pdf
05.Endocrine System 4 Blocks @eduwaves360.pdf
05.Endocrine System 4 Blocks @eduwaves360.pdf
05.Endocrine System 4 Blocks @eduwaves360.pdf
05.Endocrine System 4 Blocks @eduwaves360.pdf
05.Endocrine System 4 Blocks @eduwaves360.pdf
05.Endocrine System 4 Blocks @eduwaves360.pdf
05.Endocrine System 4 Blocks @eduwaves360.pdf
05.Endocrine System 4 Blocks @eduwaves360.pdf
05.Endocrine System 4 Blocks @eduwaves360.pdf
05.Endocrine System 4 Blocks @eduwaves360.pdf
05.Endocrine System 4 Blocks @eduwaves360.pdf
05.Endocrine System 4 Blocks @eduwaves360.pdf
05.Endocrine System 4 Blocks @eduwaves360.pdf
05.Endocrine System 4 Blocks @eduwaves360.pdf
05.Endocrine System 4 Blocks @eduwaves360.pdf
05.Endocrine System 4 Blocks @eduwaves360.pdf
05.Endocrine System 4 Blocks @eduwaves360.pdf
05.Endocrine System 4 Blocks @eduwaves360.pdf
05.Endocrine System 4 Blocks @eduwaves360.pdf
05.Endocrine System 4 Blocks @eduwaves360.pdf
05.Endocrine System 4 Blocks @eduwaves360.pdf
05.Endocrine System 4 Blocks @eduwaves360.pdf
05.Endocrine System 4 Blocks @eduwaves360.pdf
05.Endocrine System 4 Blocks @eduwaves360.pdf
05.Endocrine System 4 Blocks @eduwaves360.pdf
05.Endocrine System 4 Blocks @eduwaves360.pdf
05.Endocrine System 4 Blocks @eduwaves360.pdf
05.Endocrine System 4 Blocks @eduwaves360.pdf
05.Endocrine System 4 Blocks @eduwaves360.pdf
05.Endocrine System 4 Blocks @eduwaves360.pdf
05.Endocrine System 4 Blocks @eduwaves360.pdf
05.Endocrine System 4 Blocks @eduwaves360.pdf
05.Endocrine System 4 Blocks @eduwaves360.pdf
05.Endocrine System 4 Blocks @eduwaves360.pdf
05.Endocrine System 4 Blocks @eduwaves360.pdf
05.Endocrine System 4 Blocks @eduwaves360.pdf
05.Endocrine System 4 Blocks @eduwaves360.pdf
05.Endocrine System 4 Blocks @eduwaves360.pdf
05.Endocrine System 4 Blocks @eduwaves360.pdf
05.Endocrine System 4 Blocks @eduwaves360.pdf
05.Endocrine System 4 Blocks @eduwaves360.pdf
05.Endocrine System 4 Blocks @eduwaves360.pdf
05.Endocrine System 4 Blocks @eduwaves360.pdf
05.Endocrine System 4 Blocks @eduwaves360.pdf
05.Endocrine System 4 Blocks @eduwaves360.pdf
05.Endocrine System 4 Blocks @eduwaves360.pdf
05.Endocrine System 4 Blocks @eduwaves360.pdf
05.Endocrine System 4 Blocks @eduwaves360.pdf
05.Endocrine System 4 Blocks @eduwaves360.pdf
05.Endocrine System 4 Blocks @eduwaves360.pdf
05.Endocrine System 4 Blocks @eduwaves360.pdf
05.Endocrine System 4 Blocks @eduwaves360.pdf
05.Endocrine System 4 Blocks @eduwaves360.pdf
05.Endocrine System 4 Blocks @eduwaves360.pdf
05.Endocrine System 4 Blocks @eduwaves360.pdf
05.Endocrine System 4 Blocks @eduwaves360.pdf
05.Endocrine System 4 Blocks @eduwaves360.pdf
05.Endocrine System 4 Blocks @eduwaves360.pdf
05.Endocrine System 4 Blocks @eduwaves360.pdf
05.Endocrine System 4 Blocks @eduwaves360.pdf
05.Endocrine System 4 Blocks @eduwaves360.pdf
05.Endocrine System 4 Blocks @eduwaves360.pdf
05.Endocrine System 4 Blocks @eduwaves360.pdf
05.Endocrine System 4 Blocks @eduwaves360.pdf
05.Endocrine System 4 Blocks @eduwaves360.pdf
05.Endocrine System 4 Blocks @eduwaves360.pdf
05.Endocrine System 4 Blocks @eduwaves360.pdf
05.Endocrine System 4 Blocks @eduwaves360.pdf
05.Endocrine System 4 Blocks @eduwaves360.pdf
05.Endocrine System 4 Blocks @eduwaves360.pdf
05.Endocrine System 4 Blocks @eduwaves360.pdf
05.Endocrine System 4 Blocks @eduwaves360.pdf
05.Endocrine System 4 Blocks @eduwaves360.pdf
05.Endocrine System 4 Blocks @eduwaves360.pdf
05.Endocrine System 4 Blocks @eduwaves360.pdf
05.Endocrine System 4 Blocks @eduwaves360.pdf
05.Endocrine System 4 Blocks @eduwaves360.pdf
05.Endocrine System 4 Blocks @eduwaves360.pdf
05.Endocrine System 4 Blocks @eduwaves360.pdf
05.Endocrine System 4 Blocks @eduwaves360.pdf
05.Endocrine System 4 Blocks @eduwaves360.pdf
05.Endocrine System 4 Blocks @eduwaves360.pdf
05.Endocrine System 4 Blocks @eduwaves360.pdf
05.Endocrine System 4 Blocks @eduwaves360.pdf
05.Endocrine System 4 Blocks @eduwaves360.pdf
05.Endocrine System 4 Blocks @eduwaves360.pdf
05.Endocrine System 4 Blocks @eduwaves360.pdf
05.Endocrine System 4 Blocks @eduwaves360.pdf
05.Endocrine System 4 Blocks @eduwaves360.pdf
05.Endocrine System 4 Blocks @eduwaves360.pdf
05.Endocrine System 4 Blocks @eduwaves360.pdf
05.Endocrine System 4 Blocks @eduwaves360.pdf
05.Endocrine System 4 Blocks @eduwaves360.pdf
05.Endocrine System 4 Blocks @eduwaves360.pdf
05.Endocrine System 4 Blocks @eduwaves360.pdf
05.Endocrine System 4 Blocks @eduwaves360.pdf
05.Endocrine System 4 Blocks @eduwaves360.pdf
05.Endocrine System 4 Blocks @eduwaves360.pdf
05.Endocrine System 4 Blocks @eduwaves360.pdf
05.Endocrine System 4 Blocks @eduwaves360.pdf
05.Endocrine System 4 Blocks @eduwaves360.pdf
05.Endocrine System 4 Blocks @eduwaves360.pdf
05.Endocrine System 4 Blocks @eduwaves360.pdf
05.Endocrine System 4 Blocks @eduwaves360.pdf
05.Endocrine System 4 Blocks @eduwaves360.pdf
05.Endocrine System 4 Blocks @eduwaves360.pdf
05.Endocrine System 4 Blocks @eduwaves360.pdf
05.Endocrine System 4 Blocks @eduwaves360.pdf
05.Endocrine System 4 Blocks @eduwaves360.pdf
05.Endocrine System 4 Blocks @eduwaves360.pdf
05.Endocrine System 4 Blocks @eduwaves360.pdf
05.Endocrine System 4 Blocks @eduwaves360.pdf
05.Endocrine System 4 Blocks @eduwaves360.pdf
05.Endocrine System 4 Blocks @eduwaves360.pdf
05.Endocrine System 4 Blocks @eduwaves360.pdf
05.Endocrine System 4 Blocks @eduwaves360.pdf
05.Endocrine System 4 Blocks @eduwaves360.pdf
05.Endocrine System 4 Blocks @eduwaves360.pdf
05.Endocrine System 4 Blocks @eduwaves360.pdf
05.Endocrine System 4 Blocks @eduwaves360.pdf
05.Endocrine System 4 Blocks @eduwaves360.pdf
05.Endocrine System 4 Blocks @eduwaves360.pdf
05.Endocrine System 4 Blocks @eduwaves360.pdf
05.Endocrine System 4 Blocks @eduwaves360.pdf
05.Endocrine System 4 Blocks @eduwaves360.pdf
05.Endocrine System 4 Blocks @eduwaves360.pdf
05.Endocrine System 4 Blocks @eduwaves360.pdf

More Related Content

What's hot (20)

Bronchiectasis.
Bronchiectasis.Bronchiectasis.
Bronchiectasis.
 
Esophageal varices
Esophageal varicesEsophageal varices
Esophageal varices
 
Acute resp failure
Acute resp failureAcute resp failure
Acute resp failure
 
Sarcoidosis
SarcoidosisSarcoidosis
Sarcoidosis
 
Hypertension
HypertensionHypertension
Hypertension
 
Ankylosing spondylitis clinical features
Ankylosing spondylitis clinical featuresAnkylosing spondylitis clinical features
Ankylosing spondylitis clinical features
 
Chronic Obstructive Pulmonary Disease
Chronic Obstructive Pulmonary Disease  Chronic Obstructive Pulmonary Disease
Chronic Obstructive Pulmonary Disease
 
5.Bronchiectasis
5.Bronchiectasis5.Bronchiectasis
5.Bronchiectasis
 
Bronchiectasis
Bronchiectasis   Bronchiectasis
Bronchiectasis
 
Bronchiectasis
BronchiectasisBronchiectasis
Bronchiectasis
 
Peptic ulcer disease(PUD)
Peptic ulcer disease(PUD) Peptic ulcer disease(PUD)
Peptic ulcer disease(PUD)
 
Vasculitis syndromes
Vasculitis syndromesVasculitis syndromes
Vasculitis syndromes
 
Dypsnea
DypsneaDypsnea
Dypsnea
 
Enteritis
EnteritisEnteritis
Enteritis
 
L3 4 .copd
L3 4 .copdL3 4 .copd
L3 4 .copd
 
Approach to chronic cough
Approach to chronic coughApproach to chronic cough
Approach to chronic cough
 
Chronic obstructive pulmonary disease2
Chronic obstructive pulmonary disease2Chronic obstructive pulmonary disease2
Chronic obstructive pulmonary disease2
 
Copd Part 1
Copd Part 1Copd Part 1
Copd Part 1
 
Diabetic ketoacidosis ppt
Diabetic ketoacidosis pptDiabetic ketoacidosis ppt
Diabetic ketoacidosis ppt
 
Hyperosmolar hyperglycaemic state
Hyperosmolar  hyperglycaemic  stateHyperosmolar  hyperglycaemic  state
Hyperosmolar hyperglycaemic state
 

Similar to 05.Endocrine System 4 Blocks @eduwaves360.pdf

Answering exam questions diabetes
Answering exam questions diabetesAnswering exam questions diabetes
Answering exam questions diabeteskarendbrooks
 
Obesity: nutrients modulators of neuropeptides and neurotransmmitters
Obesity: nutrients modulators of neuropeptides and neurotransmmitters Obesity: nutrients modulators of neuropeptides and neurotransmmitters
Obesity: nutrients modulators of neuropeptides and neurotransmmitters Nutriline SRL
 
Obesidad: nutrientes moduladores de neuropeptidos y neurotransmisores
Obesidad: nutrientes moduladores de neuropeptidos y neurotransmisoresObesidad: nutrientes moduladores de neuropeptidos y neurotransmisores
Obesidad: nutrientes moduladores de neuropeptidos y neurotransmisoresNutriline SRL
 
Case study - DM 2, CKD 4
Case study - DM 2, CKD 4Case study - DM 2, CKD 4
Case study - DM 2, CKD 4Reynel Dan
 
Diabetic ketoacidosis: a case study
Diabetic ketoacidosis: a case studyDiabetic ketoacidosis: a case study
Diabetic ketoacidosis: a case studyLyndon Woytuck
 
Diabetes Cases.1 Ppt
Diabetes Cases.1 PptDiabetes Cases.1 Ppt
Diabetes Cases.1 PptMiami Dade
 
Type 2 dm etiology & reversibility (diabetes care, april 2013)
Type 2 dm etiology & reversibility (diabetes care, april 2013)Type 2 dm etiology & reversibility (diabetes care, april 2013)
Type 2 dm etiology & reversibility (diabetes care, april 2013)Endocrinology Department, BSMMU
 
Daily minimum nutritional requirements of the critically ill
Daily minimum nutritional requirements of the critically illDaily minimum nutritional requirements of the critically ill
Daily minimum nutritional requirements of the critically illRalekeOkoye
 
Diebetes mellitus type 1
Diebetes mellitus type 1Diebetes mellitus type 1
Diebetes mellitus type 1Priyank Ghanchi
 
Diabetes and Glucose Metabolism
Diabetes and Glucose MetabolismDiabetes and Glucose Metabolism
Diabetes and Glucose MetabolismPatrick Carter
 
DIABETES MELLITUS TYPE 1 & MANAGEMENT OF DIABETIC KETOACIDOSIS
DIABETES MELLITUS TYPE 1 & MANAGEMENT OF DIABETIC  KETOACIDOSIS DIABETES MELLITUS TYPE 1 & MANAGEMENT OF DIABETIC  KETOACIDOSIS
DIABETES MELLITUS TYPE 1 & MANAGEMENT OF DIABETIC KETOACIDOSIS Rakesh Verma
 
Imeglmin Slides agents in Types 2 Diabetes Mellitus
Imeglmin Slides agents in Types 2 Diabetes MellitusImeglmin Slides agents in Types 2 Diabetes Mellitus
Imeglmin Slides agents in Types 2 Diabetes Mellitusameetrathod4
 
Endocrine main [Autosaved].pptx for students
Endocrine main [Autosaved].pptx for studentsEndocrine main [Autosaved].pptx for students
Endocrine main [Autosaved].pptx for studentsEstibelMengist
 
Pharmacology Review & Perioperative Management of Diabetes
Pharmacology Review & Perioperative Management of Diabetes Pharmacology Review & Perioperative Management of Diabetes
Pharmacology Review & Perioperative Management of Diabetes Alan Todd, DNP, MSN, CRNA
 
Determination of Blood Glucose Using Glusose Oxidase-Peroxidase Method
Determination of Blood Glucose Using Glusose Oxidase-Peroxidase MethodDetermination of Blood Glucose Using Glusose Oxidase-Peroxidase Method
Determination of Blood Glucose Using Glusose Oxidase-Peroxidase MethodZoldylck
 

Similar to 05.Endocrine System 4 Blocks @eduwaves360.pdf (20)

Answering exam questions diabetes
Answering exam questions diabetesAnswering exam questions diabetes
Answering exam questions diabetes
 
Obesity: nutrients modulators of neuropeptides and neurotransmmitters
Obesity: nutrients modulators of neuropeptides and neurotransmmitters Obesity: nutrients modulators of neuropeptides and neurotransmmitters
Obesity: nutrients modulators of neuropeptides and neurotransmmitters
 
Obesidad: nutrientes moduladores de neuropeptidos y neurotransmisores
Obesidad: nutrientes moduladores de neuropeptidos y neurotransmisoresObesidad: nutrientes moduladores de neuropeptidos y neurotransmisores
Obesidad: nutrientes moduladores de neuropeptidos y neurotransmisores
 
Case study - DM 2, CKD 4
Case study - DM 2, CKD 4Case study - DM 2, CKD 4
Case study - DM 2, CKD 4
 
Diabetic ketoacidosis: a case study
Diabetic ketoacidosis: a case studyDiabetic ketoacidosis: a case study
Diabetic ketoacidosis: a case study
 
diabetes mellitus.pptx
diabetes mellitus.pptxdiabetes mellitus.pptx
diabetes mellitus.pptx
 
Diabetes Cases.1 Ppt
Diabetes Cases.1 PptDiabetes Cases.1 Ppt
Diabetes Cases.1 Ppt
 
Diagnostic Tests of Diabetes
Diagnostic Tests of DiabetesDiagnostic Tests of Diabetes
Diagnostic Tests of Diabetes
 
Type 2 dm etiology & reversibility (diabetes care, april 2013)
Type 2 dm etiology & reversibility (diabetes care, april 2013)Type 2 dm etiology & reversibility (diabetes care, april 2013)
Type 2 dm etiology & reversibility (diabetes care, april 2013)
 
Daily minimum nutritional requirements of the critically ill
Daily minimum nutritional requirements of the critically illDaily minimum nutritional requirements of the critically ill
Daily minimum nutritional requirements of the critically ill
 
Diebetes mellitus type 1
Diebetes mellitus type 1Diebetes mellitus type 1
Diebetes mellitus type 1
 
Diabetes and Glucose Metabolism
Diabetes and Glucose MetabolismDiabetes and Glucose Metabolism
Diabetes and Glucose Metabolism
 
diabetes & perio
 diabetes & perio diabetes & perio
diabetes & perio
 
DIABETES MELLITUS TYPE 1 & MANAGEMENT OF DIABETIC KETOACIDOSIS
DIABETES MELLITUS TYPE 1 & MANAGEMENT OF DIABETIC  KETOACIDOSIS DIABETES MELLITUS TYPE 1 & MANAGEMENT OF DIABETIC  KETOACIDOSIS
DIABETES MELLITUS TYPE 1 & MANAGEMENT OF DIABETIC KETOACIDOSIS
 
Imeglmin Slides agents in Types 2 Diabetes Mellitus
Imeglmin Slides agents in Types 2 Diabetes MellitusImeglmin Slides agents in Types 2 Diabetes Mellitus
Imeglmin Slides agents in Types 2 Diabetes Mellitus
 
Endocrine main [Autosaved].pptx for students
Endocrine main [Autosaved].pptx for studentsEndocrine main [Autosaved].pptx for students
Endocrine main [Autosaved].pptx for students
 
Pharmacology Review & Perioperative Management of Diabetes
Pharmacology Review & Perioperative Management of Diabetes Pharmacology Review & Perioperative Management of Diabetes
Pharmacology Review & Perioperative Management of Diabetes
 
diabetes.pdf
diabetes.pdfdiabetes.pdf
diabetes.pdf
 
Determination of Blood Glucose Using Glusose Oxidase-Peroxidase Method
Determination of Blood Glucose Using Glusose Oxidase-Peroxidase MethodDetermination of Blood Glucose Using Glusose Oxidase-Peroxidase Method
Determination of Blood Glucose Using Glusose Oxidase-Peroxidase Method
 
carbo case reports.pptx
carbo case reports.pptxcarbo case reports.pptx
carbo case reports.pptx
 

Recently uploaded

Full Stack Web Development Course for Beginners
Full Stack Web Development Course  for BeginnersFull Stack Web Development Course  for Beginners
Full Stack Web Development Course for BeginnersSabitha Banu
 
Introduction to AI in Higher Education_draft.pptx
Introduction to AI in Higher Education_draft.pptxIntroduction to AI in Higher Education_draft.pptx
Introduction to AI in Higher Education_draft.pptxpboyjonauth
 
Proudly South Africa powerpoint Thorisha.pptx
Proudly South Africa powerpoint Thorisha.pptxProudly South Africa powerpoint Thorisha.pptx
Proudly South Africa powerpoint Thorisha.pptxthorishapillay1
 
How to Configure Email Server in Odoo 17
How to Configure Email Server in Odoo 17How to Configure Email Server in Odoo 17
How to Configure Email Server in Odoo 17Celine George
 
Solving Puzzles Benefits Everyone (English).pptx
Solving Puzzles Benefits Everyone (English).pptxSolving Puzzles Benefits Everyone (English).pptx
Solving Puzzles Benefits Everyone (English).pptxOH TEIK BIN
 
Capitol Tech U Doctoral Presentation - April 2024.pptx
Capitol Tech U Doctoral Presentation - April 2024.pptxCapitol Tech U Doctoral Presentation - April 2024.pptx
Capitol Tech U Doctoral Presentation - April 2024.pptxCapitolTechU
 
Crayon Activity Handout For the Crayon A
Crayon Activity Handout For the Crayon ACrayon Activity Handout For the Crayon A
Crayon Activity Handout For the Crayon AUnboundStockton
 
call girls in Kamla Market (DELHI) 🔝 >༒9953330565🔝 genuine Escort Service 🔝✔️✔️
call girls in Kamla Market (DELHI) 🔝 >༒9953330565🔝 genuine Escort Service 🔝✔️✔️call girls in Kamla Market (DELHI) 🔝 >༒9953330565🔝 genuine Escort Service 🔝✔️✔️
call girls in Kamla Market (DELHI) 🔝 >༒9953330565🔝 genuine Escort Service 🔝✔️✔️9953056974 Low Rate Call Girls In Saket, Delhi NCR
 
Employee wellbeing at the workplace.pptx
Employee wellbeing at the workplace.pptxEmployee wellbeing at the workplace.pptx
Employee wellbeing at the workplace.pptxNirmalaLoungPoorunde1
 
CARE OF CHILD IN INCUBATOR..........pptx
CARE OF CHILD IN INCUBATOR..........pptxCARE OF CHILD IN INCUBATOR..........pptx
CARE OF CHILD IN INCUBATOR..........pptxGaneshChakor2
 
History Class XII Ch. 3 Kinship, Caste and Class (1).pptx
History Class XII Ch. 3 Kinship, Caste and Class (1).pptxHistory Class XII Ch. 3 Kinship, Caste and Class (1).pptx
History Class XII Ch. 3 Kinship, Caste and Class (1).pptxsocialsciencegdgrohi
 
Pharmacognosy Flower 3. Compositae 2023.pdf
Pharmacognosy Flower 3. Compositae 2023.pdfPharmacognosy Flower 3. Compositae 2023.pdf
Pharmacognosy Flower 3. Compositae 2023.pdfMahmoud M. Sallam
 
How to Make a Pirate ship Primary Education.pptx
How to Make a Pirate ship Primary Education.pptxHow to Make a Pirate ship Primary Education.pptx
How to Make a Pirate ship Primary Education.pptxmanuelaromero2013
 
Introduction to ArtificiaI Intelligence in Higher Education
Introduction to ArtificiaI Intelligence in Higher EducationIntroduction to ArtificiaI Intelligence in Higher Education
Introduction to ArtificiaI Intelligence in Higher Educationpboyjonauth
 
Biting mechanism of poisonous snakes.pdf
Biting mechanism of poisonous snakes.pdfBiting mechanism of poisonous snakes.pdf
Biting mechanism of poisonous snakes.pdfadityarao40181
 
DATA STRUCTURE AND ALGORITHM for beginners
DATA STRUCTURE AND ALGORITHM for beginnersDATA STRUCTURE AND ALGORITHM for beginners
DATA STRUCTURE AND ALGORITHM for beginnersSabitha Banu
 
Organic Name Reactions for the students and aspirants of Chemistry12th.pptx
Organic Name Reactions  for the students and aspirants of Chemistry12th.pptxOrganic Name Reactions  for the students and aspirants of Chemistry12th.pptx
Organic Name Reactions for the students and aspirants of Chemistry12th.pptxVS Mahajan Coaching Centre
 
Meghan Sutherland In Media Res Media Component
Meghan Sutherland In Media Res Media ComponentMeghan Sutherland In Media Res Media Component
Meghan Sutherland In Media Res Media ComponentInMediaRes1
 
Earth Day Presentation wow hello nice great
Earth Day Presentation wow hello nice greatEarth Day Presentation wow hello nice great
Earth Day Presentation wow hello nice greatYousafMalik24
 

Recently uploaded (20)

Full Stack Web Development Course for Beginners
Full Stack Web Development Course  for BeginnersFull Stack Web Development Course  for Beginners
Full Stack Web Development Course for Beginners
 
Introduction to AI in Higher Education_draft.pptx
Introduction to AI in Higher Education_draft.pptxIntroduction to AI in Higher Education_draft.pptx
Introduction to AI in Higher Education_draft.pptx
 
Proudly South Africa powerpoint Thorisha.pptx
Proudly South Africa powerpoint Thorisha.pptxProudly South Africa powerpoint Thorisha.pptx
Proudly South Africa powerpoint Thorisha.pptx
 
How to Configure Email Server in Odoo 17
How to Configure Email Server in Odoo 17How to Configure Email Server in Odoo 17
How to Configure Email Server in Odoo 17
 
Solving Puzzles Benefits Everyone (English).pptx
Solving Puzzles Benefits Everyone (English).pptxSolving Puzzles Benefits Everyone (English).pptx
Solving Puzzles Benefits Everyone (English).pptx
 
Capitol Tech U Doctoral Presentation - April 2024.pptx
Capitol Tech U Doctoral Presentation - April 2024.pptxCapitol Tech U Doctoral Presentation - April 2024.pptx
Capitol Tech U Doctoral Presentation - April 2024.pptx
 
Crayon Activity Handout For the Crayon A
Crayon Activity Handout For the Crayon ACrayon Activity Handout For the Crayon A
Crayon Activity Handout For the Crayon A
 
call girls in Kamla Market (DELHI) 🔝 >༒9953330565🔝 genuine Escort Service 🔝✔️✔️
call girls in Kamla Market (DELHI) 🔝 >༒9953330565🔝 genuine Escort Service 🔝✔️✔️call girls in Kamla Market (DELHI) 🔝 >༒9953330565🔝 genuine Escort Service 🔝✔️✔️
call girls in Kamla Market (DELHI) 🔝 >༒9953330565🔝 genuine Escort Service 🔝✔️✔️
 
Employee wellbeing at the workplace.pptx
Employee wellbeing at the workplace.pptxEmployee wellbeing at the workplace.pptx
Employee wellbeing at the workplace.pptx
 
CARE OF CHILD IN INCUBATOR..........pptx
CARE OF CHILD IN INCUBATOR..........pptxCARE OF CHILD IN INCUBATOR..........pptx
CARE OF CHILD IN INCUBATOR..........pptx
 
History Class XII Ch. 3 Kinship, Caste and Class (1).pptx
History Class XII Ch. 3 Kinship, Caste and Class (1).pptxHistory Class XII Ch. 3 Kinship, Caste and Class (1).pptx
History Class XII Ch. 3 Kinship, Caste and Class (1).pptx
 
Pharmacognosy Flower 3. Compositae 2023.pdf
Pharmacognosy Flower 3. Compositae 2023.pdfPharmacognosy Flower 3. Compositae 2023.pdf
Pharmacognosy Flower 3. Compositae 2023.pdf
 
How to Make a Pirate ship Primary Education.pptx
How to Make a Pirate ship Primary Education.pptxHow to Make a Pirate ship Primary Education.pptx
How to Make a Pirate ship Primary Education.pptx
 
Introduction to ArtificiaI Intelligence in Higher Education
Introduction to ArtificiaI Intelligence in Higher EducationIntroduction to ArtificiaI Intelligence in Higher Education
Introduction to ArtificiaI Intelligence in Higher Education
 
TataKelola dan KamSiber Kecerdasan Buatan v022.pdf
TataKelola dan KamSiber Kecerdasan Buatan v022.pdfTataKelola dan KamSiber Kecerdasan Buatan v022.pdf
TataKelola dan KamSiber Kecerdasan Buatan v022.pdf
 
Biting mechanism of poisonous snakes.pdf
Biting mechanism of poisonous snakes.pdfBiting mechanism of poisonous snakes.pdf
Biting mechanism of poisonous snakes.pdf
 
DATA STRUCTURE AND ALGORITHM for beginners
DATA STRUCTURE AND ALGORITHM for beginnersDATA STRUCTURE AND ALGORITHM for beginners
DATA STRUCTURE AND ALGORITHM for beginners
 
Organic Name Reactions for the students and aspirants of Chemistry12th.pptx
Organic Name Reactions  for the students and aspirants of Chemistry12th.pptxOrganic Name Reactions  for the students and aspirants of Chemistry12th.pptx
Organic Name Reactions for the students and aspirants of Chemistry12th.pptx
 
Meghan Sutherland In Media Res Media Component
Meghan Sutherland In Media Res Media ComponentMeghan Sutherland In Media Res Media Component
Meghan Sutherland In Media Res Media Component
 
Earth Day Presentation wow hello nice great
Earth Day Presentation wow hello nice greatEarth Day Presentation wow hello nice great
Earth Day Presentation wow hello nice great
 

05.Endocrine System 4 Blocks @eduwaves360.pdf

  • 1. www.eduwaves360.com | Telegram : @eduwaves360 Unlocked the Medical premiums Join us on Telegram : Click here : @eduwaves360 Click here : www.eduwaves360.com Medical Courses : https://t.me/usmle_study_materials_2 Discussion Group : @usmle_discussion_group
  • 2. Question # 1 A 53-year-old man comes to the physician because of fatigue, recurrent diarrhea, and an 8-kg (17.6-lb) weight loss over the past 6 months. He has a 4-month history of recurrent blistering rashes on different parts of his body that grow and develop into pruritic, crusty lesions before resolving spontaneously. Physical examination shows scaly lesions in different phases of healing with central, bronze-colored induration around the mouth, perineum, and lower extremities. Laboratory studies show: Hemoglobin 10.1 mg/dL Mean corpuscular volume 85 μm3 Mean corpuscular hemoglobin 30.0 pg/cell Serum Glucose 236 mg/dL Abdominal ultrasonography shows a 3-cm, solid mass located in the upper abdomen. This patient's mass is most likely derived from which of the following types of cells? Answer Image A Gastrointestinal enterochromaffin cells B Pancreatic α-cells C Pancreatic β-cells D Pancreatic δ-cells www.eduwaves360.com | Telegram : @eduwaves360
  • 3. Answer Image E Gastric G-cells www.eduwaves360.com | Telegram : @eduwaves360
  • 4. Hint This patient's rash, known as necrolytic migratory erythema, is strongly associated with his underlying tumor. www.eduwaves360.com | Telegram : @eduwaves360
  • 5. Correct Answer A - Gastrointestinal enterochromaffin cells Explanation Why Carcinoid tumors, which are functional neuroendocrine tumors derived from enterochromaffin cells, produce serotonin and can manifest with weight loss and recurrent diarrhea. These tumors are associated with skin lesions caused by niacin deficiency (e.g., casal necklace), but they are not associated with necrolytic migratory erythema. In addition, this patient lacks other typical manifestations of a carcinoid tumor, including flushing and wheezing. B - Pancreatic α-cells Explanation Why This patient has a glucagonoma, a functional neuroendocrine tumor derived from pancreatic α-cells. These tumors secrete glucagon, which increases gluconeogenesis and glycogenolysis and thus results in hyperglycemia. Other features of glucagonoma include chronic diarrhea, weight loss, neuropsychiatric symptoms (e.g., depression, dementia, ataxia), deep vein thrombosis, normocytic anemia (due to anemia of chronic disease and the inhibitory effect of glucagon on erythropoiesis), and necrolytic migratory erythema. The pathophysiology that underlies the development of necrolytic migratory erythema is not well understood, but it may be related to direct action of glucagon on the skin, protein deficiency (because glucagon causes proteolysis), and/or a deficiency of zinc and fatty acids due to chronic diarrhea. C - Pancreatic β-cells Explanation Why Insulinomas, which are functional neuroendocrine tumors derived from pancreatic β-cells, produce insulin and manifest with weight gain (not weight loss) and hypoglycemia (not hyperglycemia). Moreover, they are not associated with necrolytic migratory erythema. www.eduwaves360.com | Telegram : @eduwaves360
  • 6. D - Pancreatic δ-cells Explanation Why Somatostatinomas, which are functional neuroendocrine tumors derived from pancreatic δ-cells, secrete somatostatin and can manifest with weight loss and glucose intolerance. However, these tumors are not associated with necrolytic migratory erythema. In addition, this patient lacks other typical manifestations of a somatostatinoma, including abdominal pain, steatorrhea, and cholelithiasis. E - Gastric G-cells Explanation Why Gastrinomas, which are functional neuroendocrine tumors derived from gastric G-cells, secrete gastrin and can manifest with weight loss and recurrent diarrhea. However, these tumors are not associated with hyperglycemia or necrolytic migratory erythema. In addition, this patient lacks other typical manifestations of a gastrinoma, including abdominal pain and dyspepsia. www.eduwaves360.com | Telegram : @eduwaves360
  • 7. Question # 2 A previously healthy 41-year-old woman comes to the physician for the evaluation of recurrent episodes of palpitations and sweating over the past month. Her symptoms typically start after swimming practice and improve after drinking ice tea and eating some candy. She has also had a 5-kg (11-lb) weight gain over the past 3 months. She works as a nurse. Physical examination shows no abnormalities. Fasting serum studies show: Glucose 38 mg/dL Insulin 260 μU/mL (N=11–240) Proinsulin 65 μU/mL (N <20% of total insulin) C-peptide 5.0 ng/mL (N=0.8–3.1) Insulin secretagogues absent Which of the following is the most likely cause of her symptoms? Answer Image A Glucagon-producing tumor B Exogenous administration of insulin C Binge eating disorder D Pancreatic β-cell tumor E Factitious use of sulfonylureas www.eduwaves360.com | Telegram : @eduwaves360
  • 8. Answer Image F Peripheral resistance to insulin www.eduwaves360.com | Telegram : @eduwaves360
  • 9. Hint This patient's condition can be associated with MEN 1 syndrome. www.eduwaves360.com | Telegram : @eduwaves360
  • 10. Correct Answer A - Glucagon-producing tumor Explanation Why Glucagon increases blood glucose levels by stimulating gluconeogenesis and glycogenolysis. A glucagon-producing tumor (glucagonoma) typically manifests with weight loss, necrolytic migratory erythema, and impaired glucose tolerance or diabetes mellitus (hyperglycemia). However, this patient presents with weight gain, hypoglycemia, and laboratory evidence of endogenous hyperinsulinemia (↑ insulin, ↑ proinsulin, ↑ C-peptide). B - Exogenous administration of insulin Explanation Why Inappropriate exogenous administration of insulin (e.g., malingering) can result in weight gain and symptomatic hypoglycemia that is corrected with the intake of sugary foods, as in this patient. Her profession as a nurse also gives her easy access to insulin. However, this patient's elevated C-peptide and proinsulin levels are evidence that the insulin is endogenously produced, as C-peptide is released during cleavage of insulin from proinsulin. C - Binge eating disorder Explanation Why Binge eating disorder would also present with weight gain and increased endogenous insulin levels (as well as C-peptide and proinsulin) secondary to increased nutritional intake. However, hypoglycemia is not a feature of binge eating disorder. www.eduwaves360.com | Telegram : @eduwaves360
  • 11. D - Pancreatic β-cell tumor Explanation Why Insulin is secreted by pancreatic β-cells and promotes cellular uptake of glucose. Thus, a pancreatic β-cell tumor (insulinoma) manifests as hypoglycemia, which would explain this patient's post- exertional autonomic symptoms (sweating and palpitations) that resolve after consuming sugary foods. These features, together with the documented hypoglycemia are known as Whipple triad, a characteristic feature of insulinoma. Insulin also stimulates lipogenesis and inhibits lipolysis, which explains the weight gain in this patient. As C-peptide is released during cleavage of insulin from proinsulin, the high levels of serum C-peptide and proinsulin on serum analysis confirm that the hyperinsulinemia is endogenous, making insulinoma the most likely diagnosis. E - Factitious use of sulfonylureas Explanation Why Sulfonylureas are insulin secretagogues. Individuals using these drugs factitiously would present with weight gain, features of hypoglycemia, and elevated serum insulin, proinsulin, and C-peptide levels, as seen in this patient. Her profession as a nurse also gives her easy access to sulfonylureas. However, patients using sulfonylureas would have insulin secretagogues on laboratory analysis, which is absent in this patient. F - Peripheral resistance to insulin Explanation Why Patients with type 2 diabetes mellitus have peripheral resistance to insulin and may have elevated insulin, proinsulin, and C-peptide. However, type 2 diabetics will have hyperglycemia – not hypoglycemia. In addition, untreated type 2 diabetes causes weight loss due to the insulin resistance since insulin is an anabolic hormone. www.eduwaves360.com | Telegram : @eduwaves360
  • 12. Question # 3 A 48-year-old woman comes to the physician for the evaluation of 24-hour blood pressure monitoring results. Over the last 3 months, she has had intermittent nausea, decreased appetite, and increasing weakness and fatigue during the day. She has been treated twice for kidney stones within the past year. Her current medications include lisinopril, amlodipine, and furosemide. She is 178 cm (5 ft 10 in) tall and weighs 97 kg (214 lb); BMI is 31 kg/m2 . Her blood pressure is 152/98 mm Hg. Physical examination shows no abnormalities. Serum studies show: Na+ 141 mEq/L Cl− 101 mEq/L K+ 4.5 mEq/L HCO3 − 24 mEq/L Calcium 12.9 mg/dL Creatinine 1.0 mg/dL Twenty-four-hour blood pressure monitoring indicates elevated nocturnal blood pressure. Further evaluation is most likely to show which of the following findings? Answer Image A Increased serum aldosterone-to- renin ratio B Increased 24-hour urine cortisol www.eduwaves360.com | Telegram : @eduwaves360
  • 13. Answer Image C Increased serum parathyroid hormone D Decreased serum thyroid- stimulating hormone E Decreased renal blood flow F Decreased nocturnal oxygen saturation G Increased serum insulin-like growth factor 1 www.eduwaves360.com | Telegram : @eduwaves360
  • 14. Hint This patient has resistant hypertension despite treatment with 3 antihypertensive drugs, which should raise concern for secondary hypertension. She also has nephrolithiasis (stones), nausea and decreased appetite (groans), fatigue, and weakness. www.eduwaves360.com | Telegram : @eduwaves360
  • 15. Correct Answer A - Increased serum aldosterone-to-renin ratio Explanation Why Elevated serum aldosterone-to-renin ratio indicates primary hyperaldosteronism (Conn syndrome). Primary hyperaldosteronism can cause secondary hypertension with features of hypokalemia (e.g., fatigue and muscle weakness). However, this patient's potassium level is normal. Moreover, hypercalcemia and symptoms such as recurrent renal stones or gastrointestinal complaints would not be expected in Conn syndrome. B - Increased 24-hour urine cortisol Explanation Why Elevated 24-hour urine cortisol is diagnostic of Cushing syndrome (hypercortisolism), which can cause of treatment-resistant secondary hypertension with lethargy and muscle weakness. However, it typically also presents with central obesity, thin and easily bruisable skin, abdominal striae, and proximal muscle atrophy, which are not seen in this patient. www.eduwaves360.com | Telegram : @eduwaves360
  • 16. C - Increased serum parathyroid hormone Image Explanation But Primary hyperparathyroidism is also a common finding in MEN 1 and MEN 2A. MEN 2A, in particular, often also involves pheochromocytoma, another possible (concurrent) cause of secondary hypertension that should be considered in this patient. Explanation Why Increased serum parathyroid hormone would be found in primary hyperparathyroidism, a known cause of secondary hypertension, especially in patients with hypercalcemia, as seen here. Hypercalcemia, occurring as a result of primary hyperparathyroidism or otherwise, is thought to cause hypertension by affecting vascular reactivity and interrupting day-night blood pressure regulation. The most common causes of hypercalcemia are hyperparathyroidism and malignancy. www.eduwaves360.com | Telegram : @eduwaves360
  • 17. D - Decreased serum thyroid-stimulating hormone Image Explanation Why Decreased serum thyroid-stimulating hormone (TSH) would indicate hyperthyroidism, which is an important cause of treatment-resistant secondary hypertension and may present with weakness and fatigue, as seen here. Hyperthyroidism could also explain this patient's hypercalcemia, as thyroid hormones have osteoclastic activity. However, decreased appetite is not typical of hyperthyroidism, and this patient is lacking other symptoms that would be expected in the setting of thyroid hormone excess (e.g., heat intolerance, weight loss, diarrhea, tachycardia, tremor). E - Decreased renal blood flow Explanation Why Decreased renal blood flow due to renal artery stenosis or fibromuscular dysplasia is a common www.eduwaves360.com | Telegram : @eduwaves360
  • 18. cause of treatment-resistant secondary hypertension, which is often accompanied by fatigue and weakness. Decreased perfusion triggers the renin-angiotensin-aldosterone system and causes persistent vasoconstriction as well as volume retention. However, renal artery stenosis would not explain this patient's hypercalcemia or recurrent kidney stones. Furthermore, constantly elevated aldosterone would lead to hypokalemia, which this patient does not have. F - Decreased nocturnal oxygen saturation Explanation Why Decreased nocturnal oxygen saturation is characteristic of patients with obstructive sleep apnea (OSA), which can cause secondary hypertension and excessive daytime fatigue. However, OSA would not explain this patient's hypercalcemia, recurrent kidney stones, nausea, or decreased appetite. G - Increased serum insulin-like growth factor 1 Explanation Why Elevated serum insulin-like growth factor 1 in an adult can indicate acromegaly, which may cause secondary hypertension. However, acromegaly presents with enlarged hands and feet, deepening of the voice, coarsened facial features, doughy skin, oligomenorrhea in women, and possible pituitary mass effect (headache, bilateral hemianopsia). www.eduwaves360.com | Telegram : @eduwaves360
  • 19. Question # 4 A 23-year-old woman comes to physician for an annual health maintenance examination. She feels well. She is 155 cm (5 ft 1 in) tall and weighs 79 kg (174 lb); BMI is 33 kg/m2 . Examination shows a skin rash over both axillae. A photograph of her left axilla is shown. This patient's skin finding is most likely associated with which of the following conditions? Answer Image A Niacin deficiency www.eduwaves360.com | Telegram : @eduwaves360
  • 20. Answer Image B Gastric malignant lymphoma C Primary adrenal insufficiency D Polycystic ovarian syndrome E Hashimoto thyroiditis www.eduwaves360.com | Telegram : @eduwaves360
  • 21. Answer Image F Pregnancy www.eduwaves360.com | Telegram : @eduwaves360
  • 22. Hint The picture shows acanthosis nigricans, a skin condition that manifests with thickened, velvety, hyperpigmented plaques and most frequently involves intertriginous sites such as the axillae or the neck. www.eduwaves360.com | Telegram : @eduwaves360
  • 23. Correct Answer A - Niacin deficiency Image Explanation But Pellagra and acanthosis nigricans can both present with a collar around the neck; also referred to as Casal necklace in the case of pellagra. However, the skin in niacin deficiency looks sun-burnt rather than velvety as seen here. Explanation Why Niacin deficiency causes pellagra, which presents with a symmetrical, thickened, and hyperpigmented rash. However, the rash typically occurs in areas of the skin exposed to the sunlight (e.g., face, neck, hands) rather than in the axillae, as seen here. Moreover, further symptoms of pellagra, such as vomiting, diarrhea, atrophic glossitis, and neurological symptoms are absent in this patient. Niacin deficiency is not associated with acanthosis nigricans. www.eduwaves360.com | Telegram : @eduwaves360
  • 24. B - Gastric malignant lymphoma Image Explanation Why Gastrointestinal malignancies can manifest with acanthosis nigricans. However, if this skin finding occurs in association with a malignancy, such as paraneoplastic syndrome, it is characterized by rapid onset, a verrucous or papulous surface, and involvement of atypical sites (e.g., palms, soles, or mucous membranes), which is not the case in this patient. Moreover, malignant acanthosis nigricans is associated with gastric adenocarcinoma, not gastric malignant lymphoma. Lastly, gastric cancer is rare in young adults and this woman has no signs of gastrointestinal malignancy such as weight loss, anemia, abdominal discomfort, or melena. www.eduwaves360.com | Telegram : @eduwaves360
  • 25. C - Primary adrenal insufficiency Image Explanation Why Primary adrenal insufficiency (Addison disease) presents with hyperpigmented skin lesions caused by the stimulant effects of excess ACTH on the melanocytes which produce melanin (bronze disease). Unlike the skin finding in this patient, the hyperpigmentation in Addison disease is typically generalized and often involves the palm creases or oral mucosa. Instead of obesity, patients with Addison disease present with weight loss and anorexia due to hypocortisolism. Lastly, the absence of other symptoms such as hypotension and intestinal complaints (nausea, vomiting, and/or diarrhea), make Addison disease unlikely. www.eduwaves360.com | Telegram : @eduwaves360
  • 26. D - Polycystic ovarian syndrome Image Explanation But Similar skin changes can be caused by obesity alone, in which case the disorder is referred to as pseudoacanthosis nigricans. Explanation Why Acanthosis nigricans is commonly seen in disorders characterized by insulin resistance and hyperinsulinemia (e.g., polycystic ovarian syndrome, obesity, diabetes mellitus, acromegaly). Elevated levels of insulin stimulate keratinocyte and dermal fibroblast proliferation via interaction with insulin-like growth factor 1, resulting in epidermal hyperplasia and plaque-like lesions, as seen in this patient. In addition, insulin resistance in PCOS is responsible for hyperandrogenism (leading to acne, hirsutism) and anovulation (causing amenorrhea and impaired fertility). www.eduwaves360.com | Telegram : @eduwaves360
  • 27. E - Hashimoto thyroiditis Explanation Why Hashimoto thyroiditis usually leads to clinical hypothyroidism, which often presents with obesity and skin abnormalities. However, findings typically include dry, cold, pale, and scaly skin rather than the skin lesion described in this patient. Furthermore, the absence of additional features of hypothyroidism such as fatigue, hair loss, constipation, and/or goiter make Hashimoto thyroiditis unlikely. Hashimoto thyroiditis is not commonly associated with acanthosis nigricans. F - Pregnancy Image Explanation Why Pregnancy is associated with hyperpigmentation due to increased melanocyte-stimulating hormone produced by the placenta. Although the axillae may be affected, hyperpigmentation of the linea alba, www.eduwaves360.com | Telegram : @eduwaves360
  • 28. the nipples, areola, and the face (i.e., melasma) are far more common. Moreover, the hyperpigmented macules and patches often seen during pregnancy are not thickened and velvety as seen here. Pregnancy is not associated with acanthosis nigricans. www.eduwaves360.com | Telegram : @eduwaves360
  • 29. Question # 5 A 17-year-old boy with type 1 diabetes mellitus comes to the emergency department because of nausea and abdominal pain during the past 6 hours. His finger stick blood glucose concentration is 409 mg/dL. Urinalysis shows the presence of ketone bodies. Which of the following results of arterial blood gas analysis is most likely before treatment? pH PaCO2 (mm Hg) HCO3 - (mEq/L) A 7.52 52 41 B 7.46 29 20 C 7.28 51 23 D 7.29 30 14 E 7.49 33 24 F 7.34 56 29 Answer Image A A B B C C www.eduwaves360.com | Telegram : @eduwaves360
  • 30. Answer Image D D E E F F www.eduwaves360.com | Telegram : @eduwaves360
  • 31. Hint This patient with type 1 diabetes mellitus presenting with abdominal pain, nausea, acute hyperglycemia, and ketonuria most likely has an increased anion gap. www.eduwaves360.com | Telegram : @eduwaves360
  • 32. Correct Answer A - A Explanation Why Increased pH with increased PaCO2 and increased HCO3 - suggests a metabolic alkalosis, which is typically caused by severe vomiting, diuretic use, hyperaldosteronism, or Cushing syndrome. This patient can develop a metabolic alkalosis due to severe vomiting and/or hypovolemia but a different primary acid-base disorder would be expected. B - B Explanation Why Mildly increased pH with decreased PaCO2 and decreased HCO3 - suggests a respiratory alkalosis that has been compensated for by decreased renal reabsorption of bicarbonate (chronic respiratory alkalosis). Respiratory alkalosis is typically caused by hyperventilation due to anxiety, certain drug toxicities (e.g., aspirin), or mechanical ventilation. This patient with ketonuria will hyperventilate as a compensatory response but the primary acid-base disorder is not a respiratory alkalosis. C - C Explanation Why Decreased pH with increased PaCO2 and a normal HCO3 - suggests a respiratory acidosis that has not yet been compensated for by the kidneys (acute respiratory acidosis). Respiratory acidosis is typically caused by airway obstruction (e.g., COPD, bronchial asthma) or hypoventilation due to respiratory muscle weakness or CNS depression. A different acid-base disorder would be expected in this patient with ketonuria. www.eduwaves360.com | Telegram : @eduwaves360
  • 33. D - D Image Explanation Why This patient with type 1 diabetes mellitus presents with clinical features of diabetic ketoacidosis (DKA). DKA occurs when insulin deficiency stimulates lipolysis and increases the production of ketone bodies (e.g., acetoacetic acid) from free fatty acids. The presence of these inorganic acids lowers pH and increases the anion gap. In order to buffer these acidic ketones, HCO3 - is consumed. To compensate for metabolic acidosis, minute ventilation increases and PaCO2 is lowered, resulting in an arterial blood gas pattern as seen here. www.eduwaves360.com | Telegram : @eduwaves360
  • 34. E - E Explanation Why Increased pH with decreased PaCO2 and normal HCO3 - suggests a respiratory alkalosis that has not yet been compensated for by the kidneys (acute respiratory alkalosis). Respiratory alkalosis is typically caused by hyperventilation due to anxiety, certain drug toxicities (e.g., aspirin), or mechanical ventilation. This patient with ketonuria will hyperventilate as a compensatory response but the primary acid-base disorder is not a respiratory alkalosis. F - F Explanation Why Mildly decreased pH with increased PaCO2 and increased bicarbonate suggests a respiratory acidosis that has been compensated for by increased renal reabsorption of bicarbonate (chronic respiratory acidosis). Respiratory acidosis is typically caused by airway obstruction (e.g., COPD, bronchial asthma) or hypoventilation due to respiratory muscle weakness or CNS depression. A different acid- base disorder would be expected in this patient with ketonuria. www.eduwaves360.com | Telegram : @eduwaves360
  • 35. Question # 6 A 47-year-old woman comes to the physician for a 2-month history of fatigue, intermittent left-sided flank pain, and diffuse extremity pain. She was treated for a prolactinoma 5 years ago and has had recurrent antral and duodenal peptic ulcers despite medical therapy. Her only medication is omeprazole. Physical examination shows a moderately distended abdomen that is diffusely tender to palpation. There is mild left-sided costovertebral angle tenderness. Serum studies show a calcium concentration of 12 mg/dL, phosphorus concentration of 2 mg/dL, and parathyroid hormone level of 426 pg/mL. An ultrasound of the abdomen shows multiple kidney stones in the left ureteropelvic junction. This patient is most likely to have which of the following protein abnormalities? Answer Image A Inactivation of RET proto-oncogene B Altered merlin protein expression C Altered menin protein D Mutation of VHL tumor suppressor E Mutation in C-Kit protein www.eduwaves360.com | Telegram : @eduwaves360
  • 36. Hint This patient has evidence of hyperparathyroidism (hypercalcemia, elevated PTH level) and a history of recurrent ulcers, which suggests Zollinger-Ellison syndrome. The constellation of parathyroid, pituitary, and pancreatic disease should raise suspicion for multiple endocrine neoplasia type 1 (MEN1). www.eduwaves360.com | Telegram : @eduwaves360
  • 37. Correct Answer A - Inactivation of RET proto-oncogene Explanation Why Pathologic inactivation of the RET gene causes Hirschsprung disease, a congenital abnormality of the colon that results in bilious emesis, abdominal distention, and failure to pass meconium. It is not associated with the parathyroid, pituitary, and pancreatic abnormalities seen in this patient. B - Altered merlin protein expression Explanation Why Altered merlin expression is the hallmark of neurofibromatosis type 2, which is characterized by bilateral acoustic neuromas, juvenile cataracts, and meningiomas. NF2 would not explain the parathyroid, pancreatic, and pituitary abnormalities seen in this patient. C - Altered menin protein Explanation Why Altered menin protein function resulting from a mutation in the MEN1 gene is seen in multiple endocrine neoplasia type 1. MEN1 is characterized by the 3 P's: pituitary tumors (prolactinoma), parathyroid gland tumors (resulting in primary hyperparathyroidism), and pancreatic tumors (such as gastrinoma or insulinoma). MEN1 is also associated with angiofibromas and meningiomas. www.eduwaves360.com | Telegram : @eduwaves360
  • 38. D - Mutation of VHL tumor suppressor Explanation Why VHL gene mutations can result in von Hippel-Lindau disease, which is characterized by bilateral renal cell carcinoma (RCC). RCC may cause secretion of parathyroid hormone-related protein, which can also lead to symptoms of hyperparathyroidism (as seen in this patient). However, this patient lacks other distinguishing features of VHL, such as angiomatosis, pheochromocytomas, and hemangioblastomas. E - Mutation in C-Kit protein Explanation Why Proto-oncogene c-Kit is a cytokine receptor oncogene that can be found in gastrointestinal stromal tumors (GIST), as well as mastocytomas, seminomas, embryonal carcinomas, and myeloid leukemias. However, none of these malignancies would explain this patient's constellation of parathyroid, pituitary, and pancreatic abnormalities. www.eduwaves360.com | Telegram : @eduwaves360
  • 39. Question # 7 A 33-year-old woman comes to the physician because of a 4-day history of fever, anterior neck pain, and throat swelling. She has no history of serious illness. Her temperature is 38.1°C (100.6°F) and pulse is 109/min. Physical examination shows diaphoresis and a fine tremor of the outstretched hands. The thyroid gland is enlarged, firm, and tender to palpation. Serum thyroid stimulating hormone level is 0.06 μU/mL and erythrocyte sedimentation rate is 65 mm/h. 123 I scan shows an enlarged thyroid gland with diffusely decreased uptake. Histologic examination of a thyroid biopsy specimen is most likely to show which of the following findings? Answer Image A Follicular epithelial cell hyperplasia B Undifferentiated giant cells with areas of necrosis and hemorrhage C Concentric intracellular lamellar calcifications D Lymphocytic infiltration with germinal follicle formation E Noncaseating granulomas with multinucleated giant cells www.eduwaves360.com | Telegram : @eduwaves360
  • 40. Hint Signs of hyperthyroidism, a tender and enlarged thyroid gland, and an elevated ESR in a young female patient are suggestive of De Quervain thyroiditis, a type of subacute thyroiditis. www.eduwaves360.com | Telegram : @eduwaves360
  • 41. Correct Answer A - Follicular epithelial cell hyperplasia Explanation Why Diffuse follicular cell hyperplasia can be seen in Graves disease, the most common cause of hyperthyroidism. Graves disease can also present with symptoms of hyperthyroidism (e.g., tachycardia, diaphoresis, resting tremor) and a diffusely enlarged goiter. However, it would not explain the tenderness of her goiter or her elevated ESR. Furthermore, an iodine uptake scan in Graves disease would show diffusely increased uptake of radioactive iodine. B - Undifferentiated giant cells with areas of necrosis and hemorrhage Explanation Why Undifferentiated giant cells with areas of necrosis and hemorrhage are characteristic findings in anaplastic thyroid carcinoma. It is an aggressive malignancy that is most commonly seen in elderly populations. Anaplastic thyroid carcinoma typically presents with a rapidly enlarging thyroid mass that may cause dysphagia, and usually lacks the signs or symptoms of hyperthyroidism seen in this patient. C - Concentric intracellular lamellar calcifications Explanation Why Concentric lamellar calcifications are consistent with psammoma bodies, which can be seen in papillary thyroid carcinoma. This disease is usually characterized by painless thyroid nodules with focal areas of decreased uptake on iodine uptake scan. This patient's painful goiter and diffusely decreased uptake on iodine uptake scan are not consistent with papillary thyroid carcinoma. www.eduwaves360.com | Telegram : @eduwaves360
  • 42. D - Lymphocytic infiltration with germinal follicle formation Explanation Why Diffuse lymphocytic infiltration with germinal center formation is seen in Hashimoto thyroiditis, the most common cause of hypothyroidism in the US. Although Hashimoto thyroiditis may initially have a hyperthyroid phase (e.g., Hashitoxicosis), the disease is typically asymptomatic and characterized by painless goiter. This patient's tender thyroid exam is not consistent with Hashimoto thyroiditis. E - Noncaseating granulomas with multinucleated giant cells Explanation Why De Quervain thyroiditis is a transient and self-limited disease characterized by patchy granulomatous inflammation of the thyroid. This disease is often preceded by a viral upper respiratory tract infection and typically presents initially with hyperthyroidism (as seen in this patient), followed by hypothyroidism. Classic signs include thyroid tenderness, increased ESR, and jaw pain. www.eduwaves360.com | Telegram : @eduwaves360
  • 43. Question # 8 A 27-year-old man comes to the physician because of a 4-month history of unintentional weight gain, fatigue, and decreased sexual desire. There is no personal or family history of serious illness. His blood pressure is 149/88 mm Hg. Physical examination shows central obesity and abdominal striae. He has a prominent soft tissue bulge at the dorsum of his neck. Laboratory studies show a 24-hour urinary free cortisol of 200 μg (N < 50) and a morning serum ACTH of 1 pg/mL (N = 7–50). Which of the following tests is most likely to confirm the underlying etiology of this patient's symptoms? Answer Image A CRH stimulation test B ACTH stimulation test C Low-dose dexamethasone suppression test D Petrosal vein catheterization E Chest CT F Abdominal CT www.eduwaves360.com | Telegram : @eduwaves360
  • 44. Answer Image G Brain MRI www.eduwaves360.com | Telegram : @eduwaves360
  • 45. Hint This patient presents with typical features of Cushing syndrome, including weight gain, truncal obesity, striae, hypertension, fatigue, and decreased sexual desire. High cortisol levels in conjunction with low ACTH levels indicate primary hypercortisolism. www.eduwaves360.com | Telegram : @eduwaves360
  • 46. Correct Answer A - CRH stimulation test Explanation Why CRH stimulation test can be used in patients with ACTH-dependent hypercortisolism to differentiate whether the source of excess ACTH production is pituitary (Cushing disease) or ectopic. If serum ACTH and cortisol levels increase further after CRH stimulation, Cushing disease is confirmed; minimal or no change in their levels suggests ectopic ACTH production. Since the patient presents with ACTH-independent hypercortisolism, this test would not be indicated. B - ACTH stimulation test Explanation Why ACTH stimulation test is used to test the response of the adrenal glands to ACTH in patients suspected of having adrenal insufficiency (e.g., hypocortisolism), such as Addison disease. Since this patient presents with hypercortisolism, this test would not be indicated. C - Low-dose dexamethasone suppression test Explanation But High-dose dexamethasone suppression test can be used in patients with ACTH-dependent hypercortisolism to differentiate whether the source of ACTH production is pituitary (Cushing disease) or ectopic. Explanation Why A low-dose dexamethasone suppression test can be used in the initial screening for hypercortisolism. In normal individuals, dexamethasone suppresses ACTH secretion and thus decreases the level of cortisol. However, in this patient, the diagnosis of hypercortisolism using a 24–hour urine sample as www.eduwaves360.com | Telegram : @eduwaves360
  • 47. well as a decreased ACTH level in serum has already been established, indicating an ACTH- independent Cushing syndrome. Therefore a low-dose dexamethasone suppression test would not be helpful in confirming the cause of his symptoms. D - Petrosal vein catheterization Explanation Why Petrosal vein catheterization is performed in patients with suspected ACTH-secreting pituitary adenoma if brain MRI studies are unable to identify pituitary adenoma but clinical suspicion is still high. However, ACTH levels in secondary hypercortisolism would either be normal or elevated, as opposed to this patient's low ACTH levels. E - Chest CT Explanation Why A chest CT would be helpful if small cell lung cancer, which can lead to paraneoplastic, ectopic ACTH production and subsequent secondary hypercortisolism, were suspected. However, ectopic ACTH production would present with normal or elevated ACTH levels, as opposed to this patient's low ACTH. Also, this patient lacks other symptoms of lung cancer, such as cough, hemoptysis, dyspnea, and chest pain. www.eduwaves360.com | Telegram : @eduwaves360
  • 48. F - Abdominal CT Image Explanation But Prolonged glucocorticoid therapy is the most common cause of hypercortisolism. Explanation Why This patient presents with symptoms and laboratory findings consistent with ACTH-independent Cushing syndrome due to autonomous overproduction of cortisol by the adrenal gland. Once the Cushing's syndrome has been determined to be ACTH-independent, a thin-section CT or MRI of the adrenal glands should be performed to evaluate for adrenal mass. Common etiologies for excess adrenal cortisol production include adrenal adenoma, adrenal carcinoma, and adrenal hyperplasia. www.eduwaves360.com | Telegram : @eduwaves360
  • 49. G - Brain MRI Explanation Why MRI of the brain may be used to visualize pituitary masses in patients with suspected Cushing disease due to ACTH-secreting pituitary adenoma (e.g., secondary hypercortisolism). However, ACTH levels in secondary hypercortisolism would either be normal or elevated, as opposed to this patient's low ACTH levels. www.eduwaves360.com | Telegram : @eduwaves360
  • 50. Question # 9 A 45-year-old woman comes to the physician because of fatigue, irregular menses, and recurrent respiratory infections for the past 6 months. Her blood pressure is 151/82 mm Hg. Physical examination shows a round face, thinning of the skin, and multiple bruises on her arms. Further studies confirm the diagnosis of an ACTH-secreting pituitary adenoma. This patient is at greatest risk for which of the following? Answer Image A Weight loss B Eosinophilia C Hypoglycemia D Bitemporal hemianopsia E Pathologic fracture F Hyperkalemia www.eduwaves360.com | Telegram : @eduwaves360
  • 51. Hint This patient presents with typical findings of hypercortisolism (e.g., moon facies, bruisable skin, irregular menstrual cycle, recurrent infections, hypertension) due to an ACTH-secreting pituitary adenoma (Cushing disease). www.eduwaves360.com | Telegram : @eduwaves360
  • 52. Correct Answer A - Weight loss Explanation Why Cushing disease is associated with weight gain rather than weight loss. Patients with Cushing disease have a characteristic pattern of weight gain and fat deposition in the neck (buffalo hump), trunk, and abdomen (central obesity), as well as the face (round, moon facies). B - Eosinophilia Explanation Why Eosinophilia is a hematologic finding associated with chronic adrenal insufficiency, not Cushing syndrome. This patient has a cortisol-secreting pituitary adenoma and clinical features of hypercortisolism – not hypocortisolism, as would be seen in adrenal insufficiency. C - Hypoglycemia Explanation Why Cushing disease is most commonly associated with hyperglycemia rather than hypoglycemia. Hypercortisolism raises blood glucose by increasing insulin resistance and hepatic uptake of glucose to be used in gluconeogenesis. www.eduwaves360.com | Telegram : @eduwaves360
  • 53. D - Bitemporal hemianopsia Explanation Why Bitemporal hemianopsia is a complication of pituitary macroadenomas (size > 10 mm) such as prolactinoma, gonadotropinomas, somatotropinomas, and thyrotropinomas. Bitemporal hemianopsia is caused by compression of the optic nerve at the level of the optic chiasm. While this patient's hypercortisolism is due to a pituitary adenoma, she is unlikely to develop bitemporal hemianopsia, as Cushing disease is most commonly caused by microadenomas (size < 10 mm) rather than macroadenomas. E - Pathologic fracture Image Explanation Why Cushing disease is associated with pathological fractures due to osteoporosis. Excess cortisol inhibits www.eduwaves360.com | Telegram : @eduwaves360
  • 54. calcitriol and osteoblasts maturation while simultaneously increasing production of RANKL, which stimulates osteoclasts and results in increased levels of bone resorption. Other features of Cushing disease include cataracts, peptic ulcer disease, dyslipidemia, muscle atrophy/weakness, acne, hirsutism, and psychiatric conditions such as generalized anxiety disorder, major depressive disorder, and psychosis. F - Hyperkalemia Explanation Why Cushing disease is associated with hypokalemia rather than hyperkalemia. Hypokalemia is caused by the mineralocorticoid effect of cortisol on aldosterone receptors, which stimulates sodium and water reabsorption (causing hypertension), potassium excretion, and metabolic alkalosis. Hyperkalemia is usually seen in hypocortisolism as part of primary adrenal insufficiency. This patient has hypercortisolism. www.eduwaves360.com | Telegram : @eduwaves360
  • 55. Question # 10 A 38-year-old woman comes to the physician because of a 4-month history of crampy abdominal pain, recurrent watery diarrhea, and a 2.5-kg (5.5-lb) weight loss. Her husband has noticed that after meals, her face and neck sometimes become red, and she develops shortness of breath and starts wheezing. Examination shows a grade 3/6 systolic murmur heard best at the left lower sternal border. The abdomen is soft, and there is mild tenderness to palpation with no guarding or rebound. Without treatment, this patient is at greatest risk of developing which of the following conditions? Answer Image A Achlorhydria B Laryngeal edema C Pigmented dermatitis D Megaloblastic anemia E T-cell lymphoma www.eduwaves360.com | Telegram : @eduwaves360
  • 56. Hint Watery diarrhea, crampy abdominal pain, episodic cutaneous flushing triggered by eating or emotional events, asthma-like attacks, and a murmur indicative of tricuspid regurgitation are highly suggestive of carcinoid syndrome. The neuroendocrine tumor is highly metabolically active and secretes excess amounts of serotonin. www.eduwaves360.com | Telegram : @eduwaves360
  • 57. Correct Answer A - Achlorhydria Explanation Why Increased gastric pH levels are not associated with carcinoid syndrome but are seen in achlorhydria, which is a key feature of WDHA syndrome (watery diarrhea, hypokalemia, achlorhydria) seen in VIPomas. The unregulated production of VIP leads to excessive secretion of water and electrolytes into the intestinal lumen. Cutaneous flushing is seen in about 20% of patients with VIPoma. However, abdominal pain is usually mild or absent. Furthermore, this etiology does not explain why the patient also presents with a tricuspid murmur and asthma, neither of which are typical findings of VIPomas. B - Laryngeal edema Explanation Why Laryngeal edema is not associated with carcinoid syndrome but commonly occurs in hereditary angioedema, a bradykinin-mediated reaction that can also cause colicky abdominal pain and diarrhea. However, episodic cutaneous flushing and a tricuspid regurgitation murmur would not be expected. Although mast cell-mediated angioedema can cause flushing, it does not present with abdominal pain or diarrhea. www.eduwaves360.com | Telegram : @eduwaves360
  • 58. C - Pigmented dermatitis Image Explanation But In developed countries, pellagra is most commonly seen in patients with alcohol use disorder or as a complication of malabsorption. It can also occur in Hartnup disease (due to renal and intestinal loss of tryptophan) or after prolonged use of isoniazid (due to decreased levels of vitamin B6, which is essential for the production of niacin). Explanation Why Dermatitis is a characteristic feature of pellagra, or niacin deficiency; other key findings include dementia and diarrhea (i.e., the “three Ds”). Dermatitis in patients with pellagra is characterized by a symmetrical, pigmented, scaly thickening of the skin in sun-exposed areas, such as the limbs and around the neck (casal necklace). The increased metabolism of serotonin in carcinoid tumors depletes the amount of tryptophan precursors available for other organic compounds, including niacin. A lack of niacin leads to decreased production of the coenzymes NAD+ and NADP+, which are essential in redox reactions throughout cellular metabolism. Diagnosis is confirmed via 24-hour urine testing for 5-hydroxyindoleacetic acid (5-HIAA), the end product of serotonin metabolism. www.eduwaves360.com | Telegram : @eduwaves360
  • 59. D - Megaloblastic anemia Explanation But Gastric carcinoid tumors can cause pernicious anemia. However, because these tumors produce histamine and not serotonin, they lack some of the typical features of metastatic intestinal carcinoids that are present here (e.g., diarrhea, carcinoid heart disease). Explanation Why Megaloblastic anemia is not caused by carcinoid syndrome but is usually due to folate deficiency or vitamin B12 deficiency and is commonly associated with malnutrition or malabsorption. Specific causes for folate deficiency include the use of folate antagonists, whereas vitamin B12 deficiency is seen in pernicious anemia or after gastrectomy. Although malabsorption can occur in carcinoid syndrome, it is usually not severe enough to cause folate or vitamin B12 deficiency. Another complication is more likely to affect this patient. E - T-cell lymphoma Explanation But Enteropathy-associated T-cell lymphoma is a complication of ulcerative jejunitis and adenocarcinoma of the small bowel. Explanation Why Enteropathy-associated T-cell lymphoma is not associated with carcinoid syndrome but is a complication of celiac disease. Although this patient has had weight loss and diarrhea, she lacks other typical manifestations of celiac disease, such as steatorrhea, malabsorption symptoms (e.g., anemia), and dermatitis herpetiformis. Furthermore, celiac disease does not explain this patient's tricuspid murmur, asthma, and flushing. www.eduwaves360.com | Telegram : @eduwaves360
  • 60. Question # 11 A 45-year-old woman comes to the physician because of fatigue, abdominal cramps, watery diarrhea, and a weight loss of 4 kg (8.8 lb) over the last 4 months. She has recently avoided drinking alcohol and eating spicy food because it worsens her diarrhea and causes episodes of heart palpitations and reddening of the face and neck. She takes lisinopril for hypertension. Her temperature is 36.5°C (97.7°F), pulse is 98/min, and blood pressure is 149/90 mm Hg. The abdomen is soft, and there is mild tenderness to palpation with no guarding or rebound. Laboratory studies show an increased urine 5-hydroxyindoleacetic acid concentration. Further evaluation of this patient will most likely show which of the following? Answer Image A Proximal muscle wasting B Achlorhydria C Adrenal medullary mass D Purple-red colored urine E Multiple peptic ulcers F Pulmonic valve stenosis www.eduwaves360.com | Telegram : @eduwaves360
  • 61. Hint This woman has carcinoid syndrome, which manifests with abdominal cramps, diarrhea, and episodic cutaneous flushing and palpitations triggered by food ingestion. The diagnosis is confirmed by laboratory testing that shows an increased urine concentration of 5-hydroxyindoleacetic acid, which is a product of serotonin metabolism. www.eduwaves360.com | Telegram : @eduwaves360
  • 62. Correct Answer A - Proximal muscle wasting Explanation Why Proximal muscle wasting is classically seen in patients with Cushing syndrome, which can manifest with facial plethora and hypertension. However, this condition does not typically cause weight loss or diarrhea. In addition, this patient does not have the classic Cushingoid appearance (e.g., moon facies, truncal obesity, and buffalo hump), purplish skin striae, or easy bruising. Moreover, laboratory findings in Cushing syndrome include increased serum cortisol concentrations, not increased urine 5-hydroxyindoleacetic acid concentration. B - Achlorhydria Explanation Why Achlorhydria is seen in patients with VIPoma, a condition that manifests with watery diarrhea, tea- colored stools, and hypokalemia. Although 20% of patients experience flushing symptoms, they typically do not have abdominal pain. Moreover, VIPoma is associated with increased serum concentrations of vasoactive intestinal peptide, not increased urine 5-hydroxyindoleacetic acid concentration. C - Adrenal medullary mass Explanation Why Pheochromocytoma can manifest as an adrenal medullary mass and cause episodic hypertension, palpitations, tachycardia, and sweating as seen in this patient. However, this malignancy does not typically cause abdominal cramps or diarrhea. Moreover, laboratory changes associated with pheochromocytoma include increased urine concentrations of catecholamines and metanephrines, not 5-hydroxyindoleacetic acid. www.eduwaves360.com | Telegram : @eduwaves360
  • 63. D - Purple-red colored urine Explanation Why Purple-red colored urine is observed in patients with acute intermittent porphyria (AIP), which also manifests with abdominal cramps, diarrhea, and, during acute attacks, hypertension and tachycardia. In addition, alcohol can precipitate attacks of AIP. However, affected patients typically experience neurological symptoms (e.g., sensory and motor neuropathy, neuropsychiatric symptoms), which are absent in this case. Moreover, laboratory changes associated with AIP include increased serum and urine concentrations of porphobilinogen, delta-aminolevulinic acid, and porphyrins, not increased urine 5-hydroxyindoleacetic acid concentration. E - Multiple peptic ulcers Explanation Why Multiple peptic ulcers can result from excessive gastrin secretion due to a gastrinoma, which in addition manifests with severe abdominal pain, diarrhea, and weight loss as seen in this case. However, this woman has episodic palpitations and facial flushing triggered by food and alcohol, which are not features of a gastrinoma. Moreover, gastrinoma is associated with increased serum concentrations of gastrin and chromogranin A, not increased urine 5-hydroxyindoleacetic acid concentration. F - Pulmonic valve stenosis Explanation Why Pulmonic valve stenosis is a manifestation of carcinoid heart disease, a condition that eventually affects > 50% of patients with carcinoid syndrome. Carcinoid heart disease typically develops as a result of chronic exposure of the heart to serotonin from liver metastases. Serotonin enters the inferior vena cava and reaches the right side of the heart, where it induces valvular and myocardial fibrosis. This can lead to pulmonary valve stenosis, tricuspid insufficiency, and right-sided heart failure. www.eduwaves360.com | Telegram : @eduwaves360
  • 65. Question # 12 A previously healthy 27-year-old man comes to the physician because of a 3-week history of anxiety, diarrhea, and a 4.1-kg (9-lb) weight loss. On questioning, he also reports that he noticed a painless mass on his left testicle 2 weeks ago. His pulse is 110/min and irregular and blood pressure is 150/70 mm Hg. Examination shows diaphoresis and a fine tremor of the outstretched fingers. Testicular examination shows a 3-cm, firm, nontender mass on the left scrotum that does not transilluminate. This patient's underlying condition is most likely to be associated with which of the following findings? Answer Image A Hydrocele on scrotal ultrasound B Proptosis on exophthalmometry C Elevated serum AFP D Elevated serum TSH E Positive urine metanephrines F Positive urine pregnancy test www.eduwaves360.com | Telegram : @eduwaves360
  • 66. Hint The incidental finding of a testicular mass that does not transilluminate should raise suspicion for testicular cancer. Additionally, the patient's symptoms (anxiety, diarrhea, and weight loss) and other physical examination findings (tachyarrhythmia, hypertension with widened pulse pressure, diaphoresis, fine tremor) are classic features of hyperthyroidism. Paraneoplastic hyperthyroidism is a possible complication of testicular choriocarcinomas. www.eduwaves360.com | Telegram : @eduwaves360
  • 67. Correct Answer A - Hydrocele on scrotal ultrasound Explanation Why Hydroceles are fluctuant swellings of the scrotum that are rarely firm or well-circumscribed and do transilluminate. This patient presents with a firm, non-tender, testicular mass that does not transilluminate, which is not consistent with a hydrocele. Furthermore, hydroceles are not associated with hyperthyroidism. B - Proptosis on exophthalmometry Explanation Why Proptosis is associated with hyperthyroidism due to Graves disease. While this patient's symptoms are suggestive of hyperthyroidism, he has no evidence of Graves disease (e.g., diffuse goiter or pretibial myxedema). Furthermore, Graves disease would not account for his testicular mass. C - Elevated serum AFP Explanation Why Elevated serum AFP levels are seen in men with certain nonseminomatous germ cell tumors, mainly yolk sac tumors and, less commonly, embryonal carcinoma. While this patient's testicular mass is highly suggestive of a testicular tumor, AFP-producing testicular tumors would not account for his symptoms of hyperthyroidism. www.eduwaves360.com | Telegram : @eduwaves360
  • 68. D - Elevated serum TSH Explanation But Central hyperthyroidism would cause elevated levels of serum TSH, but it would not account for this patient's testicular mass. Explanation Why Elevated serum TSH is seen in primary hypothyroidism. This patient has symptoms of hyperthyroidism, which would be associated with low serum TSH. High circulating levels of thyroid hormones in patients with hyperthyroidism suppress secretion of TSH through feedback inhibition. E - Positive urine metanephrines Explanation Why Positive urine metanephrines are seen in pheochromocytoma, which is associated with symptoms of adrenergic overactivity. The classic symptom triad of pheochromocytoma (episodic headache, sweating, and tachycardia) overlaps with the symptoms of hyperthyroidism present in this patient. However, this patient's testicular mass should raise suspicion for testicular cancer, which would not cause urine metanephrines to be elevated. F - Positive urine pregnancy test Explanation But The alpha subunit of HCG is also identical to the alpha subunits of luteinizing hormone (LH) and follicle-stimulating hormone (FSH). Explanation Why A positive urine pregnancy test indicates the presence of elevated human chorionic gonadotropin www.eduwaves360.com | Telegram : @eduwaves360
  • 69. (hCG) in the patient's urine. Testicular tumors can produce elevated circulating levels of hCG. Because the alpha subunits of hCG and thyroid-stimulating hormone (TSH) are identical and the beta subunits of these hormones are structurally very similar, hCG is able to weakly stimulate TSH receptors, resulting in the release of thyroid hormones. In rare cases, conditions with extremely high levels of hCG (e.g., choriocarcinoma, as is the likely diagnosis in this man, or gestational trophoblastic disease in women) may be associated with paraneoplastic hyperthyroidism. www.eduwaves360.com | Telegram : @eduwaves360
  • 70. Question # 13 A healthy 31-year-old woman comes to the physician because she is trying to conceive. She is currently timing the frequency of intercourse with at-home ovulation test kits. An increase in the levels of which of the following is the best indicator that ovulation has occurred? Answer Image A Estrogen B Gonadotropin-releasing hormone C Follicle stimulating hormone D Progesterone E Luteinizing hormone www.eduwaves360.com | Telegram : @eduwaves360
  • 71. Hint The level of this hormone increases right after ovulation. If fertilization and implantation do not occur, the same hormone decreases in concentration, causing the endometrium to be shed. www.eduwaves360.com | Telegram : @eduwaves360
  • 72. Correct Answer A - Estrogen Explanation Why Ovulation is preceded by an increase in estrogen. After ovulation occurs (luteal phase), estrogen levels decline. B - Gonadotropin-releasing hormone Explanation Why Gonadotropin-releasing hormone (GnRH) is essential for the development and maturation of ovarian follicles and stimulation of ovulation. GnRH is secreted in a pulsatile manner and is regulated by estrogen and androgens, which influence the amount and frequency of GnRH pulses. Its plasma peaks do not coincide with ovulation, which means it is not an appropriate indicator of when ovulation has occurred. C - Follicle stimulating hormone Explanation Why During the follicular phase of the menstrual cycle, follicle stimulating hormone (FSH) stimulates the development of several ovarian follicles in the ovaries. FSH levels increase during days 1–7 of the menstrual cycle, approximately 1 week prior to ovulation. The hormone is not directly involved in ovulation and therefore is not an appropriate indicator of when ovulation has occurred. www.eduwaves360.com | Telegram : @eduwaves360
  • 73. D - Progesterone Image Explanation But Progesterone maintains the endometrium and inhibits the secretion of gonadotropins (FSH and LH) throughout the luteal phase of the menstrual cycle. If fertilization and implantation occur, the production of β-hCG maintains the corpus luteum and thus progesterone for the first weeks of pregnancy. Explanation Why Towards the end of the follicular phase of the menstrual cycle, a peak in estrogen levels triggers an LH surge, which induces ovulation. The corpus luteum then begins to produce progesterone. An increase in progesterone levels is therefore an indicator that ovulation has occurred. www.eduwaves360.com | Telegram : @eduwaves360
  • 74. E - Luteinizing hormone Explanation Why Approximately 24–36 hours prior to ovulation, a peak in luteinizing hormone (LH) occurs, which induces the rupture of the Graafian follicle. Following ovulation, the LH level rapidly decreases. Since the LH peak is very short and occurs prior to ovulation, it is not a reliable indicator that ovulation has occurred. www.eduwaves360.com | Telegram : @eduwaves360
  • 75. Question # 14 A 34-year-old woman comes to the physician because of a 3-month history of fatigue and a 4.5-kg (10-lb) weight loss despite eating more than usual. Her pulse is 115/min and blood pressure is 140/60 mm Hg. Physical examination shows warm, moist skin, and a diffuse, non-tender swelling over the anterior neck. Ophthalmologic examination shows swelling of the eyelids and proptosis bilaterally. Which of the following is the most likely cause of this patient's symptoms? Answer Image A Nongranulomatous thyroid inflammation B Thyrotropin receptor autoantibodies C Granulomatous thyroid inflammation www.eduwaves360.com | Telegram : @eduwaves360
  • 76. Answer Image D Parafollicular cell hyperplasia E Thyroid peroxidase autoantibodies F Constitutively active TSH receptor www.eduwaves360.com | Telegram : @eduwaves360
  • 77. Hint This patient presents with clinical findings of hyperthyroidism due to Graves disease, including unintentional weight loss, warm, moist skin, tachycardia, hypertension, painless goiter, and ophthalmopathy. www.eduwaves360.com | Telegram : @eduwaves360
  • 78. Correct Answer A - Nongranulomatous thyroid inflammation Explanation Why Nongranulomatous inflammation of the thyroid occurs in Hashimoto thyroiditis and subacute lymphocytic thyroiditis. Patients with either condition may initially present with an enlarged, painless thyroid gland and signs of hyperthyroidism secondary to transient thyroxine release from destroyed thyrocytes. However, these conditions are not associated with eyelid swelling or exophthalmos. B - Thyrotropin receptor autoantibodies Image www.eduwaves360.com | Telegram : @eduwaves360
  • 79. Explanation But Graves disease is associated with HLA-DR3 and HLA-B8 immunophenotypes. Histological examination of the goiter in Graves disease would reveal tall, follicular epithelial cells with scalloped colloid spaces. Explanation Why Graves disease is caused by thyrotropin-receptor autoantibodies (TRAbs) that activate the TSH receptor. TRAbs induce hyperthyroidism by stimulating follicular epithelial cells to produce abnormally high amounts of triiodothyronine (T3) and thyroxine (T4). As seen in this patient, the metabolically active free T3 increases the basal metabolic rate and lipolysis, which manifests as weight loss, high blood pressure, and increased body temperature. TSH receptor antibodies also bind to orbital TSH receptors and stimulate adipocyte proliferation and orbital fibroblast secretion of glycosaminoglycans. Increased adipocyte count, osmotic muscle swelling, and muscle inflammation cause expansion of retro-orbital tissue, which manifests as exophthalmos and decreased ocular motility. C - Granulomatous thyroid inflammation Image www.eduwaves360.com | Telegram : @eduwaves360
  • 80. Explanation Why De Quervain thyroiditis is associated with granuloma formation and multinucleated giant cells. This condition may initially manifest with symptoms of hyperthyroidism caused by transient thyroxine release from destroyed thyrocytes. However, de Quervain thyroiditis is not associated with eyelid swelling or exophthalmos. Moreover, patients with this condition classically present with tender goiter and jaw pain. D - Parafollicular cell hyperplasia Image Explanation Why Medullary thyroid cancer is caused by hyperplasia of parafollicular cells. Patients with this condition may present with painless swelling of the thyroid. However, medullary thyroid cancer does not manifest with symptoms of hyperthyroidism, and it is not associated with eye changes. www.eduwaves360.com | Telegram : @eduwaves360
  • 81. E - Thyroid peroxidase autoantibodies Image Explanation Why Thyroid peroxidase antibodies prevent the production of thyroid hormone in Hashimoto thyroiditis. Patients with this condition may initially present with an enlarged, painless thyroid gland and hyperthyroidism secondary to transient thyroxine release from destroyed thyrocytes (hashitoxicosis). However, Hashimoto's thyroiditis is not associated with eyelid swelling or exophthalmos. F - Constitutively active TSH receptor Explanation Why Constitutively active thyroid stimulating hormone (TSH) receptors can lead to the development of a toxic multinodular goiter. Although patients with the condition present with classic symptoms of hyperthyroidism, they do not have eyelid swelling or exophthalmos. Moreover, the condition www.eduwaves360.com | Telegram : @eduwaves360
  • 82. manifests with nodular neck swelling, as opposed to the diffuse neck swelling in this patient. www.eduwaves360.com | Telegram : @eduwaves360
  • 83. Question # 15 A 25-year-old woman comes to the physician because of a 4-month history of anxiety and weight loss. She also reports an inability to tolerate heat and intermittent heart racing for 2 months. She appears anxious. Her pulse is 108/min and blood pressure is 145/87 mm Hg. Examination shows a fine tremor of her outstretched hands. After confirmation of the diagnosis, the patient is scheduled for radioactive iodine ablation. At a follow-up visit 2 months after the procedure, she reports improved symptoms but new-onset double vision. Examination shows conjunctival injections, proptosis, and a lid lag. Slit-lamp examination shows mild corneal ulcerations. The patient is given an additional medication that improves her diplopia and proptosis. Which of the following mechanisms is most likely responsible for the improvement in this patient's ocular symptoms? Answer Image A Inhibition of iodide oxidation B Elimination of excess fluid C Decreased uptake of iodine D Replacement of thyroid hormones E Decreased production of proinflammatory cytokines www.eduwaves360.com | Telegram : @eduwaves360
  • 84. Answer Image F Inhibition of peripheral T4 to T3 conversion www.eduwaves360.com | Telegram : @eduwaves360
  • 85. Hint Symptoms of thyrotoxicosis (e.g., anxiety, weight loss, heat intolerance, palpitations, tachycardia, tremor) together with ophthalmopathy (e.g., conjunctival injections, proptosis, lid lag, diplopia) are highly suggestive of Graves disease. Though several drugs are effective in treating symptoms of hyperthyroidism, only one addresses the underlying cause of Graves ophthalmopathy. www.eduwaves360.com | Telegram : @eduwaves360
  • 86. Correct Answer A - Inhibition of iodide oxidation Explanation Why Thioamides (e.g., methimazole and propylthiouracil) inhibit iodine oxidation by blocking thyroid peroxidase. These drugs are used to achieve a euthyroid state in patients with mild hyperthyroidism, pregnant patients (PTU in the first trimester and methimazole in the second and third trimesters), in elderly patients with limited life expectancy, and to rapidly lower circulating thyroid hormone levels prior to radioactive iodine ablation or thyroidectomy. Although thioamides help to control this patient's hyperthyroidism-related symptoms, they are not used in the management of ocular symptoms in Graves ophthalmopathy. B - Elimination of excess fluid Explanation Why Diuretics are used to eliminate excess fluid in patients who are hypervolemic. Although one of the main causes of Graves ophthalmopathy is the accumulation of osmolar glycosaminoglycan and subsequent fluid retention in the orbital tissues, diuretics only have a small effect on orbital edema and thus should only be used as an adjunct to first-line therapy. C - Decreased uptake of iodine Explanation Why Perchlorate competitively inhibits iodoine uptake in the thyroid gland and was previously used to treat hyperthyroidism. However, it is no longer recommended due to the high risk of developing aplastic anemia and nephrotic syndrome. Furthermore, perchlorate is not used in the management of Graves ophthalmopathy. www.eduwaves360.com | Telegram : @eduwaves360
  • 87. D - Replacement of thyroid hormones Explanation Why Levothyroxine is a synthetic thyroid hormone used to treat hypothyroidism. Depending on the dose of radioactive iodine administered and the corresponding extent of thyroid tissue destruction, patients who were previously hyperthyroid might develop hypothyroidism and require levothyroxine following treatment with radioactive iodine. However, levothyroxine is not used in the management of Graves ophthalmopathy. E - Decreased production of proinflammatory cytokines Image Explanation But Radioactive iodine ablation is associated with the development or exacerbation of Graves ophthalmopathy due to the increased levels of anti-TSH receptor antibodies following treatment. www.eduwaves360.com | Telegram : @eduwaves360
  • 88. Explanation Why Glucocorticoids such as prednisolone are the preferred therapy for moderate to severe Graves ophthalmopathy, which results from inflammation and soft tissue enlargement in the orbit. In Graves disease, TSH receptor antibodies stimulate orbital fibroblasts to secrete glycosaminoglycan (GAG) and the adipocytes to proliferate. GAGs are hyperosmolar, drawing fluid out into the interstitial space, which causes edema of the orbital tissues. GAG accumulation, edema, and increased adipocytes increase the orbital pressure and push the globe forwards, causing exophthalmos, compression of the extraocular muscles, ophthalmoplegia (inability to converge eyes, diplopia, difficulty reading), and ocular pain. Glucocorticoids improve these symptoms via their anti- inflammatory and immunosuppressive effects. F - Inhibition of peripheral T4 to T3 conversion Explanation Why Propylthiouracil, glucocorticoids, and certain beta-blockers (e.g., propranolol) inhibit peripheral 5'- deiodinase, which catalyzes the peripheral conversion of prohormone T4 to active T3 (which is 3–5 times more potent than T4). Although these medications help to control this patient's hyperthyroidism-related symptoms, inhibition of the peripheral conversion of T4 to T3 plays no role in the treatment of ocular symptoms in Graves ophthalmopathy. www.eduwaves360.com | Telegram : @eduwaves360
  • 89. Question # 16 A 75-year-old man with a seizure disorder is brought to the emergency department by a friend because of progressive confusion over the past two weeks. He is unable to provide any history. His vital signs are within normal limits. He appears lethargic and is only oriented to person. Oral mucosa is moist. There is no jugular venous distention. A basic metabolic panel shows a serum sodium concentration of 115 mEq/L but is otherwise normal. Serum osmolality is low and antidiuretic hormone level is elevated. X-ray of the chest shows no abnormalities. Which of the following is the most likely cause of this patient’s hyponatremia? Answer Image A Low cardiac output B Insulin deficiency C Aldosterone deficiency D Medication effect E Excess cortisol www.eduwaves360.com | Telegram : @eduwaves360
  • 90. Hint This patient is euvolemic and has a low serum sodium level with high ADH levels, suggesting a diagnosis of SIADH. www.eduwaves360.com | Telegram : @eduwaves360
  • 91. Correct Answer A - Low cardiac output Explanation Why Low cardiac output (e.g., due to congestive heart failure) results in hypervolemic hyponatremia due to low renal perfusion and subsequent activation of RAAS and ADH secretion, which in turn stimulates water (and sodium) reabsorption. This patient lacks other signs of low cardiac output, including pitting edema, pulmonary edema, and/or elevated JVP, making this etiology unlikely. B - Insulin deficiency Explanation Why Hyperglycemia (e.g., from type 1 diabetes mellitus) can cause both hypovolemic and/or dilutional hyponatremia. Hypovolemic hyponatremia is a characteristic feature of diabetic ketoacidosis and occurs due to the osmotic diuresis from hyperglycemia and glucosuria. In dilutional hyponatremia, increased serum osmolality from hyperglycemia causes water to shift from the intracellular to the extracellular space, resulting in a decreased serum sodium concentration. This patient's normal metabolic panel and euvolemia on exam make hyponatremia due to hyperglycemia very unlikely. C - Aldosterone deficiency Explanation Why Aldosterone deficiency (e.g., due to adrenal insufficiency, type 4 RTA, or congenital hypoaldosteronism) can also manifest with hyponatremia and elevated ADH due to decreased sodium resorption in the collecting ducts. This patient does not have characteristic signs or symptoms of aldosterone deficiency, such as hyperkalemia, metabolic acidosis, or hypotension. www.eduwaves360.com | Telegram : @eduwaves360
  • 92. D - Medication effect Explanation Why SIADH is an adverse effect of several medications, including carbamazepine, SSRIs, and cyclophosphamide and is characterized by an inappropriately elevated serum ADH despite low plasma osmolality (e.g., the patient is not hypovolemic). Risk factors for developing medication- induced SIADH include older age (> 65 years old), concomitant use of other antiseizure drugs and diuretics, and a history of hyponatremia. Other causes of SIADH include CNS disorders, pulmonary disease, and ectopic production of ADH (e.g., small cell lung cancer). E - Excess cortisol Explanation Why Increased levels of cortisol (e.g., Cushing syndrome) classically manifest with hypernatremia, not hyponatremia. Other characteristic laboratory findings include hypocalcemia, hyperglycemia, and hypokalemia. www.eduwaves360.com | Telegram : @eduwaves360
  • 93. Question # 17 A 78-year-old woman is brought to the physician by her son because of progressive memory loss for the past year. She feels tired and can no longer concentrate on her morning crossword puzzles. She has gained 11.3 kg (25 lb) in the last year. Her father died from complications of Alzheimer disease. She has a history of drinking alcohol excessively but has not consumed alcohol for the past 10 years. Vital signs are within normal limits. She is oriented but has short-term memory deficits. Examination shows a normal gait and delayed relaxation of the achilles reflex bilaterally. Her skin is dry, and she has brittle nails. Which of the following is the most likely underlying etiology of this woman’s memory loss? Answer Image A Thiamine deficiency B Autoimmune thyroid disease C Normal pressure hydrocephalus D Vitamin B12 deficiency E Alzheimer disease www.eduwaves360.com | Telegram : @eduwaves360
  • 94. Hint In the early stages of this condition, patients may present with weight loss, heat intolerance, and fine tremor. www.eduwaves360.com | Telegram : @eduwaves360
  • 95. Correct Answer A - Thiamine deficiency Explanation Why Thiamine deficiency is the cause of Wernicke-Korsakoff syndrome, which can also result in short- term memory impairment. However, this patient lacks other typical findings, including ataxia and nystagmus. Although thiamine deficiency usually occurs in the setting of alcohol use disorder, this woman's history of alcohol use is remote and an unlikely cause of her current symptoms. B - Autoimmune thyroid disease Image Explanation Why Hashimoto thyroiditis is the most common cause of hypothyroidism in the United States and is www.eduwaves360.com | Telegram : @eduwaves360
  • 96. characterized by progressive destruction of thyroid tissue by antithyroid peroxidase antibodies and antithyroglobulin antibodies. Initially, it may manifest with transient hyperthyroidism, before progressing to hypothyroidism. Hypothyroidism results in a generalized decrease of the basal metabolic rate, which can present with weight gain, fatigue, dry skin and brittle nails, along with concentration and memory impairment. C - Normal pressure hydrocephalus Explanation Why Normal pressure hydrocephalus classically presents with the triad of urinary incontinence, memory changes, and a wide-based, slow gait. Although normal pressure hydrocephalus can cause progressive memory loss, it is an unlikely diagnosis in a patient without urinary incontinence and gait disturbances. Furthermore, this condition would not explain her other findings (weight gain, dry skin, brittle nails). D - Vitamin B12 deficiency Explanation Why Vitamin B12 deficiency can cause impaired memory and should be considered in patients with a history of alcohol use disorder. However, this patient has not consumed alcohol in the last 10 years. Moreover, the absence of other classic findings, such as anemia, peripheral neuropathy, and gait abnormalities, make this an unlikely cause in this patient. E - Alzheimer disease Explanation Why Alzheimer disease is characterized by amyloid protein accumulation and also affects short-term memory and cognitive function. Although this patient's positive family history increases her risk, this condition does not account for her other findings, such as weight gain, skin and nail changes, and delayed relaxation phase of deep tendon reflexes. www.eduwaves360.com | Telegram : @eduwaves360
  • 97. Question # 18 A 42-year-old man comes to the physician for a follow-up examination. His blood pressure was 146/91 mm Hg at his appointment 1 month ago; subsequent home blood pressure measurements have ranged from 135/83 mm Hg to 156/96 mm Hg. His blood pressure today is 141/85 mm Hg. Physical examination shows no abnormalities. Pharmacotherapy with lisinopril is initiated. Administration of this drug is most likely to result in decreased activity of which of the following sections of a normal adrenal gland? Answer Image A A www.eduwaves360.com | Telegram : @eduwaves360
  • 98. Answer Image B B C C D D E E www.eduwaves360.com | Telegram : @eduwaves360
  • 99. Hint Lisinopril is an angiotensin-converting enzyme (ACE) inhibitor that blocks the conversion of angiotensin I to angiotensin II. Angiotensin II is the primary regulator of aldosterone production in the adrenal cortex. www.eduwaves360.com | Telegram : @eduwaves360
  • 100. Correct Answer A - A Explanation Why This is the fibrous adrenal capsule, which keeps the organ mechanically stable. Angiotensin II does not regulate the activity of the capsule. B - B Image Explanation Why The zona glomerulosa of the adrenal cortex is the outermost layer of adrenal parenchyma and lies directly beneath the adrenal capsule (A). The cells of the zona glomerulosa are stimulated by angiotensin II to produce mineralocorticoids, most importantly aldosterone. Inhibition of ACE by www.eduwaves360.com | Telegram : @eduwaves360
  • 101. lisinopril leads to decreased availability of angiotensin II, in turn resulting in decreased activity of the zona glomerulosa. The response of the zona glomerulosa to angiotensin II is attenuated by low serum potassium or high serum sodium levels. C - C Explanation Why The zona fasciculata is the middle portion of adrenal cortical parenchyma. The cells of the zona fasciculata are stimulated by adrenocorticotropic hormone (ACTH) to produce glucocorticoids, including cortisol. Inhibition of ACE and subsequent changes in angiotensin II levels are not involved in ACTH secretion. The activity of the zona fasciculata is therefore not affected by lisinopril therapy. D - D Explanation Why The zona reticularis is the innermost portion of the adrenal cortical parenchyma. The cells of the zona reticularis are stimulated by adrenocorticotropic hormone (ACTH) to produce androgens, including androstenedione and dehydroepiandrosterone. Inhibition of ACE and subsequent changes in angiotensin II levels are not involved in ACTH secretion. The activity of the zona reticularis is therefore not affected by lisinopril therapy. www.eduwaves360.com | Telegram : @eduwaves360
  • 102. E - E Image Explanation Why The adrenal medulla is the innermost part of the adrenal gland and contains chromaffin cells that secrete norepinephrine and epinephrine when triggered by the cholinergic sympathetic preganglionic nerve fibers lying in close proximity. The inhibition of ACE and the subsequent changes in angiotensin II levels do not directly affect acetylcholine stimulation of the adrenal medulla. www.eduwaves360.com | Telegram : @eduwaves360
  • 103. Question # 19 A 51-year-old woman comes to the physician because of a 3-month history of fatigue, increased urinary frequency, and low back pain. She reports frequent passing of hard stools, despite using stool softeners. During this time, she has not been as involved with her weekly book club. Her family is concerned that she is depressed. She has no history of serious illness. She has smoked 1 pack of cigarettes daily for the past 20 years. Her pulse is 71/min and blood pressure is 150/90 mm Hg. Abdominal examination shows right costovertebral angle tenderness. The patient's symptoms are most likely caused by hyperplasia of which of the following? Answer Image A Chief cells in the parathyroid gland B Chromaffin cells in the adrenal gland C Chromophobic cells in the pituitary gland D Parafollicular cells in the thyroid gland E Spindle cells in the kidney www.eduwaves360.com | Telegram : @eduwaves360
  • 104. Answer Image F Kulchitsky cells in the lung www.eduwaves360.com | Telegram : @eduwaves360
  • 105. Hint This patient presents with classic features of hypercalcemia such as fatigue, hypertension, polyuria (“thrones”), abdominal and lower back pain (“groans, bones”), costovertebral angle tenderness (renal “stones”), constipation, and depression (“psychiatric overtones”). www.eduwaves360.com | Telegram : @eduwaves360
  • 106. Correct Answer A - Chief cells in the parathyroid gland Image Explanation Why Parathyroid chief cells are the site of parathyroid hormone (PTH) production and secretion. Under physiological conditions, parathyroid chief cells maintain calcium homeostasis through PTH-mediated increases in bone resorption, renal calcium absorption, and phosphate excretion. This patient's hypercalcemia is most likely caused by excessive production and secretion of parathyroid hormone (primary hyperparathyroidism). The most common causes of primary hyperparathyroidism include parathyroid adenoma (approx. 80% of cases) and hyperplasia (approx. 15% of cases). www.eduwaves360.com | Telegram : @eduwaves360
  • 107. B - Chromaffin cells in the adrenal gland Explanation Why Chromaffin cells in the adrenal gland are the site of catecholamine release (e.g., epinephrine and norepinephrine). Hyperplasia of chromaffin cells is associated with pheochromocytoma, which commonly manifests as hypertension, which is seen here. This patient, however, lacks other classic features of pheochromocytoma, such as paroxysmal headaches, diaphoresis, and palpitations. C - Chromophobic cells in the pituitary gland Explanation Why Chromophobic cell hyperplasia in the pituitary gland is typically seen with non-secretory pituitary adenomas. Destruction of normal pituitary tissue by a large pituitary adenoma can lead to reduced production of FSH/LH, GH, ACTH, TSH, and ADH (panhypopituitarism). Hypothyroidism caused by TSH deficiency could result in fatigue, constipation, depression, and musculoskeletal pain. Diabetes inspidius caused by ADH deficiency would result in increased frequency of micturition due to polyuria. However, this patient's hypertension and abdominal pain are not explained by hypopituitarism, and clinical features of a large pituitary tumor (e.g., bitemporal hemianopsia, headache) are not present. D - Parafollicular cells in the thyroid gland Explanation But Although calcitonin reduces osteoclastic activity and prevents hypercalcemia, an excess of calcitonin is not known to cause hypocalcemia because of the compensatory increase in PTH secretion, which has a greater effect on calcium homeostasis. Rarely, excess calcitonin can result in cutaneous flushing and diarrhea. www.eduwaves360.com | Telegram : @eduwaves360
  • 108. Explanation Why Parafollicular cell hyperplasia is associated with medullary thyroid carcinoma, an aggressive thyroid malignancy. Parafollicular cells (C cells) of the thyroid gland are the site of calcitonin production and secretion. Calcitonin excess, however, is typically asymptomatic. Also, this patient does not have symptoms suggestive of a thyroid malignancy (e.g., thyroid swelling, cervical lymphadenopathy, vocal cord palsy). E - Spindle cells in the kidney Explanation Why Sheets of spindle cells in the kidney can be seen in patients with angiomyolipoma. However, angiomyolipoma is not associated with paraneoplastic hypercalcemia. Renal cell carcinoma, which can cause paraneoplastic hypercalcemia, is typically characterized by polygonal clear cells on biopsy. F - Kulchitsky cells in the lung Explanation Why Kulchitsky cells (K cells) are neuroendocrine bronchial cells that can undergo malignant transformation to become small cell lung cancer (SCLC). SCLC is associated with paraneoplastic syndromes, which could explain some of the features seen in this patient (e.g., fatigue due to paraneoplastic SIADH-induced hyponatremia). However, unlike squamous cell lung carcinoma, SCLC is not associated with paraneoplastic hypercalcemia. www.eduwaves360.com | Telegram : @eduwaves360
  • 109. Question # 20 A 36-year-old woman, gravida 2, para 1, at 30 weeks' gestation comes to the physician for evaluation of increased urinary frequency. She has no history of major medical illness. Physical examination shows no abnormalities. Laboratory studies show an increased serum C-peptide concentration. Ultrasonography shows polyhydramnios and a large for gestational age fetus. Which of the following hormones is predominantly responsible for the observed laboratory changes in this patient? Answer Image A Estrogen B Human placental lactogen C Thyroxine D Human chorionic gonadotropin E Adrenocorticotropic hormone F Progesterone www.eduwaves360.com | Telegram : @eduwaves360
  • 110. Hint An increased serum C-peptide concentration and ultrasound findings of polyhydramnios and macrosomia suggest a diagnosis of gestational diabetes. Hyperglycemia causes osmotic diuresis, which leads to increased urinary frequency. www.eduwaves360.com | Telegram : @eduwaves360
  • 111. Correct Answer A - Estrogen Explanation Why Estrogen may play a role in the development of gestational diabetes in pregnancy via increased insulin resistance, but it is not predominantly responsible for the increased serum C-peptide concentration in this patient. The main functions of estrogen during pregnancy are to promote uterine growth and proliferation and differentiation of mammary glands. B - Human placental lactogen Explanation But Screening for gestational diabetes is recommended for all pregnant women because this condition increases the risk of maternal and fetal complications such as preeclampsia and birth trauma. Most patients are usually asymptomatic. Diagnosis and appropriate therapy of gestational diabetes mellitus can decrease fetal and maternal morbidity. Explanation Why Human placental lactogen causes pancreatic beta-cell hyperplasia and leads to an increase in insulin (and C-peptide) secretion as well as maternal insulin resistance. This ensures adequate glucose availability for the fetus. If maternal pancreatic function does not overcome insulin resistance, patients can develop gestational diabetes. In response to increased serum glucose concentrations, fetal production of insulin increases, which leads to increased fetal growth (macrosomia) as seen in this case. www.eduwaves360.com | Telegram : @eduwaves360
  • 112. C - Thyroxine Explanation Why Thyroxine is not responsible for the increased serum C-peptide concentration in this patient and does not play a role in the development of gestational diabetes mellitus. Thyroxine, produced by the maternal thyroid gland and the fetal thyroid gland after 12 weeks of gestation, is vital for fetal neurologic development and CNS myelination. D - Human chorionic gonadotropin Explanation Why Human chorionic gonadotropin is not responsible for the increased serum C-peptide concentration in this patient and does not play a role in the development of gestational diabetes mellitus. The main function of hCG in early pregnancy is to maintain the corpus luteum, which secretes progesterone. Around 7 to 9 weeks of gestation, the placenta becomes the primary producer of progesterone. E - Adrenocorticotropic hormone Explanation Why Adrenocorticotropic hormone is not responsible for the increased serum C-peptide concentration in this patient and does not play a role in the development of gestational diabetes mellitus. During pregnancy, ACTH stimulates the secretion of glucocorticoids in the maternal adrenal cortex, which promotes surfactant production and the development of striae gravidarum. www.eduwaves360.com | Telegram : @eduwaves360
  • 113. F - Progesterone Explanation Why Progesterone may play a role in the development of gestational diabetes in pregnancy via increased insulin resistance, but it is not predominantly responsible for the increased serum C-peptide concentration in this patient. The main functions of progesterone during gestation are to maintain pregnancy, prevent menstruation, induces closure of the cervix, inhibit uterine contractions, and stimulate the expression of uterine oxytocin receptors prior to delivery. www.eduwaves360.com | Telegram : @eduwaves360
  • 114. Question # 21 A 26-year-old primigravid woman at 25 weeks' gestation comes to the physician for a prenatal visit. She has no history of serious illness and her only medication is a daily prenatal vitamin. A 1-hour 50-g glucose challenge shows a glucose concentration of 167 mg/dL (N < 135). A 100-g oral glucose tolerance test shows glucose concentrations of 213 mg/dL (N < 180) and 165 mg/dL (N < 140) at 1 and 3 hours, respectively. If she does not receive adequate treatment for her condition, which of the following complications is her infant at greatest risk of developing? Answer Image A Restricted growth B Elevated calcium levels C Islet cell hyperplasia D Decreased amniotic fluid production E Omphalocele F Decreased hematocrit www.eduwaves360.com | Telegram : @eduwaves360
  • 115. Hint Glucose readily crosses the placenta, meaning that elevated maternal glucose levels lead to elevated fetal blood glucose and, accordingly, cause physiologic changes that can lead to complications in the newborn infant. www.eduwaves360.com | Telegram : @eduwaves360
  • 116. Correct Answer A - Restricted growth Explanation Why Infants born to mothers with pregestational diabetes, not gestational diabetes, are at risk for intrauterine growth restriction. Infants born to mothers with gestational diabetes are at increased risk for macrosomia due to the anabolic effects of high insulin levels. B - Elevated calcium levels Explanation Why Infants of diabetic mothers are at risk for hypocalcemia, not hypercalcemia. Although the exact mechanism is not fully understood, this type of hypocalcemia may be due to abnormal maternal calcium-phosphorus metabolism, and/or decreased maternal vitamin D concentrations. Infants of diabetic mothers are also at risk for hypomagnesemia due to maternal renal loss of magnesium from glycosuria. These low levels of magnesium can impair parathyroid hormone release, resulting in hypoparathyroidism that may also contribute to hypocalcemia. C - Islet cell hyperplasia Explanation Why Hyperplasia of islet cells, the pancreatic cells responsible for producing insulin, can occur in utero in response to elevated maternal blood glucose levels, e.g., due to in gestational diabetes. This can cause infantile hyperinsulinemia and put the infant at risk for hypoglycemia once the maternal placental transfer of glucose ceases after birth. www.eduwaves360.com | Telegram : @eduwaves360
  • 117. D - Decreased amniotic fluid production Explanation Why Infants born to diabetic mothers have an increased risk for polyhydramnios, or increased amniotic fluid, not oligohydramnios. Fetal hyperglycemia results in fetal polyuria and, thus, increased amniotic fluid production. E - Omphalocele Explanation Why Untreated diabetes during pregnancy can have teratogenic effects, putting infants at increased risk for congenital malformations including hypertrophic cardiomyopathy and small left colon syndrome. They are not, however, at increased risk for omphalocele. Pregestational diabetes (but not gestational diabetes) puts infants at increased risk of caudal regression syndrome. F - Decreased hematocrit Explanation Why Infants born to diabetic mothers are at increased risk for polycythemia (Hct > 65%), not anemia. Fetal hyperinsulinemia results in an increased metabolic demand that can lead to fetal hypoxia and increased RBC production to compensate. Infants with polycythemia are at increased risk for RBC sludging, ischemia, and infarction. www.eduwaves360.com | Telegram : @eduwaves360
  • 118. Question # 22 A 15-year-old girl comes to the physician because of a 2-month history of progressive fatigue and weakness. She also reports recurrent headaches for 2 years, which have increased in severity and frequency. Her blood pressure is 185/95 mm Hg. Serum studies show a morning renin activity of 130 ng/mL per hour (N=1–4), a morning aldosterone concentration of 60 ng/dL (N=5-30), and a potassium concentration of 2.9 mEq/L. Further evaluation is most likely to show which of the following? Answer Image A ACTH-producing growth in the pituitary gland B Increased urinary excretion of metanephrines C Increased 17-hydroxyprogesterone levels D Pleomorphic modified smooth muscle cells in the renal cortex E Involution of zona glomerulosa of the adrenal gland www.eduwaves360.com | Telegram : @eduwaves360
  • 119. Hint The findings of hypertension and elevated morning renin and aldosterone concentrations, in a patient presenting with fatigue, weakness, and headache, is diagnostic of secondary hyperaldosteronism. www.eduwaves360.com | Telegram : @eduwaves360
  • 120. Correct Answer A - ACTH-producing growth in the pituitary gland Explanation Why Cushing disease owing to an ACTH-producing pituitary adenoma can present with hypertension and signs of hypokalemia, as seen in this patient, as cortisol also has mineralocorticoid properties. However, not only would affected individuals likely exhibit additional features of Cushing disease (e.g., central and neck obesity, easy bruising, striae), but they would also have low aldosterone and renin levels in contrast to this patient. B - Increased urinary excretion of metanephrines Explanation Why Elevated urinary metanephrine levels are suggestive of pheochromocytoma, which would also cause hypertension and headache. However, patients would rather exhibit an episodic pattern with asymptomatic episodes in between. Moreover, aldosterone and renin are not affected by pheochromocytoma, as metanephrines are responsible for the classic symptoms. C - Increased 17-hydroxyprogesterone levels Explanation Why Increased 17-hydroxyprogesterone levels can be found in patients with congenital adrenal hyperplasia due to 21-β-hydroxylase deficiency, which leads to insufficient aldosterone synthesis with ensuing salt wasting and hyperkalemia. This patient does not exhibit symptoms of 21-β- hydroxylase deficiency. www.eduwaves360.com | Telegram : @eduwaves360
  • 121. D - Pleomorphic modified smooth muscle cells in the renal cortex Explanation Why The presence of pleomorphic smooth muscle cells in the renal cortex in combination with elevated renin and aldosterone concentrations is highly indicative of a juxtaglomerular tumor. Excessive renin production by these tumor cells stimulates aldosterone release. Consequently, patients exhibit symptoms of secondary hyperaldosteronism. E - Involution of zona glomerulosa of the adrenal gland Explanation Why In secondary hyperaldosteronism, elevated renin levels ultimately induce excessive aldosterone production through the RAAS. Therefore, the zona glomerulosa would rather be hypertrophied, not involuted, in this patient. www.eduwaves360.com | Telegram : @eduwaves360
  • 122. Question # 23 A 55-year-old woman with type 2 diabetes mellitus comes to the physician for evaluation of worsening tingling of her feet at night for the last 6 months. Two years ago, she underwent retinal laser photocoagulation in both eyes. She admits to not adhering to her insulin regimen. Her blood pressure is 130/85 mm Hg while sitting and 118/70 mm Hg while standing. Examination shows decreased sense of vibration and proprioception in her toes and ankles bilaterally. Her serum hemoglobin A1C is 11%. Urine dipstick shows 2+ protein. Which of the following additional findings is most likely in this patient? Answer Image A Dilated pupils B Incomplete bladder emptying C Increased lower esophageal sphincter pressure D Resting bradycardia E Loss of satiety F Hyperreflexia www.eduwaves360.com | Telegram : @eduwaves360
  • 123. Hint This patient has clinical and laboratory evidence of poorly-controlled diabetes mellitus, including elevated HbA1C, peripheral neuropathy, nephropathy, and retinopathy. The presence of postural hypotension in this patient suggests diabetic autonomic neuropathy, which is a subtype of diabetic neuropathy that results from poorly controlled diabetes and is characterized by damage to small fiber autonomic nerves. www.eduwaves360.com | Telegram : @eduwaves360
  • 124. Correct Answer A - Dilated pupils Explanation Why Diabetic autonomic neuropathy may lead to impaired dark adaptation and miosis from sympathetic denervation. Mydriasis would not be expected in this patient. B - Incomplete bladder emptying Explanation Why Genitourinary manifestations of diabetic autonomic neuropathy include incomplete bladder emptying, urinary retention, erectile dysfunction, retrograde ejaculation, and dyspareunia. The loss of afferent and efferent autonomic innervation of the bladder results in the inability to sense a full bladder and incomplete emptying, predisposing patients to overflow incontinence and recurrent UTI. C - Increased lower esophageal sphincter pressure Explanation Why Esophageal dysmotility, one of the gastrointestinal complications of diabetic autonomic neuropathy, typically presents with decreased lower esophageal sphincter (LES) pressure and subsequent GERD. Hence, increased LES pressures would not be expected in this patient. www.eduwaves360.com | Telegram : @eduwaves360
  • 125. D - Resting bradycardia Explanation Why Cardiovascular autonomic neuropathy, which often affects patients with long-standing diabetes, can present with resting tachycardia (due to unopposed sympathetic nerve activity), orthostatic hypotension, syncope, arrhythmias, ischemia, or sudden death. Resting bradycardia is not a common presentation of diabetic autonomic neuropathy. E - Loss of satiety Explanation Why Gastrointestinal manifestations of diabetic autonomic neuropathy include esophageal dysmotility, gastroparesis, chronic diarrhea, and constipation. Patients with gastroparesis present with nausea, vomiting, bloating, early satiety, abdominal pain, and weight loss. Hence, loss of satiety would not be expected with this patient. F - Hyperreflexia Explanation Why Diabetic polyneuropathy affects large myelinated and small myelinated and unmyelinated fibers, leading to impaired proprioception, vibration, pain, touch, and temperature sensations. Patients present with early symmetric sensory loss and distal hyporeflexia, followed by intrinsic muscle atrophy and weakness. Hence, hyperreflexia would not be expected in this patient. www.eduwaves360.com | Telegram : @eduwaves360
  • 126. Question # 24 A 3200-g (7.1-lb) female newborn is delivered at 38 weeks' gestation to a 24-year-old woman. The mother had regular prenatal visits throughout the pregnancy. The newborn's blood pressure is 53/35 mm Hg. Examination in the delivery room shows clitoromegaly and posterior labial fusion. One day later, serum studies show: Na+ 131 mEq/L K+ 5.4 mEq/L Cl− 102 mEq/L Urea nitrogen 15 mg/dL Creatinine 0.8 mg/dL Ultrasound of the abdomen and pelvis shows a normal uterus and ovaries. Further evaluation of the newborn is most likely to show which of the following findings? Answer Image A Decreased dehydroepiandrosterone B Increased 17-hydroxyprogesterone www.eduwaves360.com | Telegram : @eduwaves360
  • 127. Answer Image C Increased 11-deoxycorticosterone D Increased bicarbonate E Increased corticosterone F Decreased renin activity www.eduwaves360.com | Telegram : @eduwaves360
  • 128. Hint This female newborn presents with hypotension, ambiguous external genitalia (clitoromegaly with labial fusion), hyponatremia, and hyperkalemia, which is characteristic of the salt-losing form of 21β-hydroxylase deficiency. www.eduwaves360.com | Telegram : @eduwaves360
  • 129. Correct Answer A - Decreased dehydroepiandrosterone Explanation Why Dehydroepiandrosterone (DHEA) is a direct precursor for estrogen and testosterone biosynthesis, and therefore, will be elevated (not decreased) in patients with hyperandrogenism due to 21β- hydroxylase deficiency. B - Increased 17-hydroxyprogesterone Image Explanation But Patients with 21β-hydroxylase enzyme deficiency are treated with lifelong glucocorticoid and fludrocortisone replacement therapy. Glucocorticoids correct hypocortisolism and improve www.eduwaves360.com | Telegram : @eduwaves360
  • 130. virilization by reducing ACTH secretion and adrenal hyperandrogenism via negative feedback. Fludrocortisone acts as an aldosterone substitute. Explanation Why Increased levels of 17-hydroxyprogesterone, the physiological substrate for 21β-hydroxylase, confirm the diagnosis of 21β-hydroxylase enzyme deficiency. This enzyme deficiency is the most common cause of congenital adrenal hyperplasia and results in decreased synthesis of mineralocorticoids (e.g., aldosterone) and glucocorticoids (e.g., cortisone). Hypoaldosteronism causes hypotension and hyperkalemia and hypocortisolism leads to an increase in the secretion of ACTH from the pituitary gland. Stimulation of the adrenal cortex by ACTH results in adrenal hyperplasia and an increase in the production of adrenal androgens from steroid precursors such as 17-hydroxyprogesterone, thereby causing virilization in females. C - Increased 11-deoxycorticosterone Explanation Why Increased levels of 11-deoxycorticosterone are seen in 11β-hydroxylase deficiency, a different cause of congenital adrenal hyperplasia, which also presents with female virilization. However, congenital adrenal hyperplasia typically presents with hypertension and hypokalemia rather than hypotension and hyperkalemia. 11-deoxycorticosterone is synthesized by 21β-hydroxylase from progesterone in the aldosterone synthesis pathway and would thus be decreased, not increased, in this patient. D - Increased bicarbonate Explanation Why The lack of aldosterone in patients with 21β-hydroxylase deficiency leads to decreased Na+ -resorption (hyponatremia), decreased K+ -secretion (hyperkalemia), and decreased H+ -secretion (metabolic acidosis). Therefore, serum bicarbonate would be decreased, not increased in this patient, in an effort to buffer the lower pH. www.eduwaves360.com | Telegram : @eduwaves360